Dies ist ein Archiv der Seite Wikipedia:Auskunft. Es enthält alle Abschnitte, die in der Kalender-Woche 05 im Jahr 2022 begonnen wurden.

Möchtest du in einer neuen Diskussion auf dieses Archiv verweisen? Nutze folgenden Link und ersetze ________ durch den Namen des Abschnittes. Die Nummer im Inhaltsverzeichnis gehört nicht dazu:

[[Wikipedia:Auskunft/Archiv/2022/Woche 05#________]]


← vorherige Woche Gesamtarchiv nächste Woche →

Ukraine vs. Russland

Was kann uns die Ukraine im Gegensatz zu Russland bieten? Rein wirtschaftlich sollte sich man doch eher mit den Russen gut stellen. Warum stellt man sich nicht deutlicher auf die Seite Russlands und nimmt endlich Nord Stream 2 in Betrieb?

--2A00:20:B04F:F6E8:98A9:2793:4E9C:9F44 09:29, 1. Feb. 2022 (CET)

Du suchst das Wikipedia:Café. --Rôtkæppchen₆₈ 10:01, 1. Feb. 2022 (CET)
Archivierung dieses Abschnittes wurde gewünscht von: Hier falsch. --Rôtkæppchen₆₈ 10:01, 1. Feb. 2022 (CET)

Ernst Erich Müller (Politiker) (1904–1982), deutscher Politiker (NSDAP), Fremdenlegionär und Spion

Wer ist mit Ernst Erich Müller (Politiker) (1904–1982), deutscher Politiker (NSDAP), Fremdenlegionär und Spion gemeint? --Dr Lol (Diskussion) 07:32, 31. Jan. 2022 (CET)

jener.--Mhunk (Diskussion) 09:15, 31. Jan. 2022 (CET)
Danke. 62.157.2.126 13:49, 1. Feb. 2022 (CET)
Archivierung dieses Abschnittes wurde gewünscht von: 62.157.2.126 13:49, 1. Feb. 2022 (CET)

Dialoge in Spielfilmen

Müssen die Filmschauspieler heutzutage immer noch ihre Texte auswendig wissen oder gibt es da elektronische Hilfsmittel? Ich hatte erst den Einfall, dass die Texte nachvertont werden, aber das geht nicht wegen den Lippenbewegungen ... --95.114.105.55 05:41, 31. Jan. 2022 (CET)

Die Texte werden mittlerweile via Kontaklinsen direkt auf die Netzhaut projeziert. Dank digitaler Nachbearbeitung sieht man die Antennen die den Schauspielern aus der Fontalle ragen im fertigen Film auch kaum... Suchtestt du nach sowas in der Art? -Ani--46.114.156.33 07:55, 31. Jan. 2022 (CET)
 
Cue card
Und für Impfgegner, die such weigern, sich einen entsprechenden Chip injizieren zu lassen, gibt es immer noch die guten alten Cue Cards. --Rôtkæppchen₆₈ 08:18, 31. Jan. 2022 (CET)
Ich finde in Manche mögen's heiß die Information nicht, dass Marilyn Monroe ihren Text angeblich abgelesen hat, was man an den Augenbewegungen erkennen soll. Jedenfalls sind Cue Cards nicht beliebt, weil man eben an den Augen erkennen kann, dass der Text abgelesen wird. --BlackEyedLion (Diskussion) 09:11, 31. Jan. 2022 (CET)
(Quetsch) Hier im Video bei 03:56 recht gut zu erkennen. --Karotte Zwo (Diskussion) 12:17, 31. Jan. 2022 (CET)
Anders als im Theater müssen die Schauspieler ihren Text nur für die jeweiligen Szenen kennen. Und diese werden ja oft an unterschiedlichen Tagen gedreht. Meist sind es nur ein paar Sätze, sodass es nicht schwer ist, den Text zu lernen. Ich bin sicher, dass mithilfe von künstlicher Intelligenz schon bald auch hier leicht nachgeholfen werden kann, wenn der Text mal nicht sitzt oder etwas nicht lippensynchron ist. Dann macht die AI es nachträglich lippensynchron. Das funktioniert ja jetzt schon halbwegs gut, wie man an Videos von Obama und Trump sieht, denen irgendetwas in den Mund gelegt wird. Wenn wir noch drei Jahre warten, ist soetwas perfektioniert. Mal sehen, ob man dann überhaupt noch Synchronsprecher braucht. Google Translate und Deepl funktionieren ja jetzt schon immer besser. Und neulich gab es eine Anwendung, bei der man nur ein paar Sekunden sprechen musste und die App hat gelernt, wie die Stimme klingt und konnte sie danach imitieren. Wenn man es weiterdenkt: Braucht man in 10 Jahren überhaupt noch Schauspieler? 80.71.142.166 09:06, 31. Jan. 2022 (CET)
Diese Diskussion lief schon vor gut 100 Jahren: Braucht man nach der Erfindung der Schallplatte überhaupt noch Musiker, oder gibt es schon bald Apparaturen, mit denen die Komponisten die Rillen selbst in die Platten schnitzen können, ohne sich mit Proben und Aufnahmen herumärgern zu müssen? Und als das durch war, war der Tonfilm der neue Feind, der die Kultur zerstört … --Kreuzschnabel 09:44, 31. Jan. 2022 (CET)
Da gute Schauspielleistung nicht darin besteht, einen Text vorzulesen oder herunterzuleiern, sondern eine Rolle zu spielen und dabei einen vorgegebenen Text wie eine ganz natürliche spontane Äußerung der dargestellten Rolle klingen zu lassen: Ja, ich persönlich bin davon überzeugt, dass das ohne Auswendiglernen (besser noch Inwendiglernen, die Englischen haben da mit „to learn by heart“ eindeutig den besseren Ausdruck) des Textes nicht möglich ist. Eine Alternative ist nur Improvisieren, wo es erwünscht ist. Aber ein von wo auch immer abgelesener Text klingt prinzipiell anders als ein in die jeweilige Szene integrierter Text. --Kreuzschnabel 09:49, 31. Jan. 2022 (CET)
Kann zumindest nicht schaden für den Notfall ein paar Schauspieler im Keller zu lagern. Strom kann ja auch mal ausfallen. Mal ernsthaft: Was sollen die Schauspieler denn dann machen? Von ein paar Alkohol- & Drogenexzessen kann man doch nicht anständig leben. Und noch ernsthafter (diesmal wirklich): daran möchte ich nicht denken. -Ani--46.114.156.33 09:48, 31. Jan. 2022 (CET)

Teleprompter lautet die naheliegende Antwort. Wird auch beim Fim eingesetzt, wenn auch wohl seltener als bei TV-Moderationen oder Politikerreden. --FordPrefect42 (Diskussion) 12:32, 31. Jan. 2022 (CET)

Anschlussfrage

Aus persönlichem Interesse, wo wir gerade mal dabei sind: Sagen wir mal, ein Film wird im Original englisch produziert und soll dann deutsch synchronisiert werden. Dafür braucht das Synchronstudio eine Internationale Tonspur (IT), die zwar die Geräusche der Szenen enthält, nicht aber die zu übersetzenden Dialoge. Dafür kann ich mir drei Vorgehensweisen vorstellen: 1.: Die IT wird von einem Geräuschemacher nachträglich erzeugt, d.h. sämtliche Szenengeräusche, Atmo, Vogelzwitschern, Meeresbrandung etc. der deutschen Version sind nicht die am Set aufgenommenen Originalgeräusche, der Film wird vollständig stumm übernommen. 2.: Der Film wird am Set als IT aufgezeichnet, also mit stummen Lippenbewegungen, und auch für die Originalversion anschließend von den Originaldarstellern synchronisiert. 3.: Der Film wird Szene für Szene zweimal aufgezeichnet, einmal mit gesprochenen Dialogen, einmal stumm für den IT, d.h. die in der Synchro zu sehenden Szenen sind nicht dieselben Takes wie die der Originalversion. – Ich halte 1. für die wahrscheinlichste, aber ich weiß es erstaunlicherweise wirklich nicht. --Kreuzschnabel 10:08, 31. Jan. 2022 (CET)

Das Erste ist richtig. en:Foley (filmmaking)#Uses: „Scenes where dialogue is replaced using dubbing also feature Foley sounds.“ Siehe auch music and effects track (M&E track) oder sound effects track, en:Field recording und en:Dubbing (filmmaking)#ADR/post-sync. --BlackEyedLion (Diskussion) 10:29, 31. Jan. 2022 (CET)
Der Artikel Internationale Tonspur ist tatsächlich ausbaufähig was die Entstehung jener Tonspur angeht. Es ist in der Praxis so, dass mit verschiedenen Mikros aufgenommen wird. Am Set interessieren in der Regel hauptsächlich die Dialoge, deswegen die berühmten Mikrofonarme, die über den Köpfen der Schauspieler schweben, aber auch Mikros am Körper, etc. Die sind nicht dafür da, irgendwelche Hintergrundgeräusche aufzunehmen, sondern ausschliesslich die Dialoge. Wenn man die Original-Geräuschkulisse am Set erfassen will, wird das mit anderen, entsprechend platzierten und ausgerichteten Mikrofonen gemacht. Im Tonstudio werden da dann noch weitere Geräusche ergänzt, oder eben gleich alle Hintergrundgeräusche erzeugt - und bei den Dialogaufnahmen eben auch Störgeräusche entfernt. Es kommt auch vor, dass Schauspieler ihre eigenen Aufnahmen nachsynchronisieren müssen, weil die Dialogaufnahme eben doch zu viele Hintergrundgeräusche erfasst hat, aber so weit, dass man eine Produktion gleich von vorneherein auf dein Szenario 2 oder 3 auslegt, geht man nicht. --Karotte Zwo (Diskussion) 10:34, 31. Jan. 2022 (CET)
In der Praxis gibt es dafür Geräuschexperten. Pferdehufen werden da durch das Klappern halber Kokosnüsse auf dem Tisch nachgeahmt. Manches kommt vom Band. 80.71.142.166 10:59, 31. Jan. 2022 (CET)
Dass es den Beruf gibt, ist mir bekannt, sonst hätte ich ihn nicht verlinkt :) Meine Frage war, ob der IT in Filmproduktionen so oder anders erzeugt wird. --Kreuzschnabel 11:19, 31. Jan. 2022 (CET)
Das wäre mindestens 1a oder sogar 4: Es sind dieselben Takes und zu Teilen auch die originalen Geräusche, wobei eine spezielle IT-Tonmischung dafür sorgt, dass die am Set gesprochenen Originaldialoge nur so leise mit draufkommen, dass sie nach der Synchronisation nicht stören. --Kreuzschnabel 11:00, 31. Jan. 2022 (CET)
"Ich hatte erst den Einfall, dass die Texte nachvertont werden...". Ich habe vor einiger Zeit gelesen, dass das tatsächlich passiert. Teils, weil sich hinterher herausstellt, dass die Stimme von der Aufnahme am Set nicht gut rüberkommt, teils weil man möchte, dass etwas anders betont wird oder etwa erst jetzt auffällt, dass einer der Schauspielenden sich versprochen hat etc. Leider finde ich die Quelle nicht mehr, der das zu entnehmen ist. --77.10.19.205 15:48, 31. Jan. 2022 (CET)
Hatte Karotte Zwo ja bereits erwähnt. Nennt sich übrigens en:Automated dialogue replacement (ADR). --Magnus (Diskussion) 15:51, 31. Jan. 2022 (CET)
Auch hier erklärt und ausbaufähig STEM (die Tonspur besteht aus "Teilspuren".--Wikiseidank (Diskussion) 18:16, 31. Jan. 2022 (CET)
Beides. Meist wird in dialogfreien Szenen der Originalton verwendet, geht dies nicht, dann wird der Ton selbst kreiert, wie etwa Straßenlärm, Stimmengemurmel im Hintergrund etc. In der Regel vom Band, es gibt Geräusche-CDs, DVDs, die auch Hobbyfilmer verwenden, z. B. eine von George Lucas. --5DKino (Diskussion) 18:46, 31. Jan. 2022 (CET)

Unsynchroner Ton

Gelegentlich fällt mir bei manchen YouTube-Videos auf, daß Bild und Ton wohl etwas unterschiedlich schnell laufen oder sonstwie außer Tritt geraten, so daß nach ca. 1 h Zeitverschiebungen von einigen Sekunden vorliegen, um die der Ton dem Bild nachhinkt. Wie kommt sowas eigentlich zustande, d. h. wer hat denn da was, und vor allem wie, vergurkt? --95.116.29.20 12:41, 31. Jan. 2022 (CET)

Haste mal n Link? Generell sind Ton und Bild bei digitalen Videos nicht wie im chemischen Tonfilm fest miteinander verheiratet, sondern zwei separate Streams, um deren Synchronität sich der wiedergebende Player bemühen muss. Wenn er andererseits kaputte Eingangsdaten bekommt, kann er das auch nicht mehr. --Kreuzschnabel 13:45, 31. Jan. 2022 (CET)
Link --95.116.29.20 22:08, 31. Jan. 2022 (CET)
 
Grafikkarte, Framegrabber und TV-Tuner aus den 1990ern
Es kommt aber auch auf Übertragungsweg und Dateiformat an. Es gibt Aufnahmeformate, die Bild, Ton und Untertitel als unabhängige Streams speichern. Da kann es dann bei Quarzfrequenzabweichungen von Framegrabber und Audioadapter zu Abweichungen kömmen. Bei manchen Dateiformaten sind Bild und Ton über einen Zeitcode gekoppelt. Kommt es da zu zu großen Abweichungen, machen Bild oder Ton einen kleinen Sprung, um die Synchronizität wiederherzustellen. Bei Überttragungssstörungen (DVB-S,C,T etc) kann es zu Störungen des Zeitcodes kommen. Bei der Weiterverarbeitung eines gestörten Mitschnittes sind dann Verschiebungen von Bild und ton möglich. Vor 25 Jahren hab ich AVI-Mitschnitte vom Analogfernsehen mit einer Matrox Rainbow Runner gemacht. Da hatte ich immer einen Versatz, der mit der Zeit immer größer wurde. Meine erste DVB-C-Software 2005 hat in regelmäßigen Abständen geknackst, weil sie Bild und Ton nicht synchron bekommen hat. Erst wenn der PC 20 Minuten lief, hörte das auf, war also ein Temperaturproblem. --Rôtkæppchen₆₈ 16:18, 31. Jan. 2022 (CET)

„Verbind“

Was könnte mit „Verbind“ (taucht bei jedem Eintrag auf) gemeint sein https://i.ibb.co/QpryB8y/Ohne-Titel.jpg ? --Dr Lol (Diskussion) 14:10, 31. Jan. 2022 (CET)

Gebinde dürfte gemeint sein ([1]). Laut meinen Infos liegt das Amt Gandersheim (den Ortsnamen lese ich als Dernhausen, aber so einen Ort scheint es nicht zu geben, ich verlese mich also wohl; wie heißt der Ort?) außerhalb des Verbreitungsgebiets des Hallenhauses (Eine Kartierung anhand von Commons-Bildmaterial habe ich vor Zeiten hier mal versucht). Ich weiß nicht, ob in der Region eine Gebind-Bauweise üblich war (oder ob sie im 18. Jahrhundert üblich gewesen ist). --::Slomox:: >< 14:28, 31. Jan. 2022 (CET)
Ja, ein Teil der Dachkonstruktion, Grundlage zur Bezahlung der Zimmerleute ([2]). 62.157.2.126 14:31, 31. Jan. 2022 (CET)
Dannhausen? 2A00:23C6:150B:8101:C2D1:B99D:8A84:5616 15:10, 31. Jan. 2022 (CET)
BK: ::Der Ort ist sehr wahrscheinlich Dannhausen (Bad Gandersheim). --Proofreader (Diskussion) 15:11, 31. Jan. 2022 (CET)
Stimmt, danke. Ich war so fixiert auf mein Dernhausen, dass ich nicht geschnallt habe, dass das ein a ist und Danhausen heißt... --::Slomox:: >< 15:33, 31. Jan. 2022 (CET)

Bundesarchiv Best. DS/A 225 Müller, Ernst

Wo ist die Signatur Bundesarchiv Best. DS/A 225 Müller, Ernst in Invenio https://invenio.bundesarchiv.de/invenio/main.xhtml zu finden? Die Signatur stammt aus http://www.rppd-rlp.de/pkc0213 --Hamburgum (Diskussion) 15:50, 31. Jan. 2022 (CET)

Mit dem Murks invenio lässt sich allzu häufig nichts finden, es dient nur zum Verstecken der Bestände vor möglichen Anfragen oder Benutzern, vgl. Besprechung. --Pp.paul.4 (Diskussion) 18:06, 31. Jan. 2022 (CET)

Cumbre Vieja, La Palma, Geologie

Bekannt ist eine geologische Instabilität an der Westküste La Palmas und „dass sich die Flanke langsam Richtung Meer bewegt“. (Egar Zorn am 1. Oktober 2021 im dlf). Bekannt ist jedoch auch, dass die seriöse Geologie sagt, die Flanke könne und werde zwar irgendwann abrutschen, in naher Zukunft sei das wohl nicht erwartet und dass andererseits (auch hinsichtlich der oft kolportierten Katastrophenszenarios als Folge wie ein Tsunami an der Ostküste der USA) ohnehin bisher keine sicheren Prognosen über einen Zeitpunkt möglich sind. Nun gehe ich davon aus, dass man die Bewegungen der Flanke, die ja messbar sind, während des und nach dem Vulkanausbruch beobachtet hat, weil die Frage der Wirkung des Ausbruchs auf das problematische Phänomen ja von großem Interesse ist. Ich finde aber nirgendwo vorläufige Ergebnisse solcher Beobachtungen. Ist es noch zu früh dafür oder steht so etwas nur in der Fachpresse? --84.58.54.112 19:04, 31. Jan. 2022 (CET)

Ich glaube, wenn so etwas aktuell publiziert wäre, hätten es die Autoren von en:Cumbre Vieja tsunami hazard bemerkt; dort findet sich aber nichts vermerkt. --Pp.paul.4 (Diskussion) 21:21, 31. Jan. 2022 (CET)

Impressumspflicht bei com- und org-Domänen?

Wenn man sich das Impressum von Sonnenstaatland, Psiram, Wikimannia etc. anschaut... gibt es da keine Impressumspflicht, obwohl zur Zielgruppe sicherlich nur Leser in Deutschland gehören? --BanditoX (Diskussion) 22:55, 31. Jan. 2022 (CET)

Der von dir verlinkte Artikel Impressumspflicht hilft da weiter: § 5 Telemediengesetz: „Diensteanbieter haben für geschäftsmäßige, in der Regel gegen Entgelt angebotene Telemedien folgende Informationen leicht erkennbar, unmittelbar erreichbar und ständig verfügbar zu halten“. Die genannten Seiten fallen nicht unter „geschäftsmäßige, in der Regel gegen Entgelt angebotene Telemedien“. -- Chaddy · D   23:43, 31. Jan. 2022 (CET)
Hier ist §18 Staatsvertrag für Rundfunk und Telemedien[3] einschlägig, nicht das Telemediengesetz. --Rôtkæppchen₆₈ 23:54, 31. Jan. 2022 (CET)
Du meinst den Medienstaatsvertrag, der den Staatsvertrag für Rundfunk und Telemedien ersetzt hat. Hm, ja, je nach Auslegung kann man diese Websites dann womöglich als Medienintermediäre ansehen, dann würde tatsächlich der Medienstaatsvertrag greifen. -- Chaddy · D   01:21, 1. Feb. 2022 (CET)
Das mit dem "gegen Entgelt angeboten" (geschäftsmäßig) gilt aber mittlerweile auch als veraltet bzw. eher unbeachtlich. --Grzeszik (Diskussion) 00:00, 1. Feb. 2022 (CET)
Soweit ich das sehe schalten Sonnenstaatland und Psiram keine Werbung. Insofern sollten sie nicht als geschäftsmäßig anzusehen sein. -- Chaddy · D   01:21, 1. Feb. 2022 (CET)
Unter den Begriff „geschäftsmäßig“ fielen früher auch alle stetigen, nicht-gewerblichen Angebote. Ist das heute nicht mehr so? Zum anderen wird auf diesen oben genannten Seiten laufend über andere Personen berichtet. Die betroffenen können sich an keine ladungsfähige Anschrift wenden, da das Impressum fehlt. Liegt hier also ein Pflichtenverstoss vor? Wie könnten die betroffenen Personen dann weiter vorgehen, um den Betreiber zu ermitteln? --BanditoX (Diskussion) 01:41, 1. Feb. 2022 (CET)
Nein, ja, Anzeige erstatten, auf Auskunft klagen. --77.0.249.48 03:38, 1. Feb. 2022 (CET)

Rätselhafte Grabinschrift

Ich habe gerade die diese Dokumentation über den Entführungsfall Orlandi angeschaut. Bei all den absurden Wendungen und Zusammenhängen möchte ich hier nur eine Frage zu einem unscheinbaren und wohl unwichtigen Detail stellen: An den Stellen 28:37 und 32:58 ist deutlich sichtbar die Grabinschrift ENRICO DE PEDIS zu lesen, aber auf der etwas später gezeigten zerbrochenen Steintafel (33:34) von seinem Grab (?) steht dann plötzlich ENRICO DE PEDRIS. Woher kommt das zusätzliche R? Bildmanipulation? Eine zweite Tafel mit Steinmetzfehler? Wieso hat das niemand bemerkt bzw. wird dazu nichts gesagt? Hat jemand eine Idee oder weiß sogar etwas sicheres darüber? --80.218.144.56 03:33, 1. Feb. 2022 (CET)

Die Version mit dem "R" ist nicht "in Stein gemeißelt", sondern eine dem Bild überlagerte Filmschrift, was man u. a. daran sieht, daß alle Buchstaben völlig intakt sind und über die Ränder der vermeintlichen Schäden hinausragen. Da hat sich einfach der Filmproduzent bzw. ein Mitarbeiter vertan. (Ich sehe da übrigens keine zerbrochene Steintafel, sondern ein rissiges Blatt Papier.) --77.0.249.48 05:07, 1. Feb. 2022 (CET)
Sicher? Beim G siehts tatsächlich ein bisschen so aus, als ob der obere Bogen auf dem Hintergrund schwach sichtbar wäre, aber ganz eindeutig scheint mir das nicht, und ansonsten sieht es für mich wirklich so aus, als ob das eine echte zerbrochene Inschrift sein könnte. Aber vorausgesetzt, du hast recht: Warum fabriziert man so etwas, und wie kann man sich bei drei Wörtern dermaßen vertun? --80.218.144.56 05:32, 1. Feb. 2022 (CET)
Der Film ist von http://www.bbfilm.tv/ , du müsstest wegen Fehlern dort fragen. Der Gangster hiess definitiv De Pedis. —MBq Disk 07:08, 1. Feb. 2022 (CET)
Warum schreibt man "G", wenn im Namen gar keines vorkommt? (Warum macht man Fehler? Das habe ich mal eine Kommilitonin - Psychologiestudentin - gefragt. Sie antwortete, das wüßte sie nicht, sie machte keine. Womit sie übrigens recht hatte.) Wie kommt "Verschissmus" auf eine Kranzschleife? --77.0.249.48 07:45, 1. Feb. 2022 (CET)
Und wieso hat nur deutsch.rt.com ein Foto dieser Schleife? So zieht eine Frage die andere nach sich... –MBq Disk 16:30, 1. Feb. 2022 (CET)
Du hast wahrscheinlich den falschen Begriff gegoogelt oder die falsche Suchmaschine genommen. Bei mir hat [4] zahlreiche Treffer gebracht, vor und nach dem Wegklappen des Fehlertextes. --Rôtkæppchen₆₈ 16:35, 1. Feb. 2022 (CET)

Was bedeutet Anametame

Ich bin auf der Suche: was bedeutet Anametame - können sie mir helfen --87.161.62.63 13:23, 1. Feb. 2022 (CET) (nicht signierter Beitrag von 87.161.62.63 (Diskussion) 13:23, 1. Feb. 2022 (CET))

Gibt's nicht. Wo kommt der Begriff her? --Magnus (Diskussion) 13:27, 1. Feb. 2022 (CET)
Anameta ist schonmal eine ausgestorbene Spinnenart.💅...n'est pas? (nicht signierter Beitrag von 89.204.153.169 (Diskussion) 17:25, 1. Feb. 2022 (CET))

Zerbrochener Glaseinsatz für eine Uralt-Thermoskanne, wo bekommt man so was?

Früher konnte man die Glaseinsätze extra nachkaufen. Heute bei Neupreisen um 8 Euro für Metallthermosflaschen anscheinend nicht mehr. Werden die noch (von wem?) erzeugt? Glas, verspiegelt mit Kunststoffhülle?

--Luziwuzi (Diskussion) 18:21, 1. Feb. 2022 (CET)

Bei thermos.eu schon geschaut? --77.119.211.190 19:02, 1. Feb. 2022 (CET)
Ich würde mal bei Alfi (sic!) in Wertheim fragen. --77.0.249.48 19:06, 1. Feb. 2022 (CET)

BILD und Atze Schröder

Was ist im Nachgang aus der Affäre https://www.bild.de/unterhaltung/leute/comedian/hubertus-albers-verliert-vor-gericht-54657886.bild.html geworden? Kann man den Artikel über den Künstler mit Perücke und Sonnenbrille nun überarbeiten? --BanditoX (Diskussion) 23:13, 31. Jan. 2022 (CET)

Nein. Ich denke, dass der Künstler gegenüber Wikimedia bisher in seiner Eigenschaft als Kunstfigur agiert hat und daher nichts zu verändern und sein Bestreben zu respektieren ist, dass er öffentlich möglichst konsequent als Kunstfigur agieren will. Wenn der BILD-Artikel vom 31. Januar 2018 tatsächlich richtig informiert (wovon man nach meinem Eindruck bei diesem Blatt in der Regel noch nie ausgehen konnte), dann bestand ein öffentliches Informationsinteresse an dem bürgerlichen Namen, weil er, wie BILD aus dem noch nicht rechtskräftigen Urteil zitiert „die ,Rangelei‘ mit Niels Ruf nicht als ,Atze Schröder‘, sondern als Hubertus Albers vom Zaune brach.“ Zwei Aspekte sind jedoch bemerkenswert:
Zum einen ist weder irgendein BILD-Artikel noch ein Urteil, dessen Rechtskraft noch nicht eingetreten ist, eine seriöse Grundlage für was auch immer. Vielleicht juckt es viele, den Klarnamen zu verbreiten und das vielleicht erst recht, weil er sich gegenüber Wikipedia dagegen verwehrt hat. Aber genau das geht über das Niveau der Arbeitsweise bei BILD meines Erachtens nicht hinaus und ist nicht weniger erbärmlich. Es ist kein überzeugendes Argument zu erkennen, welche konkreten Interessen eigentlich befriedigt werden und unter Maßgabe welcher ethisch-moralischen, wissenschaftlichen und journalistischen Ansprüche und Leitlinien diese Interessen auch befriedigt werden müssen, wenn bei einem Künstler, der unter seinem Künstlernamen agiert und seinen bürgerlichen Namen nicht öffentlich genannt haben will dieser Wille unterlaufen wird. Der Wert dieser Information für die Gesellschaft ist nicht nur gegen die Rechte der Person sondern auch ihre Wünsche abzuwägen, aber mir ist klar, dass dieser zarte Hauch von Zivilisation und Menschlichkeit über unserer Kultur dafür vielleicht noch viel zu dünn und verletzlich ist. Das gilt auch für mich selbst.
Der zweite Aspekt ist der Charakter dieses Vorgangs als Paradebeispiel für den Streisand-Effekt. Denn so kann der Künstler auf die Dauer nur verlieren. Dass er von Niels Ruf angezeigt wurde war vielleicht nicht vermeidbar. Wir wissen nichts über das, was da als Rangelei bezeichnet wird und wir wissen nicht, was aus der Anzeige von Ruf eigentlich geworden ist. Bis zu diesem Punkt ist der Künstler möglicherweise noch Opfer seiner vielleicht etwas albernen und in einer profitorientierten Informationsgesellschaft schwer durchzuhaltenden Strategie. Dass er sich dann aber für eine Klage gegen ein auflagenstarkes Boulevardmedium entscheidet, das schon seit vielen Jahrzehnten im gesellschaftlichen Diskurs nicht nur als verdummend angesehen wird sondern dessen Publikationsmethoden auch mehrfach als gewissen- und verantwortungslos beschrieben werden, kommt einer Kohlhaasiade gleich. Oder der Künstler war einfach sehr schlecht beraten. Es gibt nicht nur in den USA oder in England sondern auch bei uns Ebenen der Politik und der Publizistik, in denen ein ruinierter Ruf nichts mehr bewirkt und ungeniertes Handeln nicht verhindert. In denen die Tatsache der Berichterstattung mehr zählt als der Tenor der Berichterstattung.
Der verlinkte Artikel ist natürlich die Ohrfeige, die rausgetreckte Zunge, das Nachtreten und die Häme gegen den, der es gewagt hat, sich zu wehren und die Warnung an alle, die mit solchen Gedanken spielen. Faktisch hätte BILD aber auch den Künstler weiterhin an seiner Nase durch den Zirkusring gezogen, wenn er mit seiner Klage gegen das Blatt Erfolg gehabt hätte. Die Namensvermeidungsstrategie des Künstlers erzeugt oder verstärkt ja erst den aufmerksamkeitsökonomischen Marktwert. (Vorsicht! Das soll nicht heißen, das Opfer sei selbst schuld sondern: das Opfer ist so oder so ohnmächtig und das entscheidende Moment liegt nicht in der Verhinderung der Namensverbreitung sondern darin, im Verhältnis zu sich selbst im Reinen zu sein und zu bleiben. Ein klassischer dramatischer Topos.) Wer BILD kauft hat diesem Blatt doch schon ganz andere Klopper nachgesehen. Und Mao sagt, man müsse den Feind besser kennen als sich selbst. Das hat das Handeln des Künstlers offenbar nicht bestimmt, denn das alles müsste der Künstler eigentlich wissen. Das ethisch-moralische Niveau seiner Kunst ist ja von dem Niveau der BILD-Zeitung nicht so weit entfernt und Überschneidungen bei den Konsumentengruppen der beider Protagonisten wären sicher nicht überraschend. Weshalb - als praktisches, das Lernen beflügelnde Beispiel für den unvermeidlichen Streisand-Effekt - sein bürgerlicher Name vielleicht etwas eleganter aber nicht weniger erbärmlich als bei der BILD-Zeitung hier in diesen Beitrag eingestreut ist. Da dürfen jetzt diejenigen ran, die gerne den russischen oder chinesischen Weg gehen. --84.58.54.112 03:55, 1. Feb. 2022 (CET)
Gut argumentiert, wenn auch mit etwas viel Rotwein geschrieben. Im Wechselspiel zwischen dem Idealismus, sich selbst schützen zu können, und einer gnadenlos ausgespielten Medienmacht nehmen Gerichte sicherlich nur eine Zuschauerrolle ein. 80.71.142.166 07:19, 1. Feb. 2022 (CET)
"Sich verwehren" ist unidiomatisch bzw. furniertes Wissen. --77.0.249.48 10:02, 1. Feb. 2022 (CET)
Es wurden hier ja Rechtsmittel von Seiten des Künstlers angekündigt. Was ist denn nun das rechtskräftige Ergebnis?--BanditoX (Diskussion) 14:48, 1. Feb. 2022 (CET)
Was hast du denn dazu bisher recherchiert? --84.58.54.112 03:07, 2. Feb. 2022 (CET)

ab wann gelten Pflichten aus einem Vermächtnis?

Der Erblasser bestimmte in einem Erbvertrag das Vermächtnis eines Wohnungsrechts für den Vertragspartner, der selbst zum Vollstrecker des Vermächtnisses bestimmt wurde. Der Vermächtnisnehmer nahm das Vermächtnis erst einige Monate nach dem Tod des Erblassers konkludent an, indem er das Wohnungsrecht im Grundbuch eintragen ließ. Er wohnte allerdings schon vor dem Tod des Erblassers im fraglichen Objekt, und zwar allein. Zu welchem Zeitpunkt gelten mit dem Vermächtnis verbundene Pflichten wie die Zahlung von Nebenkosten? Ab der konkludenten Annahme des Vermächtnisses, oder ab Tod des Erblassers? -- Seelefant (disk.) 10:23, 2. Feb. 2022 (CET)

Vielleicht solltes du dich mit Benutzer:Anonyme Frage zusammen tuen und Ihr euch einen Anwalt für Erbrecht suchen.--Oberkaffeetante (Diskussion) 10:44, 2. Feb. 2022 (CET)
Liebe Oberkaffeetante, ein Anwalt wird natürlich beauftragt. Wie beim Arzt ist 'ne zweite Meinung aber sicher nicht verkehrt. -- Seelefant (disk.) 10:58, 2. Feb. 2022 (CET)
Es ist sicher klug, neben dem Anwalt als Profi auch völlig unqualifizierte Meinungen aus dem Internet zu befragen. --212.88.147.22 11:44, 2. Feb. 2022 (CET)
Für ’ne zweite Meinung würde ich mir eher einen zweiten Anwalt suchen. Was du hier bekommst, hat im 2felsfall vor Gericht keinen Bestand, was die Frage aufwirft, wozu. Lies bitte oben verlinkten Rechtshinweis, insonderheit den Teil nach dem ersten Komma. --Kreuzschnabel 11:47, 2. Feb. 2022 (CET)
Ich unterstütze die Empfehlung, einen Anwalt einzuschalten, sehr. Ab wann die dem Vermächtnisnehmer auferlegten Pflichten gelten, bestimmt sich nach dem Inhalt und der Auslegung des Testaments. Steht drin, dass der Vermächtnisnehmer in den Genuss des Vermächtnisses nur kommt, wenn er rückwirkend irgend etwas bezahlt, ist die Sache klar. Ansonsten würde ich grundsätzlich davon ausgehen, dass Verpflichtungen ("Auflagen", § 1940 BGB) erst ab Annahme des Vermächtnisses "gelten" - spiegelbildlich zu dem Vermögensvorteil (wohnrecht), der dem Vermächtnisnehmer auch erst mit Annahme eingeräumt wird. 92.79.101.164 11:48, 2. Feb. 2022 (CET)
Wenn im Erbvertrag nichts abweichendes geregelt ist, frühestens ab Eintragung des Wohnrechts ins Grundbuch, denn erst ab dann gilt das Vermächtnis als angenommen. Das heißt aber nicht, dass der Vermächtnisnehmer für die Zeit davor von der Zahlung von Nebenkosten befreit wäre, denn bis zur Annahme des Vermächtnisses haben die Eigentümer einen Anspruch auf Nutzungsentschädigung gegen den geduldeten Bewohner, und der beinhaltet neben einer angemessenen Miete auch die Nebenkosten. -- 2A02:908:121:9900:0:0:0:EED9 12:12, 2. Feb. 2022 (CET)
Ah ja, interessant. Aus welcher Rechtsgrundlage soll sich denn in diesem Fall ein Anspruch der Erben auf "Nutzungsentschädigung" ergeben? Vielleicht kannst Du ja mal die entsprechende Vorschrift nennen. 15:02, 2. Feb. 2022 (CET) (unvollständig signierter Beitrag von 92.79.101.164 (Diskussion) )

Krim-Anschluss an Russland

Wer hat den Wahrheitsgehalt der bei Wiki dargelegten Ereignisse im Krimparlament anlässlich der Abstimmung über einen Anschluss an die Russische Förderation geprüft ???

--2A02:2455:D9C:8500:D43E:B9C4:81D:BDAE 15:18, 2. Feb. 2022 (CET)

Fragst DU allgemein oder hast Du spezielle Bedenken? Bei Ersterem: siehe bspw. hier (die WP:Versionsgeschichte). Bei Zweiterem ist, mit konkreter Frage/Aussage, Dein Frage vermutlich auf Diskussion:Referendum über den Status der Krim besser aufgehoben ...Sicherlich Post 17:03, 2. Feb. 2022 (CET)

Call Recorder App

Hi,

kennt jemand von euch eine Call Recorder-App, die auch zuverlässig mit Android 12 funktioniert? Meine bisherige App BB hat nach dem Android - Update den Geist aufgegeben. Ist nur für absolut legale Nutzung bestimmt. Danke und Grüße

--84.138.86.19 18:17, 2. Feb. 2022 (CET)

Joseph Goebbels Mutter

... hat wohl bis 1953 gelebt. Weiß man, wie sie zum NS und zu der Rolle ihres Sohnes gestanden und ob bzw. wie sie sich dazu jemals öffentlich geäußert hat (etwa nach dem Krieg)?--2A0A:A541:9FFF:0:E50C:B95A:2630:6463 22:16, 31. Jan. 2022 (CET)

Will man's wissen? --77.0.249.48 03:35, 1. Feb. 2022 (CET)
War es nicht Frau Himmler, die Mutter vom Heinrich, die zur Frau Goebbels sagte: „Wissen‘S, mein Heinrich ist ja ein lieber Kerl, aber er ist jetzt mit Leuten zusammen, die sind so bißl unangenehm!“ Oder sagte sie das zur Frau Heisenberg? Jetzt weiß ich es nicht mehr genau. Aber ist es nicht egal, was so eine Mutter dachte? Sippenhaft gibt es nicht bei uns und ab einem gewissen Alter ist jeder selbst für sich verantwortlich. (Wobei berichtet wird, Nazis wie Göring oder Höß seien zu ihrer eigenen Familie geradezu liebevoll gewesen, die Kinder fielen teils wohl später aus allen Wolken, als sie realisierten, was ihre Väter getan. Auf das Urteil von Familienangehörigen kommt es also eher nicht an. Weil befangen.) --Heletz (Diskussion) 08:31, 1. Feb. 2022 (CET)
Hat nicht Goebbels seine Kinder getötet, bevor er selbst Suizid beging? --Digamma (Diskussion) 21:00, 1. Feb. 2022 (CET)
Ja. Seine Frau und er hielten das für fürsorglich.--Blue 🔯 11:24, 2. Feb. 2022 (CET)
Nach Frau Himmler und der Ermordung der Kinder Goebbels wurde nicht gefragt. Gruß --Bohème21 (Diskussion) 12:57, 2. Feb. 2022 (CET)
Ja, die Mütterlichkeit der deutschen Mutter! --Heletz (Diskussion) 08:22, 3. Feb. 2022 (CET)

Warum keine Schienenräumer?

Ich lese gerade eine Pressemitteilung der Bundespolizei, nach der mal wieder irgendwelche verpeilten Jugendlichen Schottersteine auf die Schienen gelegt hatten, was wohl häufiger vorkommt und wegen der fliegenden Steinsplitter beim Überfahren gefährlich ist. Ich würde gerne wissen, warum nicht an der ersten Achse Prallbleche montiert sind, die in ca. 1 cm Höhe über der Schiene vor den Rändern herlaufen und so ein Zeug von den Schienen runterschubsen, damit es nicht überrollt wird. Wenn das im Laufe der Zeit verdellt wird, muß das Anprallstück halt ab und zu mal ausgewechselt werden - kann ja wohl nicht die Welt kosten... --77.0.249.48 12:11, 1. Feb. 2022 (CET)

Gibt es, siehe Schienenräumer. --Rudolph Buch (Diskussion) 12:39, 1. Feb. 2022 (CET)
Schon die reichen aber in der Regel nicht soweit herrunter, dass sie Schottersteine sicher wegkriegen. Nicht wenn sie aus festem Material sind. Das hat praktische Gründe. So die Federung und die Notwendikeit, dass sie beispielsweise auf Entgleisungsvorrichtungen fahren können müssen. Bei dem Mass an Freiraum das dafür notwendig ist, passt einfach ein Schotterstein dazwischen. Die Lössung mit Gummiräumer -wie sie bei Baumaschinen praktiziet wird-, ist nicht praktikabel da die zu schnell verschleisen.--Bobo11 (Diskussion) 12:53, 1. Feb. 2022 (CET)
Habe dazu auf der Diskussionsseite von Schienenräumer etwas geschrieben. Der Fehler ist offenbar, daß die Vorrichtungen am gefederten Wagenkasten befestigt sind. Das wäre aber gar nicht notwendig, die könnten auch am Achslagerträger montiert sein und dann mit sehr viel weniger Abstand von der Schienenoberfläche laufen. --77.0.249.48 13:21, 1. Feb. 2022 (CET)
Wenn es es etwas gibt, dass man bei einem Schienefahrzeug etwas NICHT habe will, dann ist das ungefederte Masse.--Bobo11 (Diskussion) 13:40, 1. Feb. 2022 (CET)
Ja klar, deswegen wurden die Radsätze auch abgeschafft... (So ein angedachtes Anbauteil wiegt vielleicht 20 kg - ja und? Was wiegen Radsatz und Achslager? Eine oder mehrere Tonnen?) --77.0.249.48 14:22, 1. Feb. 2022 (CET)
20kg? Schön wärs, das wird eher um die 200kg wiegen. Denn es solte such ja nicht verbiegen. Ich denke du unterschätzt was so ein Bauteil aushalten müsste. Wenn das auf einer ICE Strecke ist, dann sind das 300+ km/h bei der das Bauteil mit dem Schotterstein kolidieren würde. Da ist definitive nichts mit Leichtbau wenn es formstabil sein soll (Schottersteine die mit derselben Geschwindigkeit auf Köpfe treffen hinterlassen hässliche Löcher). Schlieslich könnte ja noch ein weiter Stein auf den Schienen liegen.--Bobo11 (Diskussion) 14:59, 1. Feb. 2022 (CET)
 
Schienenräumer am Drehgestellrahmen
Bei hohen Geschwindigkeiten ergibt so ein Schienenräumer überhaupt keinen Sinn, da ja hier cdie Gesetze der Physik gelten. Egal, ob der Schotterstein beim Aufprall zersplittert oder nicht: Der Stein oder dessen Bruchstücke werden mit hoher Geschwindigkeit fortgeschleudert und können auch so Schaden anrichten. Insofern ist es egal, ob der Schotterstein gegen den Radreifen oder einen Schienenräumer prallt. Drehgestellrahmen mit Schienenräumer hat es aber durchaus schon gegeben, z.B. bein Maschinenfabrik Esslingen GT4. Der kam aber nur auf 65 km/h bergab mit Rückenwind. --Rôtkæppchen₆₈ 19:49, 1. Feb. 2022 (CET)
Nur darf auch der nur soweit herabreichen, dass Gleiseinrichtungen wie beispielsweise eine Gleissperre nicht in ihrer Funktion gestört werden. Genau da liegt bei Vollbahnen ja das Problem. Die "Auffahrbleche" (Ich mag jetzt nicht suchen gehen wie die bei der DB genannt werden) bei Entgleisungsvorrichtungen sind so dick, dass eben bei festen Schienenräumer so viel Freiraum vorhanden ist, dass nicht mehr alle Schottersteine erwischt werden können. In der Schweiz beträgt das Abstand das jedes feste Fahrzeugteil bei einer LÜ (ausser den Rädern zu den Schienen) zu festen Bauteilen (Ja, dazu gehören auch die Schienen) einhalten muss 50mm (AB EBV, Seite 343). Das heisst ein nicht zugelassenes Schienenfahrzeug (Das iste eine Variante einer aussergewöhnlichen Sendung), dass einen Schieneräumer hat, der weniger als 50mm Abstand zur Schienenoberkante hat, kann in der Schweiz nicht überführt werden. Und ind er Regel ist es so, dass Masse die bei nicht zugelassenen Fahrzeugen einzuhalten sind, sind es erst recht einzuhalten, wenn man für dieses Fahrzeugen eine Zulassung will.
@Rotkaeppchen68 das Drehgestell ist auch bei Vollbahnen der übliche Anbauort für den Schienenräumer. Bitte nicht mit Räumschild verwechseln, der durchaus am Wagenkasten beefestigt sein kann. Denn wenn ein Räumschild bzw. Schneeräumer vorhanden ist, üÜbernehmen nicht selten die Sandrohr die Funktion des Schienenräumers. --Bobo11 (Diskussion) 21:43, 1. Feb. 2022 (CET)
@Bobo11: Du verlinkst mit Sandrohr auf ein Gras? --Windharp (Diskussion) 07:51, 2. Feb. 2022 (CET)
Uppsi, da hat sich hinter dem blauen Link mal wieder ein anderes Wort versteck. Windharp damit ist ein Teil des Sandstreuer gemeint. Die Weiterleitung ist Mist, das Wort ist mehrdeutig. --Bobo11 (Diskussion) 12:48, 2. Feb. 2022 (CET)
Ich hab dem Artikel Land-Reitgras, worauf Sandrohr weiterleitet, mal einen BK-Hinweis auf Sandstreuer verpasst. --Rôtkæppchen₆₈ 15:03, 2. Feb. 2022 (CET)
Mit den "Gesetzen der Physik" ist das auch nicht so in Stein gemeißelt: Angenommen, die Front des Schienenräumers ist eine ebene Fläche, die in Fahrtrichtung zeigt. Dann werden Steinchen auf den Schienen grundsätzlich in Fahrtrichtung geschossen, ganz oder in Bruchteilen. Da können sie wahrscheinlich weniger Schäden anrichten, als wenn Splitter eines überrollten Steins seitlich wegfliegen. (Vermutlich ist das Überrollen sowohl für Schiene als auch Radlauffläche nicht eben ersprießlich und sollte tunlichst vermieden werden - schon deswegen sollte eine Schienenputzvorrichtung eine gewisse Bedeutsamkeit haben. Am Radreifen prallen die mutmaßlich nicht ab, sondern werden in den Spalt zwischen Rad und Schiene eingeklemmt und überrollt, wobei auf jeden Fall etwas kaputtgeht, Stein bzw. sonstiger Fremdkörper, Reifen oder Schiene oder mehreres davon, und es knallt auf jeden Fall beachtlich.) Die Fluggeschwindigkeit ist auch nicht so absolut festgelegt: Wenn die Prallfläche nicht senkrecht steht, sondern ein wenig nach oben geneigt ist, werden die Fremdkörper nach oben geschossen und können dann Fang- und Dämpfungsvorrichtungen am Wagenkasten treffen, die das Geschoß entschärfen. Und letztlich hängt der Betrag der Abprallgeschwindigkeit von Masse und Härte der Prallplatte ab: Das könnte ein nachgiebiges Compoundmaterial sein, in dem anprallende Steinchen steckenbleiben. Ich würde sowas ungefähr so konstruieren: ein nach vorn zeigender Haltearm von vielleicht 2-3 kg Masse, also Größenordnung und Stabilität "Fahrradrahmenrohr", vorne ein relativ weiches Federelement, und an dem ein Prallklotz von 3-5 kg Masse angebaut. Nicht zu große Fremdkörper hinterlassen ihre Treffermarken auf dem Klotz, wobei die Federung verhindert, daß der Haltearm Schaden nimmt. Und wenn's mal heftiger rappelt, weil so ein Vollidiot Pflastersteine oder Betonplatten auf die Schienen gelegt hat (doch, das kommt vor), dann ist die ganze hübsche Konstruktion leider futsch, weil sie abgerissen wird. Na und? Macht nichts, dann wird sie eben ersetzt, Materialkosten im unteren dreistelligen Eurobereich, also Peanuts, und kommt nur selten vor. (Wenn ein Zug Gehwegplatten überfahren hat, dann ist das sowieso teuer, weil erst einmal alle froh sind, daß er nicht entgleist ist, und der zweitens vor einer Untersuchung nicht weiterfährt, sondern in die Werkstatt geschickt wird.) Abstand wegen Entgleisungsvorrichtungen und dergl.? Ja, sorry, Pech für die Vorschriften, die müssen dann leider geändert werden, sprich: da muß man sich dann halt etwas anderes einfallen lassen. Aber vielleicht gibt es auch "smarte" Alternativen zu "klassischen" mechanischen Schienenräumern: Vorstellen könnte ich mir eine schlaue Sensorik mit angeschlossener KI, die den Schienenraum vor dem ersten Fahrzeug überwacht und bei Erkennung von "geeigneten" Fremdkörpern diese von der Schiene wegschießt oder -haut. Es könnte z. B. in einem solchen Fall ein Fallarm vom Wagenkasten heruntergeklappt werden, dessen Ende dann auf der Schiene schleift und mit dem Fremdkörper beabsichtigt kollidiert, der dann nach Bereinigung des Vorfalls automatisch wieder hochgeklappt wird. Wenn dieser bewegliche Fänger genügend massiv ausgeführt ist, könnte er sogar mit "dicken Brocken" wie Pflastersteinen und Gehwegplatten fertig werden. Deren Masse ist schließlich dadurch begrenzt, daß es noch die rahmenfesten Fangeinrichtungen gibt, die kuhfängermäßig im Prinzip alles abräumen, was weiter hoch steht. --77.8.217.248 10:26, 2. Feb. 2022 (CET)
Entscheidend ist, dass sich ein derartiger Riesenaufwand nicht lohnt. Spielmätze, die Schottersteine auf die Schienenfahrflächen legen, gab es schon immer und für den Bahnbetrieb stellen sie schon wegen der Fahrzeugmassen und Achslasten keine Gefahr dar. Weit herunter reichende Bahnräumer aus verhältnismäßig weichem Material müsste man ständig wechseln, ohne dass dem ein spürbarer Nutzen entgegensteht.
Wenn es im Landverkehr ein Gebiet gibt, wo dringend etwas für den Unfallschutz getan werden muss, dann ist das noch immer der Kraftfahrzeugverkehr. Ein guter Anfang wäre eine verpflichtende GPS-gestützte Geschwindigkeitsbegrenzung für alle Kraftfahrzeuge und eine klare Trennung von Rennstrecken und öffentlichen Straßen. –Falk2 (Diskussion) 18:34, 2. Feb. 2022 (CET)
Leider ist es komplett Knülle, ob Du irgendwelche Auskunftsleser überzeugst. Versuche doch lieber Kontakt Siemens Mobility, Kontakt Alstom Deutschland, Kontakt Windhoff Group, Kontakt Schoema -- southpark 18:19, 2. Feb. 2022 (CET)
Ich will niemanden überzeugen. Aber das Argument der Irrelevanz (bzw. des fehlenden Bedarfs) könnte tatsächlich die Antwort sein, im Gegensatz zur "das hat's ja noch nie gegeben, das ist gar nicht zulässig, wo kämen wir denn da hin"-Argumentation. (Übrigens wendet man sich mit Vorschlägen für regulatorische Vorschriften nicht an die Hersteller und auch nicht an die Betreiber, sondern natürlich an den Gesetzgeber bzw. die zuständige Fachbehörde, aber das erreicht PA-erlenpflanzende Mieslinge natürlich nicht.) --77.8.244.100 03:16, 3. Feb. 2022 (CET)

unheimliche PeV-Neutrinos

Könnten die nicht erklärbaren [5] antarktisch gemessenen Petaelektronenvolt-Neutrinos auch als Hinweis auf ein in die finale Fraßphase übergehendes (ex Mini-)Schwarzes Loch im Erdinneren gedeutet werden? --2.247.255.102 23:13, 2. Feb. 2022 (CET)

"Extrem schnelle ... Neutrinos" - ja nee, is' klar... Und was sollen SL mit Neutrinos zu tun haben? Produzieren die welche? "Finale Fraßphase": Die dauert höchstens eine halbe Stunde, nämlich solange, bis die Erdmasse sich ins SL hineingequetscht hat. Da der Artikel aber schon ein paar Jahre alt ist (bzw. überhaupt existiert), ist die Antwort Nein. --77.8.244.100 03:44, 3. Feb. 2022 (CET)
Hier sind einige Überlegungen zu schwarzen Löchern in der Erde: https://www.e-stories.de/view-kurzgeschichten.phtml?49045 -- Karl Bednarik (Diskussion) 07:21, 3. Feb. 2022 (CET).

Temperatur

Welche Arbeitstemperatur und welche Höchsttemperatur haben Konzentrator-Solarzellen bei der 500-fachen Sonnenintensität? -- Karl Bednarik (Diskussion) 07:14, 3. Feb. 2022 (CET).

Kann jemand (alt?)deutsche Schreibschrift lesen?

Guten Abend. Ich wollte fragen, ob jemand hier deutsche Schreibschrift(?)lesen kann und Lust hätte, mir die zu "übersetzen". Ich schaffs nicht und Transkribus auch nicht wirklich :-(
Ist auch kein Roman, sondern zwei A5-Seiten mit nicht soo viel Text.
Hintergrund ist ein spannender Dachbodenfund ... eine Kassette mit allerlei Unterlagen und Feldpost vom Onkel meiner Partnerin. Der Brief ist wohl der Letzte aus dem Lazarett bei Wien(?) und muss von jemand anders geschrieben worden sein (alle anderen sind leserlich) da er laut Brief von seinem Kompaniechef mit Schussbruch rechter Oberarm zum Verbandsplatz gebracht wurde. Ende '44 ist er mit 17 im Lazarett gestorben.
Scans sind hier: https://c.web.de/@334598271899012280/3qizgZnaRLGtV9GI271rrg Schonmal vielen Dank für die Aufmerksamkeit ;-) --Strange (Diskussion) 22:13, 1. Feb. 2022 (CET)

Ich lese:
Reenhauen [?] 16. XI. 44.
Liebe Mutter!
Zuvor die besten Grüße und Küsse.
Erschrecke nicht, daß ich nicht selber schreiben kann aber ich bin am rechten Oberarm verwundet so daß ich selbst nicht schreiben kann.
Mir geht es soweit gut nur das Fieber macht mir zu schaffen aber bis das [sic] ich einen neuen Gips habe wird es schon gehen.
Eben tragen sie mich zum Gipser.
Und wie geht es euch?
Wenn es dir möglich ist liebe Mutter, daß du mir ein bischen [sic] was schickst. Nur wenn es dir möglich ist.
Was macht Vater? Schreibt er?
Sonst weiß ich für heute nichts mehr.
Mit besten Grüßen
Euer Sohn Alfred.
Liebe Frau Grußmann!
Ich alls [sic] die Schwester Ihres Sohnes möchte auch Sie herzlich grüßen.
Will Ihnen mitteilen, daß es ihm ganz gut geht. Nur das Fieber will nicht weg. Aber wenn er einen neuen Gipsverband bekommt wird auch das Fieber nachlassen.
Essen will er auch nicht viel aber ich schaue schon, daß er wenigstens was zu sich nimmt. Die Kamraden [sic] sind auch sehr nett zu ihm geben ihm Kuchen Kompott u s w.
Also in Sorge brauchen Sie nicht sein. Wenn was sein würde möchte [?] ich Ihnen sofort schreiben aber einstweilen ist es ja nicht nötig
Seien Sie recht herzlich gegrüßt
von Ihrer Schwester Friedl
--Jossi (Diskussion) 01:04, 2. Feb. 2022 (CET)
"Nur das Fieber will nicht weg". Scheiß-Krieg! Pardon my French. -- Seelefant (disk.) 10:26, 2. Feb. 2022 (CET)
Kannste laut sagen! Kann mir kaum vorstellen, wie wahnsinnig hart das für die Mutter war. 1. Brief von der Kompanie: Sohn bei Kaba(?) gefallen. 3 Wochen später: Sohn doch nicht tot, nur am Arm verwundet. Noch ne Woche später kam wohl der Brief oben. Dann kam wohl nur noch: am 8.12. für FuV gestorben :-( Und der Vater war seit September 44 vermisst. --Strange (Diskussion) 11:22, 2. Feb. 2022 (CET)
Superklasse! Vielen herzlichen Dank, Jossi! Das vor dem Datum oben soll vermutlich Pressbaum heissen. Da gabs bei Wien ein Reservelazarett. Sieht auch im Wehrpass so aus. Auf der Sterbeurkunde steht Wien. Nochmals Dankeschön! --Strange (Diskussion) 11:03, 2. Feb. 2022 (CET)
Gern geschehen! „Pressbaum“ ist sehr plausibel. Was mich (als medizinischen Laien) gewundert hat, ist der Zusammenhang zwischen Gipsverbandwechsel und Höhe des Fiebers, den beide ganz selbstverständlich annehmen. Dass ein neuer Gipsverband fiebersenkend wirken soll, ist mir noch nie begegnet. Gibt es Mediziner hier, die dazu etwas sagen können ? --Jossi (Diskussion) 12:36, 2. Feb. 2022 (CET)
Hm, in der Kiste sind auch noch Briefe vom Vater aus der Gefangenschaft. Er wurde bei der Befreiung Antwerpens von den Briten einkassiert. In einem schreibt er: "...trotz der Amputation wäre es doch so schön gewesen, wenn Alfred nach Hause gekommen wäre" - Vielleicht wollten sie im Lazarett dem Jungen nicht sagen, dass sie ihm den Arm abschneiden würden. Oder der Mutter nicht :-( --Strange (Diskussion) 13:21, 2. Feb. 2022 (CET)
Es wird nicht so sein, daß ein neuer Gipsverband fiebersenkend wirkte, sondern daß man zwischen dem Wechsel des Gipses die wohl entzündete Wunde ("Schussbruch rechter Oberarm") untersuchen und vielleicht behandeln konnte. 62.157.2.126 13:25, 2. Feb. 2022 (CET)
Ich weiß nicht, ob ich das darf, aber ich kann zu Hilfe bei Übersetzung von deutscher Schreibschrift diese Facebookgruppe sehr empfehlen (kein Interessekonflikt, ich bin dort bloß Mitglied). --= (Diskussion) 12:25, 3. Feb. 2022 (CET)

Nürnberger Spielwarenmesse 1972

Von wann bis wann? ARD behaupted, das Bobby-Car sei am 2.2.1972 vorgestellt worden, der WDR zieht den 5.2.1972 vor. Kann man die Vorstellung konkretisieren? Papa tanzt Tango (Diskussion) 17:21, 2. Feb. 2022 (CET)

Ist das ein Spendenaufruf für Beiträge zur Sammlung nutzlosen Wissens? --77.8.244.100 03:28, 3. Feb. 2022 (CET)
Vom 5. bis 11. Februar 1972 --Nightflight to Venus (Diskussion) 04:37, 3. Feb. 2022 (CET)
Herzlichen Dank! D.h. Samstag bis Freitag - und die Tagesschau hat sich vertan, als sie gestern, am 2.2. bereits 50 Jahre Bobby-Car gefeiert hat. Ich werde sehen, ob ich das einbauen kann. Damit ist die Frage erledigt. Nochmals Danke! Papa tanzt Tango (Diskussion) 10:37, 3. Feb. 2022 (CET)
War denn die Weltpremiere erst auf der Messe oder gab es ggf. schon eine Vorstellung im Vorfeld der Spielwarenmesse?--Vertigo Man-iac (Diskussion) 13:53, 3. Feb. 2022 (CET)
Kluger Gedanke! Die Antwort eines Redakteurs:
Das stimmt, die Spielwarenmesse 1972 war vom 5. bis 11. Februar , aber das Bobby - Car ist laut Unternehmensangaben wohl schon vorher am 2. Februar einem kleinen Branchenkreis vorgestellt worden und das hatte das Unternehmen zum Anlass genommen, um dieses Jubiläum auf den Starttag der ersten wieder in Präsenz stattfindende Spielwarenmesse 2022 bei der Eröffnung feiern zu können. Leider ist daraus nichts geworden. Kompliziert und Corona geschuldet.
Also eine Weltpremiere quasi im Hinterzimmer. Papa tanzt Tango (Diskussion) 18:43, 3. Feb. 2022 (CET)
Ich werte das mit dem "klugen Gedanken" mal als Kompliment. :-) --Vertigo Man-iac (Diskussion) 19:12, 3. Feb. 2022 (CET)

Tool für 3D-Vektor-Rechnerei gesucht

Es gibt in der Gleitschirmszene eine kleine Glaubensfrage über den richtigen Rettungsfallschirm: Die klassische Rundkappe, wie man sie vom Militär kennt, kommt bewährt runter, ist aber nicht steuerbar, kann also an einer gefährlichen Stelle enden. Es gibt Rettungsfallschirme in Rogallo-Form, die einen größeren Teil der Energie in Vorwärtsfahrt umsetzen, und in hinreichendem Maße steuerbar sind, die haben dieses Problem scheinbar nicht.

Schafft der Pilot es durch äußere Umstände (meist störender Haupt-Gleitschirm) jedoch nicht, zu steuern, hat aber trotzdem die Vorwärtsfahrt. Passiert das Malheur nun nahe einer Felswand, besteht also das Risiko, mit der Vorwärtsfahrt gegen die Felswand zu fliegen, während eine Rundkappe einfach an der Felswand entlang abwärts segeln würde.

Ich möchte das Problem nun mit nackter Rechenpower angehen und ein Tool schreiben, das für eine gegebene Retter-Wurf-Koordinate im 3-dimensionalen Raum anhand von Vorwärtsfahrt (fix), Sinkgeschwindigkeit (fix) und horizontaler Richtung (zufällig, 0-360 o.ä.) den Punkt berechnet, an dem der nicht steuernde Pilot das Terrain (idealerweise echtes aus hgt-Daten) trifft. Alle anderen Faktoren lassen wir weg, d.h. ruhige Luft, perfekt gerade Flugstrecke usw. Das ganze soll dann pro Rettungsschirm-Modell zigtausendmal auf verschiedene Punkte angewendet werden, und eine heatmap generieren, wie oft man nun von welcher Stelle aus auf Terrain mit noch zu definierendem ungünstigen Winkel trifft.

Nun meine Wissensfrage: Kann mir jemand ein Tool empfehlen? Meine erste Idee war Unity oder Unreal, aber da müsste ich mich einarbeiten, und ich brauche die fancy Grafik nicht. Ein Python-Modul oder sowas in der Richtung würde auch schon helfen. Ich denke, dass es ähnliche Problemstellungen ständig gibt, aber ich weiß nicht so recht, wonach ich da googeln kann.

Dass ich die Glaubensfrage durch diese Berechnungen nicht abschließend beantworten kann ist mir klar, mir geht es mittlerweile mehr um den Spaß an der Aufgabe.

--2A01:598:88B6:3921:C906:8EF1:89B0:3849 22:24, 2. Feb. 2022 (CET)

Früher hätte ich das wahrscheinlich mit Matlab gemacht. Heute gibt es mehrere FLOSS-Alternativen zu Matlab, siehe Matlab#Alternativen. --Rôtkæppchen₆₈ 22:49, 2. Feb. 2022 (CET)
Matlab (oder alternativen) galt lange als das Tool, mit dem sich am schnellsten entwickeln lässt. Python hat jede Menge Module, also könnte sein dass da schon jemand was geschrieben hat, was du benutzen kannst. Python gilt aber als relativ langsam, (wobei das evtl nicht viel ausmacht - die Rechnung selber hört sich recht simpel an und für die Heatmap reichen wahrscheinlich 10000 Läufe. Das müsste Python auch schaffen. In den letzten 10 Jahren versucht sich Julia als bessere (und erheblich schnellere) Alternative zu Python zu etablieren. 86.159.17.115 12:30, 3. Feb. 2022 (CET)

Stempel auf mobiler Briefmarke?

Werden Briefe, die mit einer „mobilen Briefmarke“ frankiert worden sind (Wenn man es so schreibt, klingt die Bezeichnung komplett bescheuert …), im Zustellungsgang eigentlich gestempelt? Google spuckt mir nur Ergebnisse zur Briefmarke mit Matrixcode („digitale Briefmarke“) aus, gemeint ist aber das #PORTO zum selbst aufschreiben. --91.221.58.22 12:43, 2. Feb. 2022 (CET)

 
Die Post scheint bei der Mobile Briefmarke einfach einen Aufkleber mit QR-Code neben die handschriftlichen Zahlencodes zu kleben (zumindest solange das noch "Handyporto" hieß, ob das jetzt in 2022 immer noch so ist weiß ich nicht). Siehe etwa das Bild.--Naronnas (Diskussion) 12:59, 2. Feb. 2022 (CET)
Briefe mit #PORTO plus Buchstaben werden nicht gestempelt. Quelle: OR. --Studmult (Diskussion) 19:43, 3. Feb. 2022 (CET)
Ein Stempel auf einer mobilen Briefmarke geht rein technisch nicht. Die Stempelmaschinen finden anhand der Fluoreszenz die Position der Briefmarke auf dem Brief oder der Karte und stempeln dort. Briefe und Karten ohne Fluoreszenz werden aussortiert und getrennt behandelt. Bei einer Sendung ohne Marke aber mit handschriftlichem Code wüsste die Maschine gar nicht, wo sie stempeln soll. Bei der Bearbeitung des Briefes wird dieser sowieso fotografiert und die Anschrift per OCR erfasst. Da ist es ohne großen Zusatzaufwand möglich, auch eine handschriftliche Mobilbriefmarke zu erkennen und bei Bedarf dann einen Aufkleber mit Datamatrixcode anzubringen. --Rôtkæppchen₆₈ 09:44, 4. Feb. 2022 (CET)
Unser Artikel schmeißt da die aktuelle mobile Briefmarke und das alte Handyporto durcheinander, die eigentlich nichts mit einander zu tun haben. Bei der mobilen Briefmarke gibt es keinen Aufkleber o.ä., wir haben sogar ein passendes Bild im Artikel wo oben im Eck der "unbehandelte" Code sitzt, man am aufgedruckten Zielcode aber sieht dass er schon zugestellt wurde. --Studmult (Diskussion) 09:57, 4. Feb. 2022 (CET)

Ist Taekwondo effektiv für den Straßenkampf?

Was mich schon immer interessiert ist ob auch Taekwondo für den Straßenkampf tauglich sein kann. Angenommen man hat Taekwondo schon sehr gut drauf und man wird in einer Großstadt im Dunkeln von Schlägertypen attackiert und überfallen, so hätte ich bitte gewusst ob man sich auch allein nur mit Taekwondo Tritttechniken gut verteidigen kann? Was meint ihr? Ist Taekwondo wirklich richtig effektiv wenn es generell zu einer ernsthaften, gefährlichen Situation kommt? Ich bitte um genaue, seriöse Antworten. Mein Anliegen ist ernst. Und ja, ich interessiere mich deswegen für Taekwondo weil Taekwondo eines der günstigsten Kampfsportvereinen in meiner Umgebung ist. 185.155.125.26 11:41, 4. Feb. 2022 (CET)

Kann man sich auch nur mit Taekwondo Techniken gut wehren oder verteidigen, auch wenn man Taekwondo nur einmal in der Woche trainiert? 185.155.125.26 11:42, 4. Feb. 2022 (CET)
Bitte alles Haar genau beantworten. Dankeschön. --185.155.125.26 11:42, 4. Feb. 2022 (CET)
Wikipedia:Auskunft/Archiv/2014/Woche_28#Taekwondo - wie gut kann man man wirklich kaempfen? --Magnus (Diskussion) 11:46, 4. Feb. 2022 (CET)
besser als blöckflötenspielen. -- southpark 12:00, 4. Feb. 2022 (CET)
schlechter als blöckflötenspielen, weil er sich hemmungslos überschätzen und bald an den Falschen geraten wird.--Mhunk (Diskussion) 12:16, 4. Feb. 2022 (CET)
Das verstehe ich nicht. Man soll das Haar beantworten? Und zwar alles Haar? Und dann noch genau? --Digamma (Diskussion) 19:07, 4. Feb. 2022 (CET)
Jetzt sind sieben Jahre vergangenn, und Du hast noch mmer nicht damit begonnen? Ich glaube, in Deinem Alter wird das nichts mehr. --2001:871:F:DBCB:B5DF:90CB:8444:EEDF 12:53, 4. Feb. 2022 (CET)

Einiges findert man auch auf der FAQ-Unterseite:

--Optimum (Diskussion) 13:01, 4. Feb. 2022 (CET)

Tagpfauenauge

Bei mir ist mit dem Brennholz aus dem Holzschuppen in der Wohnung ein Tagpfauenauge aufgetaucht. Was mach ich jetzt mit dem beziehungsweise der mitten im Winter? fz JaHn 11:42, 4. Feb. 2022 (CET)

Wenn Du ihn retten willst, bring ihn einfach zurück in den Holzschuppen. Mehr Tipps unter [6] --2A02:AA13:A141:2600:4FD:BA3:E209:476A 12:34, 4. Feb. 2022 (CET)

Ok, danke für den Link! fz JaHn 13:19, 4. Feb. 2022 (CET)

Anwesende bei den Äußerungen des deutschen Flottenchefs in Indien

Wer waren neben dem Generaldirektor Sujan R. Chinoy (en) die Anwesenden im MP-IDSA (en) bei den Äußerungen des (ehemaligen) Flottenchefs (insbesondere die indischen Fragesteller, aber auch die deutschen Marineoffiziere)? (Video) --Parlamentspoet (Diskussion) 17:00, 4. Feb. 2022 (CET)

Zu den Deutschen ein wenig Hilfe: Links im Bild ist ein Fregattenkapitän, in der Mitte ist ein Kapitän zur See. Die Person links ist bei 58:15 ohne Maske zu sehen. Es handelt sich nicht um Tilo Kalski, den Kapitän der Fregatte Bayern. --BlackEyedLion (Diskussion) 22:18, 4. Feb. 2022 (CET)

Rechtschreibung

Guten Tag,
wird ein Patient in oder auf die oder der Kranken(haus)station aufgenommen/eingewiesen ? Was ist richtig ?
Zusatzfrage : beschreibt Solschenizyn in Krebsstation eine Abteilung innerhalb eines Krankenhauses oder eine in einem gesonderten Pavillon untergebrachte Krankenstation ? Letzteres suggeriert in verschiedenen Sprachen die Übersetzung des Titels (fr/es/it/pt). Besten Dank im Voraus, --Bohème21 (Diskussion) 16:21, 31. Jan. 2022 (CET)

IMO nur nach Gefühl: Auf eine Station aufgenommen und in ein Krankenhaus eingewiesen ...Sicherlich Post 17:10, 31. Jan. 2022 (CET)
Was ist gemeint? Der "geographische" Vorgang ("Sie gehen in die Intensivstation und dort das dritte Zimmer rechts.") oder der administrative Vorgang ("Wir mussten Herrn Müller auf die Intensivstation verlegen." "Auf dieser Krankenhausstation fehlt Personal."). Pte. Salt (Diskussion) 17:39, 31. Jan. 2022 (CET)
Ohne es in vorliegenden Fall genau zu wissen: In sehr vielen Fällen gibt es nicht die eine Präposition, sondern mehrere Varianten sind denkbar. 80.71.142.166 18:48, 31. Jan. 2022 (CET)
Was meinst Du mit "denkbar"? Es soll hier doch wohl um die korrekte Zuordenung gehen und nicht um irgendwelche Gedankenspiele. --88.68.87.67 02:05, 1. Feb. 2022 (CET)
Einen irgendwie gearteten "geographischen" Vorgang, bei dem man in die Intensivstation geht, gibt es nicht. Das hat Pte. Salt sich ausgedacht. --88.68.87.67 02:05, 1. Feb. 2022 (CET)
<quetsch> Ganz klares Jein! Bei denen wäre ich lieber "drin" als "drauf". scnr -- Nightflight to Venus (Diskussion) 11:02, 1. Feb. 2022 (CET)
Auch hier gilt: im Flugzeug, auf Station, im Bett. --88.68.87.67 03:25, 2. Feb. 2022 (CET)
Ach, ich will nicht mit dir streiten, im Normalfall hast du ja Recht. Aber diese mobilen resp. fliegenden Intensivstationen bilden die Ausnahme. "Der Patient wurde in einer fliegenden Intensivstation ins Heimatland überführt; die Anschlagsopfer wurden in einer mobilen Intensivstation erstversorgt". Mit der Hoffnung, dass wir es niemals nötig haben, -- Nightflight to Venus (Diskussion) 04:09, 2. Feb. 2022 (CET)
Dann meinetwegen speziell zu diesem Sonderfall: Bei einer "mobilen resp. fliegenden Intensivstation" kann im Flugzeug zu in der fliegenden Intensivstation werden, wenn es sich um ein und dasselbe Objekt handelt und es keine weiteren Stationen gibt, zwischen denen zu differenzieren ist. --178.4.180.189 02:01, 4. Feb. 2022 (CET)
Aus dem Web: "Wenn Sie als Besucher in die Intensivstation [hinein] kommen, so werden Sie unter Umständen mit einem ungewohnten Erscheinungsbild oder mit einem ungewohnten Verhalten des Patienten konfrontiert sein."
Nein, ich habe mir vorher Beispiele angesehen. Ich sehe diesen Unterschied ähnlich wie das englische in/into, also ein Zusatz von "hinein" ist möglich. Oder auch "mitten" wie in Weisser Kittel, blaue Maske: Chefarzt Stephan Jakob steht mitten in der Intensivstation des Berner Inselspitals. Ich gebe zu, dass es nicht immer exakt verwendet wird. Pte. Salt (Diskussion) 10:52, 1. Feb. 2022 (CET)
Man kann für jeden noch so bekloppten Sprachfehler Anwendungsbeispiele finden. Dadurch wird der Fehler nicht richtiger. Recherche ist ganz offenbar nicht Deine Stärke und eigene Erfahrungen im Medizinbetrieb hast Du ganz offensichtlich auch keine, also halte Dich doch bitte bei der Beantwortung dieser Wissensfrage zurück mit Deinen zusammengegoogelten Ansichten. --88.68.87.67 03:25, 2. Feb. 2022 (CET)
Richtig ist: keine Leerzeichen vor mehrteiligen Satzzeichen. --95.116.29.20 21:43, 31. Jan. 2022 (CET)
Kannst du das bitte genauer sagen? Geht es um den Doppelpunkt?--Blue 🔯 22:52, 31. Jan. 2022 (CET)
Wie gut das Leerzeichen keine Satzzeichen sind... -Ani--46.114.155.76 06:49, 1. Feb. 2022 (CET)
"Das Leerzeichen" ist richtig - "Zeichen" hat das neutrale Genus. Ansonsten: Satz kein Satz und völlig wirr. --77.0.249.48 10:29, 1. Feb. 2022 (CET)
Der Artikel Leerzeichen steht nicht ohne Grund in der Kategorie:Satzzeichen, denn es muß aktiv mittels eines Spatiums gesetzt werden, wenn ein Leerzeichen im fertigen Text erscheinen soll. --88.68.87.67 03:25, 2. Feb. 2022 (CET)
... und nicht ohne Grund wird zwischen Satz- und Leerzeichen unterschieden - wenn es zum Beispiel um die Regelung geht mit denen ein Satz beendet wird -> Kein Leerzeichen vor einem Satzzeichen bzw Leerzeichen nach einem Satzzeichen. Heißt also genau genommen "kein Leerzeichen vor einem Leerzeichen"? Bzw "immmer eine Leerzeichen nach einem Leerzeichen"? Come on! -Ani--46.114.155.195 18:44, 2. Feb. 2022 (CET)
Worauf beziehst Du Dich konkret? Antwort bitte ausnahmsweise mal mit Beleg, wenn Du kannst. --88.68.87.67 22:27, 2. Feb. 2022 (CET)
Wenn du's unbedingt brauchst um's nachvollziehen zu können, bitte sehr - https://www.tastschreiben.de/p0400040.htm Eines von x Beispielen zur Din-Norm 5008, in dem zwischen Satz- und Leerzeichen unterschiedem wird. Ein Leerzeichen bzw eine -stelle ist ein Formatierungszeichen, kein Satzzeichen. -Ani--46.114.155.195 00:17, 3. Feb. 2022 (CET)
Na, ob "TASTstar direkt" WP:BEL konform ist, möchte ich doch mal bezweifeln. Aber egal.
Zu "DIN 5008: Satzzeichen": Eine DIN-Norm ist ein unter Leitung des DIN Deutschen Instituts für Normung erarbeiteter freiwilliger Standard […] Die Norm DIN 5008 bietet Schreib- und Gestaltungsregeln für die Text- und Informationsverarbeitung. Wie kommst Du darauf, daß der TO hieran gebunden ist?
Zu Satzzeichen/Leerzeichen siehe Satzzeichen#Die häufigsten Satzzeichen: Das Leerzeichen ( ) steht zwischen zwei Wörtern und nach verschiedenen Satzzeichen wie Punkt oder Komma. Damit dürfte das geklärt sein. --178.4.180.189 02:01, 4. Feb. 2022 (CET)
Da du Leerzeichen verlinkt hast, gehe ich mal davon aus das du den Artikel nicht wirklich auf daraufhin unter die Lupe genommen hast ob er in irgendeiner Weise untermauert das ein Leerzeichen auch tatsächlich ein Satzzeichen ist, wenn du meinst es sei alles klar nur weil Leerzeichen im Artikel Satzzeichen als Satzzeichen gelistet ist. Btw, ich schrieb "einer von vielen Links", soll heißen, einen besseren Link als Beleg zu finden ist annehmbar. Hingegen ist Slbstreferenzierung mit dem Satzzeichen-Link auch fragwürdig. Mein Hinweis auf den Leerzeichen-Link ist also auch nur ein Hinweis darauf das hier nicht alles klar sein kann, wenn zwei Infos aus der Quelle WP nicht decken das ein Leerzeichen ein Satzzeichen ist. Leerzeichen regt zumindest recht umfangreich dazu an das zu bezweifeln. Satzzeichen ist eher eine unbedarfte Sammlung ohne Referenzen im Abschnitt "häufigste Satzzeichen. Gruß, -Ani--46.114.155.244 08:20, 4. Feb. 2022 (CET)
"Leerzeichen" steht im Artikel Satzzeichen und in der Kategorie:Satzzeichen. Das ist keine Selbstreferenzierung, da sich das nicht auf ein Werk von mir, sondern auf die WP bezieht. Zu was Dich das anregt, ist an dieser Stelle und auch ansonsten irrelevant. --94.219.22.201 01:37, 5. Feb. 2022 (CET)
Naja. Der Abschnitt indem es unter Satzzeichen gelistet ist ist unreferenziert. Im gleichen Artikel fehlt es in der Auflistung in der Einleitung. Man könnte sich mal fragen wieso es denn nicht auch im Artikel Leerzeichen steht, welcher ja speziell dem Leerzeichen gewidmet ist. Wenn, dann sollte man die Information Leerzeichen = Satzzeichen doch auf jeden Fall dort finden, aber da ist Fehlanzeige. In beiden Artikeln findet man aber als Einzelnachweis den Link zu einem PDF von Karl Beinz Best. Dieses PDF gibt Untersuchungen zu der Häufigkeit von Schriftzeichen wieder. Und siehe da: Best fasst Buchstaben, Leerzeichen und Satzzeichen unter Schriftzeichen zusammen.

https://www.ram-verlag.eu/wp-content/uploads/2018/08/g11zeit.pdf#page=12 (Größe ca 1,57 MB)
Ich weiß, Artikelarbeit ist bei dir selten bis gar nicht eingeplant (du stänkerst lieber unter 77er/88er/94er-IP im Meta, manchmal sogar mit guten Ansätzen was die Recherchebereitschaft betrifft), deswegen hier der Tip sich sachlicher zu orientieren und auf Abwertungen ad hominem grundsätzlich zu verzichten. Sowas verbaut mitunter die Sicht auf die Dinge. Vielleicht ist der Hinweis auf Quellenreferenzierung ja ein hilfreicher Ansatz es mal anders zu versuchen, und Fehler in der WP zu korrigieren statt sie ungeprüft zu wiederholen. Ansonsten gilt deine untenstehende Aufforderung selbstverständlich auch für dich mit. Gruß mir den Ponyhof, -Ani--46.114.152.4 08:19, 5. Feb. 2022 (CET)

Geht woanders spielen. --88.68.87.67 02:05, 1. Feb. 2022 (CET)
Korrekt ist: im Krankenhaus, auf Station, im Zimmer. --88.68.87.67 02:05, 1. Feb. 2022 (CET)

Guten Tag, vielen Dank für die Aufmerksamkeit, Beteiligung und richtige Antwort, zu der ich inzwischen auch einen Nachweis gefunden habe. Es sei für das Mitmachen auch jenen gedankt, die ungefragt wenig hilfreiche vom Thema abweichende Diskussionen führen, mit dem freundlichen Hinweis, das dies in der Auskunft wohl fehl am Platze ist und unterlassen werden sollte. Das Leerzeichen setzt ein aus guten Gründen auf Standardsprache Französisch eingestellter PC automatisch und es möge mir gnädigst vergeben werden, dies nicht korrigiert zu haben. Beste Grüße und frohes Schaffen --Bohème21 (Diskussion) 12:01, 2. Feb. 2022 (CET)

Ein Blick in den NGram Viewer sagt oft mehr als tausend Worte: in/auf die Krankenstations in/auf die Intensivstation in/auf die Notfallstation. --Megalogastor (Diskussion) 01:18, 3. Feb. 2022 (CET)
Bei dermaßen wenig Treffern hat das wenig Aussagekraft. Außerdem sind Deine Beispiele weitestgehend ungeeignet, weil "Krankenstation" und "Notfallstation" völlig ungebräuchlich sind, weshalb man darunter vorwiegend Nonsens finden wird. Einzig "Intensivstation" wird regelmäßig in dieser Form verwendet. Für die Verwendung des Ausdrucks "Krankenstation" gibt es nur selten einen Anlaß, weil in einem Krankenhaus alle Stationen "Krankenstationen" sind, so daß der Ausruck "Krankenstation" zu einer sinnfreien Tatologie verkommt, weil die notwendige Unterscheidung zwischen den verschiedenen Fachrichtungen fehlt. Tatsächlich kommt der Patient "auf Intensiv", "auf die Innere" o.ä. Eine "Notfallstation" gibt es i.d.R. nicht, aber sehr wohl eine Notaufnahme, für die dann tatsächlich gilt, daß der Patient "in die Notaufnahme" geht. So viel dazu in aller Kürze. Recherche ist ganz offenbar nicht Deine Stärke und eigene Erfahrungen im Medizinbetrieb hast Du ganz offensichtlich auch keine, also halte Dich doch bitte bei der Beantwortung dieser Wissensfrage zurück mit Deinen zusammengegoogelten Ansichten. --178.4.180.189 02:43, 3. Feb. 2022 (CET)
Krankenstationen gibt es wahrscheinlich nicht in Krankenhäusern, aber z.B. in Gefängnissen und in Kasernen. --Digamma (Diskussion) 20:12, 3. Feb. 2022 (CET)
In Star Trek gibt es sie auf Raumschiffen und Raumstationen, wie es auf realen Schiffen aussieht, weiß ich tatsächlich nicht. --Universal-InteressierterDisk.Arbeit 20:18, 3. Feb. 2022 (CET)
Das ist korrekt. In nicht medizinischen Einrichtungen, die eine große Anzahl von Menschen über einen längeren Zeitraum beherbergen, gibt es häufig Krankenstationen. Eine Google Recherche, die sich hierauf beschränkt, ist ungeeignet, wenn es um die Darstellung des allgemeinen Sprachgebrauchs in Kontext von Krankenhäusern geht. --178.4.180.189 02:01, 4. Feb. 2022 (CET)
„Auf Station“ ohne Artikel ist ebenso umgangssprachlich wie das ostdeutsche „auf Arbeit“, wenn auch sehr verbreitet. Gruselig ist auch das beliebte „auf Intensiv“. --Kabänes 06:12, 4. Feb. 2022 (CET)
Behalte doch bitte Deine Ansichten darüber, was gruselig ist und was nicht, ganz einfach für Dich. --94.219.22.201 01:37, 5. Feb. 2022 (CET)

Wie lang war die Amtszeit von Helmut Kohl?

Im Artikel Bundeskanzler (Deutschland) steht 5870 wohingegen im Artikel Helmut Kohl 5869 steht. Was ist nun richtig? --2A00:20:B04F:F6E8:98A9:2793:4E9C:9F44 21:27, 31. Jan. 2022 (CET)

Das ist ein Zaunpfahlfehler: Wenn ich mit MS Excel die Datumsdifferenz zwischen 27.10.1998 und 1.10.1982 ausrechne kommt 5869 raus, dazu muss man aber 1 addieren, weil er ja sowohl am ersten als auch am letzten Tag im Dienst war, also war seine Amtszeit 5870 Tage. Man könnte es auch so rechnen dass man eben die Differenz nimmt (weil z.B . wenn man 24h im Amt ist ist man einen Tag lang im Amt aber an 2 Tagen) aber ich glaub das ist unüblich. Oder Kurz: 5870 ist richtig. --MrBurns (Diskussion) 21:35, 31. Jan. 2022 (CET)
Laut unserer Artikel wurde Helmut Kohl am 1. Oktober 1982 zum Bundeskanzler gewählt. Sein Nachfolger Gerhard Fritz Kurt Schröder wurde am 27. Oktober 1998 gewählt. Das ergibt eine Amtszeit von 5870 Tagen für Helmut Kohl. --Rôtkæppchen₆₈ 00:10, 1. Feb. 2022 (CET)
Es steht aber auch dass er schon am 26.10. abgetreten ist. Sonst wärens 5871 Tage, da man ja üblicherweise sowohl den Antrittstag als auch den Abtrittstag mitzählt. --MrBurns (Diskussion) 01:03, 1. Feb. 2022 (CET)
Du widersprichst dir. --Digamma (Diskussion) 20:56, 1. Feb. 2022 (CET)
ich glaub eher du hast dich verlesen. Ich hab sinngemäß geschrieben dass man wenn einer 24h im Amt war könnte man das als 1 Tag zählen, man zählt es aber als 2 Tage weil er datumsmäßig an 2 Tagen im Amt war. --MrBurns (Diskussion) 05:26, 2. Feb. 2022 (CET)
Du hast oben geschrieben, dass man, wenn man den ersten und den letzten Tag mitzählt, auf 5870 Tage kommt, sonst auf 5869 Tage. Im nächsten Beitrag schreibst du, dass man auf 5871 Tage kommt, weil man ja Antrittstag und Abtrittstag mitzählt. --Digamma (Diskussion) 19:46, 2. Feb. 2022 (CET)
Nein, mit "sonst" meine ich wenn er erst 27.10. abgetreten wäre. --MrBurns (Diskussion) 00:00, 3. Feb. 2022 (CET)
Wenn er einen Tag früher abgetreten ist, dauern die Amtszeit einen Tag länger?
Du hast oben behauptet, dass die Datumsdifferenz vom 1. Oktober 1982 bis zum 27. Oktober 1998 5869 Tage beträgt. Dabei ist die Datumsdifferenz zwischen zwei aufeinanderfolgenden Tagen gleich 1. Da man den ersten (1. Oktober 1982) und den letzten (27. Oktober 1998) mitzählt, muss man 1 addieren und kommt dann auf 5870 Tage.
Im nächsten Beitrag behauptest du dann aber, dass man, wenn die Amtszeit vom 1. Oktober 1982 bis zum 27. Oktober 1998 geht und man Anfangstag (1. Oktober 1982) und Endtag (27. Oktober 1998) mitzählt, auf 5871 Tage kommen würde. --Digamma (Diskussion) 20:18, 3. Feb. 2022 (CET)
PS: Richtig ist natürlich, dass die Amtszeit am 26. Oktober 1998 mit dem Zusammentritt des neuen Bundestags endete. --Digamma (Diskussion) 20:27, 3. Feb. 2022 (CET)
vielleicht hab ich mich mit dem 27. einfach vertippt. Muss es überprüfen wenn ich iweder Zugriff auf Excel hab. --MrBurns (Diskussion) 00:00, 5. Feb. 2022 (CET)

Streit um alte Gasrechnungen zwischen Erbe des Wohnungseigentümers und Bewohnerin

Folgender Ablauf:

  • 2016 kaufte der Erblasser eine Eigentumswohnung.
  • Seine Lebensgefährtin zog (nachweislich) 2017 dort ein. Der Erblasser blieb in der alten Wohnung.
  • 2020 starb der Erblasser
  • Erben sind die Kinder; die LGF erhielt per Vermächtnis ein lebenslanges Wohnungsrecht in der von ihr seit 2017 bewohnten Wohnung, wobei nur sie die Verbrauchskosten zu tragen hat.
  • Ende 2021 meldeten sich die Gaswerke mit einer Rechnung von 2016 bis heute. Das Gas war nie ordentlich angemeldet worden.

Jetzt kommt die Streitfrage, wer bezahlt welche Gasrechnungen? insbesondere für den Zeitraum:

  1. vom Einzug der LGF bis zum Tod des Erblassers
  2. vom Tod des Erblassers bis zum grundbuchlichen Eintrag des vermachten Wohnungsrechts

Nach Auffassung der Eigentümer und Erben ist die LGF seit ihrem Einzug 2017 alleinige Nutzerin gewesen und daher auch verpflichtet zu zahlen. Lediglich der Zeitraum vom Hauskauf bis zu ihrem Einzug fällt in die vererbten Verbindlichkeiten.

Die wohnberechtigte LGF findet dagegen, weder die Rechnungen vor dem Tod des Erblassers gingen sie etwas an, noch die aus der Zeit zwischen dem Tod des Erblassers und der grundbuchlichen Eintragung des Wohungsrechts. Lediglich ab dieser Eintragung sei sie "bereit" die Kosten zu übernehmen.

Danke, -- Seelefant (disk.) 18:01, 1. Feb. 2022 (CET)

Kommt darauf an, mit wem das Gaswerk (konkludent) einen Gasabnahmevertrag geschlossen hat. Ich denke mit dem Erblasser, da die Lebensgefährtin erst später eingezogen ist. Die entstandenen Verbindlichkeiten sind damit auf die Erben übergegangen, es besteht gegen sie ein Anspruch der Gaswerke. Möglicherweise sind aber einzelne Forderungen bereits verjährt. Die Frage ist dann, ob sich die Erben etwas von der Lebensgefährtin zurückholen können. Für den Zeitraum bis zum Tode des Erblassers kommt es auf die (konkludenten) Absprachen zwischen Erblasser und Lebensgefährtin an. Wenn eine Absprache oder die allein die Praxis (aus der man dann auf eine stillschweigende Absprache schließen könnte) bestand, dass die Lebensgefährtin nichts bezahlen muss (Strom, TV, Müllabfuhr, Reparaturen etc.), dürfte gegen sie insoweit kein Anspruch gem. § 812 Abs. 1 S. 1 BGB bestehen (weil ja in Gestalt der Absprache ein Rechtsgrund vorliegt). Für die Zeit nach dem Tod ist es etwas tricky: Bestand eine (stillschweigende) Absprache zwischen dem Erblasser und Lebensgefährtin, gilt diese zunächst bis zu einer Beendigung der Absprache über den Tod des Erblassers hinaus fort. Erst nach Beendigung (die man in der Annahme des Vermächtnis sehen kann) ergibt sich dann aus dem Vermächtnis, sofern die Lebensgefährtin es angenommen hat, ein Anspruch auf Ausgleich der Gaskosten als Verbrauchskosten. Das sollte man aber möglicherweise auch vertraglich regeln. 92.79.101.164 18:15, 1. Feb. 2022 (CET)
Es dürfte sich hier um einen Fall der Grundversorgung handeln. Wenn der Hauseigentümer den Gasanschluss nutzt bzw. nutzen lässt ohne ihn ordentlich anzumelden, haftet er persönlich gegenüber dem Grundversorger. Mit dem Tod des Hauseigentümers ist diese Verpflichtung auf die Erben übergegangen. Einen Weg die Lebensgefährtin heranzuziehen, jedenfalls für Zeiträume vor Eintragung des Wohnrechts im Grundbuch, sehe ich hier nicht. Allerdings: Die Forderungen des Gasversorgers aus 2016 und 2017 dürften verjährt sein - die Erben können die Einrede der Verjährung gegenüber dem Gasversorger geltend machen. -- 2A02:908:121:9900:0:0:0:EED9 20:00, 1. Feb. 2022 (CET)
Fängt die Verjährungsfrist für Grundversorger nicht erst ab Rechnungsstellung an? -- Seelefant (disk.) 20:02, 1. Feb. 2022 (CET)
Negativ, für Grundversorger gibt es keine gesonderten Regelungen zur Verjährung, also greift die Regelverjährung von drei Jahren, und die wäre bei Rechnungsstellung Ende 2021 für die Jahre 2016 und 2017 bereits abgelaufen. Ab Rechnungsstellung beginnt die Fälligkeit der Forderung, d. h. vor Rechnungsstellung kann man nicht in Verzug geraten. -- 2A02:908:121:9900:0:0:0:EED9 20:16, 1. Feb. 2022 (CET)
Das scheint der BGH 2019 anders entschieden zu haben. -- Seelefant (disk.) 09:53, 5. Feb. 2022 (CET)
Und wann beginnt die Verjährungsfrist, wenn kein Vertrag geschlossen wurde? Mit Abnahme des Gases oder mit Kenntnis des Gasversorgers von der ungerechtfertigten Abnahme des Gases (§ 199 (1) Nr. 2 BGB)? 62.157.2.126 09:12, 2. Feb. 2022 (CET)
Mit Entstehen des Anspruchs, das ist in dem Fall der Ablauf des Abrechnungszeitraums. Ob der Versorger monatlich oder (eher üblich) jährlich abrechnet, geht aus den AGB hervor. Dem Gasversorger muss jedenfalls ein Verschulden vorgeworfen werden, dass er die Gasversorgung nicht gesperrt hat und auch danach über einen Zeitraum von fünf Jahren, in dem regelmäßig die Zählerstände abgelesen wurden, nicht aufgefallen ist, dass da ein Zähler weiterläuft, für den kein Vertrag verzeichnet ist, er wird sich also schwerlich darauf berufen können, dass er nichts wusste. -- 2A02:908:121:9900:0:0:0:EED9 09:32, 2. Feb. 2022 (CET)
Bei der hiesigen Gasanstalt läuft es genau andersherum. Der Ableser hat eine nach Adresse sortierte Liste der Kunden und für jeden Kunden eine Liste der Zählernummern. Zähler ohne aktuellen Kunden tauchen gar nicht in der Liste auf und werden so weder abgelesen noch erfasst. --Rôtkæppchen₆₈ 09:41, 2. Feb. 2022 (CET)
Der Hinweis auf den Zähler und das Verschulden des Gasversorgers ist nachvollziehbar. Wenn ein Gasanschluß gelegt und ein Gaszähler eingebaut ist, könnte es schon unter die in § 199 BGB genannte grobe Fahrlässigkeit fallen, den Zähler nicht ab und zu mal abzulesen. 62.157.2.126 09:53, 2. Feb. 2022 (CET)
Wobei das hier alles Stochern im Nebel ist. Der Fragesteller schreibt ja, der Erblasser habe das Gas "nie ordentlich angemeldet" - was immer das heißt. 92.79.101.164 11:28, 2. Feb. 2022 (CET)
Mir fällt noch ein, dass zwischen Erblasser und Lebensgefährtin ein atypischer Mietvertrag bestand, wonach die Lebensgefährtin zumindest keine Miete zahlen sollte. Wenn man nicht ohnehin von einer (stillschweigenden) Abrede, die Lebensgefährtin habe vollkommen kostenlos dort wohnen dürfen, ausgehen muss, wären die Betriebskosten nach § 556 Abs. 3 BGB "spätestens bis zum Ablauf des zwölften Monats nach Ende des Abrechnungszeitraums mitzuteilen", d.h. es könnten von den Erben ohnehin nur noch Kosten des Jahres 2021 geltend gemacht werden. 92.79.101.164 21:39, 1. Feb. 2022 (CET)
Ich bin mir nicht sicher, ob man hier die mietvertragliche Regelung analog anwenden kann. Und selbst wenn, könnte die Lebensgefährtin dem Anspruch einen eigenen Schadensersatzanspruch entgegenhalten, denn: Wäre das Gas ordentlich angemeldet worden, hätte die Lebensgefährtin die Möglichkeit gehabt, zu einem günstigeren Versorger zu wechseln und nicht in der teuren Grundversorgung zu bleiben. -- 2A02:908:121:9900:0:0:0:EED9 09:40, 2. Feb. 2022 (CET)
Es sind im (ansonsten recht umfangreichen und vollständigen) Nachlass an Papieren keinerlei Hinweise darauf zu finden, dass jemals ein Gasvertrag abgeschlossen wurde. Es waren also sowohl der Eigner=Erblasser, als auch die Bewohnerin=Vermächtnisnehmerin in dieser Hinsicht jahrelang untätig. Wie bei dieser Konstellation der Eigner der Bewohnerin dabei im Weg gestanden haben soll, sich um einen günstigeren Anbieter zu bemühen, erschliesst sich mir nicht. -- Seelefant (disk.) 12:01, 2. Feb. 2022 (CET)
Zum Abschluss eines Gasvertrags braucht man die Zählernummer. Die hat nur der Eigentümer, denn der Gaszähler ist (zumindest in den Gebäuden, die ich kenne) verschlossen und nur mit einem gesonderten Schlüssel zugänglich. -- 2A02:908:121:9900:0:0:0:EED9 12:18, 2. Feb. 2022 (CET)
Die Bewohnerin hat alle Schlüssel. Auch der Erblasser hatte Schlüssel. Der Erbe hat, weil sie Wohnberechtigte ist und nicht Mieterin, überhaupt kein Betretungsrecht, und hat darum auch keine Schlüssel mehr. -- Seelefant (disk.) 19:22, 2. Feb. 2022 (CET)

Versorger haben gelegentlich seltsame Vorstellungen. Zwei Vorkommnisse, die ich erlebt hatte: Ein Umzug fand in einem Schaltjahr statt, der jährliche Grundpreis wurde nach Tagen anteilig aufgeteilt. Nur erhöhte der programmierte Algorithmus dabei mal eben den Grundpreis um 1/365, weil er den anteiligen täglichen Grundpreis mal 366 Tage berechnete... Ich war mal in eine Mietwohnung eingezogen, deren Vormieterin vermutlich ohne Erben verstorben war - irgendjemand hatte sie dann als Stromkundin bei den Stadtwerken abgemeldet. Als ich einzog, las ich den Zähler ab und meldete mich als neuer Kunde an. Worauf die Stromer auf die geniale Idee kamen, mir den Grundpreis für die Monate des Wohnungsleerstands und noch zwei oder drei kWh in Rechnung stellen zu wollen. Konnten die gar nicht verstehen, daß ich das mit dem Götzzitat kommentierte. (Wie das dann genau ablief, weiß ich nicht mehr, aber ich habe dafür jedenfalls nicht gezahlt.) Wobei es dabei letztlich nur um Kleingeld ging. Anders sähe es natürlich aus, wenn eine Wohnung im Winter monatelang leer steht und darin dann eine Heizung läuft, also richtig Energie verbraucht wird. Das kann richtig Ärger geben. --77.8.217.248 10:51, 2. Feb. 2022 (CET)

Dass ein Schadensersatzanspruch besteht, kann man anhand des mitgeteilten Sachverhalts nicht beurteilen, und es ist zunächst auch einmal eine Unterstellung, der betreffende Gaslieferant sei nicht der günstigste. Der Vermieter ist nach § 556 Abs. 3 BGB nicht verpflichtet, immer das günstige Angebot zu wählen, sondern er muss den Wirtschaftlichkeitsgrundsatz beachten. Im Münchener Kommentar heißt es hierzu (Zehelein in MüKoBGB, 8. Aufl. 2020, § 556 Rn. 117 f.):
"Keiner ordentlichen Geschäftsführung entspricht es, wenn sich der Vermieter auf unangemessene, marktunübliche oder überhöhte Entgeltvereinbarungen mit Dritten einlässt (zB zu hohe Grundkosten im Wärmelieferungsvertrag). Der Preis muss in das Verhältnis zum Leistungsinhalt gesetzt werden. Der Vermieter muss nicht unbedingt den billigsten Anbieter wählen, darf aber auch nicht ohne sachlichen Grund ein günstigeres Angebot ausschlagen. Da sich ein exakt angemessenes Entgelt kaum abstrakt ermitteln lässt, wird es unter Heranziehung der Wesentlichkeitsgrenze des § 5 WiStG zugelassen, dass die aufgewendeten die üblichen Kosten um 20 % übersteigen können. Dies kann jedoch nicht als starre Grenze angesehen werden. Der Vermieter ist aus dem Wirtschaftlichkeitsgebot heraus grundsätzlich zu kostengünstiger Beschaffung, auch unter Ausnutzung besonders günstiger Beschaffungsmöglichkeiten verpflichtet, soweit ihm dies zumutbar ist. Maßgeblicher Ansatzpunkt für die Kontrollüberlegung ist daher keine Wertgrenze, sondern die Frage, ob ein verständiger Vermieter die Kosten auch veranlasst hätte, wenn er sie selbst tragen müsste. Da im Rahmen der Wirtschaftlichkeit auch der Verwaltungsaufwand zu berücksichtigen ist, kann vom Vermieter jedoch nicht verlangt werden, dass er alle denkbaren Anbieter zur Abgabe eines Angebots auffordert. Es muss aber zumindest eine ausreichende Anzahl von Angeboten eingeholt werden, um sich einen Marktüberblick zu verschaffen. Eine Ausschreibung ist nicht erforderlich.
Der Wirtschaftlichkeitsgrundsatz gilt nur innerhalb der vom Vermieter gewählten Versorgungsart und verpflichtet den Vermieter nicht, schon bei der Auswahl die wirtschaftlich vorteilhafteste Versorgungsvariante zu wählen."
92.79.101.164 11:22, 2. Feb. 2022 (CET)
Es gibt keinen Vermieter. Wir reden von einem ins Grundbuch eingetragenen, lebenslangen Wohnungsrecht. -- Seelefant (disk.) 11:56, 2. Feb. 2022 (CET)
Da Du die Rechtslage nicht ganz umreißt, wäre ein Besuch beim Anwalt umso dringlicher. Es geht um die Zeit, als der Erblasser noch lebte: Da gab es ja, so wie Du das schilderst, noch kein dingliches Wohnrecht zugunsten der Lebensgefährtin, sondern irgendeine andere Grundlage für die Überlassung der Wohnung: Das kann eine reine Gefälligkeit, eine Leihe oder eine Miete gewesen sein - abschließend kann man das hier nicht beurteilen. Wenn es keine expliziten Vereinbarungen gab, kann man auf solche auch aus den Umständen schließen, also daraus, wie die Überlassung und die Übernahme von (sonstigen) Kosten zwischen Erblasser und Lebensgefährtin praktiziert wurden. 92.79.101.164 12:01, 2. Feb. 2022 (CET)
Es sind im (ansonsten recht umfangreichen) Papiernachlass keine schriftlichen Vereinbarungen über die Wohnung zu finden, die sich auf die Lebenszeit des Erblassers beziehen. Allerdings zog die Lebensgefährtin nachweislich in dem Moment in das Haus, in dem ihr erbvertraglich das Wohnrecht für den Todesfall des Erblassers vermacht worden war. -- Seelefant (disk.) 12:04, 2. Feb. 2022 (CET)
So wie Du alles schilderst, war es doch offenbar der Wille des Erblassers, dass einerseits seine Lebensgefährtin zu seinen Lebzeiten mit Kosten für die Wohnung nicht belastet werden sollte, dass andererseits aber auch seine Kinder als Erben nach Eintritt des Erbfalls nicht für ihre "Stiefmutter" aufkommen sollten. Das sollte man respektieren und den vermutlich ohnehin mindestens fragilen Familienfrieden nicht weiter stören. 92.79.101.164 15:09, 2. Feb. 2022 (CET)
Lässt sich denn eruieren, wer damals (also zu Lebzeiten des Erblassers) für die anderen Kosten, wie z.B. Strom aufkam? --Digamma (Diskussion) 20:07, 2. Feb. 2022 (CET)

Haltung von Schreibgeräten beim Schreiben von Hand

Liebe Auskunft, üblicherweise werden Stifte beim Schreiben von Hand zumindest in Deutschland zwischen Daumen und vorderstem Glied des Zeigefingers gehalten; siehe [7], Abschnitt 5. Was bedeutet gute Stifthaltung? . Personen, die viel schreiben müssen, nach meiner Wahrnehmung zum Beispiel in Banken, halten Stifte dagegen oft so, dass der Stift mit dem vordersten Gelenk des Zeigefingers umfasst wird. Welche Informationen gibt es dazu (Bezeichnung dieser Haltung, Verbreitung, ergonomischer Hintergrund)? Es gibt zum Beispiel hier den 4-Punkt-Griff, ich bin mir aber nicht sicher, ob damit die von mir wahrgenommene Haltung gemeint ist. Vielen Dank für Antworten! --BlackEyedLion (Diskussion) 17:51, 4. Feb. 2022 (CET)

Personen in Banken müssen viel schreiben? --Digamma (Diskussion) 19:16, 4. Feb. 2022 (CET)
Ich schreibe in meinem Büroberuf nur alle paar Tage mal etwas handschriftlich (außer vielleicht noch Unterschriften, die ich aber hier nicht mitzähle), meine Bankberaterin musste heute alleine für mich ein Formular handschriftlich ausfüllen. Die von mir beschriebene Haltung des Schreibgeräts kenne ich jedenfalls vor allem von Beschäftigten von Banken und dann nach meiner Einschätzung bei einem großen Teil der Beschäftigten, weshalb ich vermute, dass diese Stifthaltung auch schon anderen aufgefallen ist. --BlackEyedLion (Diskussion) 21:39, 4. Feb. 2022 (CET)
Da gibts noch Varianten mit Bild (Seite 13)[8], ich glaube in der Frage ist der interdigitale Dreipunktgriff gemeint. Die übliche Haltung zwischen Daumen und Zeigefinger wird IMHO immer vom Mittelfinger gestützt (und ergibt damit den Dreipunktgriff). --Ailura (Diskussion) 21:53, 4. Feb. 2022 (CET)
Ich halte den von mir gemeinten Griff ausweislich der genannten Quelle für einen seitlichen Dreipunktgriff (funktional) oder auch einen seitlichen Griff mit eingestecktem Daumen (von Seite 12, ungünstig). Ich werde in der nächsten Zeit versuchen, den Artikel Manuelles Schreiben zu ergänzen. --BlackEyedLion (Diskussion) 22:36, 4. Feb. 2022 (CET)
Ich war neulich auf der Bank. Die Bankberaterin hat für die Kontoeröffnung eine Menge Papier ausgedruckt, aber der einzige, der dabei etwas von Hand geschrieben hat, war ich, nämlich um einige Unterschriften zu leisten.
Als Lehrer schreibe ich täglich eine ganze Menge von Hand und zwar immer mit der üblichen Haltung. Eine andere Stifthaltung spricht m.E. dafür, dass der Schreiber nicht oft von Hand schreibt. --Digamma (Diskussion) 22:33, 4. Feb. 2022 (CET)
Ja, schön, aber das hilft bei der Beantwortung der Frage nicht. Wie gesagt habe ich diesen Griff vor allem bei Bankbeschäftigten gesehen, und dort bei einem großen Anteil. Das Schreiben von Bankbeschäftigten ist auch insgesamt besonders, weil zumeist Druckschrift verwendet wird und es im Übrigen auch auf eine große Geschwindigkeit ankommt. Ich denke, der von mir wahrgenommene Griff ist besonders für schnelles Schreiben von Druckschrift geeignet. --BlackEyedLion (Diskussion) 22:36, 4. Feb. 2022 (CET)
"... und es im Übrigen auch auf eine große Geschwindigkeit ankommt."
Das würde ich bezweifeln. Vielleicht große Geschwindigkeit dafür, dass in Druckschrift geschrieben wird.
Was ich mir vorstellen könnte, ist, dass für das Schreiben von Durchschlägen mit mehr Druck eine steilere Stifthaltung nützlich ist. --Digamma (Diskussion) 22:41, 4. Feb. 2022 (CET)

Ich meine, dass bei einen Vierpunktgriff auch noch der Ringfinger (als vierter Punkt) den Kuli berührt (ohne Gewähr). Hier ist noch ein interessanter Artikel: https://www.hs-tutorials.eu/images/Handreichungen/HS-Tutorials_Modul_2_Kindergarten.pdf --Doc Schneyder Disk. 23:09, 4. Feb. 2022 (CET) Ein weiterer Link: https://de.wikihow.com/Einen-Stift-halten --Doc Schneyder Disk. 23:13, 4. Feb. 2022 (CET)

Ich glaube das ist eine individuelle Sache. Man hält den Stift so, wie es sich "richtig" anfühlt. Wie beim Händefaölten, welcher Daumen oben ist bestimmt das Gefühl. Sowohl mein Bruder als auch meine Tochter haben von Anfang Zeige- und Mittelfinger auf dem Stift liegen gehabt und mit dem Ringfinger abgestützt. Das sieht sehr unbeholfen aus und die Lehrerin meinter Tochter wollte die Haltung korrigieren. Das konnte ich ihr aber ausreden, da ich wusste dass die Stifthaltung nichts mit der Geschwindigkeit oder Schönheit der Schrift zu tun hat. --62.143.214.141 09:46, 5. Feb. 2022 (CET)

Fernempfang von Hörfunksendern aus Xinjiang

In der Volksrepublik China (und somit auch in Xinjiang) gibt es einige Hörfunksender, die auf Nennfrequenzen ab einschließlich 87,0 MHz senden. Was müssen Funkamateure im D-A-CH-Raum beachten, wenn sie einen Fernempfang solcher Hörfunksender vornehmen wollen? Und können Telekommunikationsvergehen zum Verlust des Asylstatus führen? Falls ja: Gilt für Chinesen bzw. Uiguren eine Ausnahme? --2003:D2:4F1B:4706:B4A7:5274:8516:A6FD 21:27, 4. Feb. 2022 (CET)

Fernempfang im UKW-Bereich? Das wird nicht funktionieren, diese Frequenzen werden üblicherweise nicht von der Ionosphäre reflektiert. Überreichweiten etwas über die Kimm kommen zwar vor, aber um ein gutes Drittel des Erdumfanges halte ich das für ausgeschlossen.
Davon abgesehen, wieso sollte es ein Vergehen sein, empfangbare Rundfunksender auch zu empfangen? Das geht doch keinen was an. Derartige Verbote gab es unter Adolf H. und mit rechtsstaatlichen Verhältnissen hatte der mit seinen Kumpanen nichts am Hut. –Falk2 (Diskussion) 21:40, 4. Feb. 2022 (CET)
Ich hatte schon Kanal-2-Fernsehfernempfang über 1600 Kilometer. 87 MHz des chinesischen UKW-Radios sind aber etwas mehr als Fernsehkanal 2 (48,25 MHz) und bis China sind es über 5000 Kilometer. Da sehe ich schwarz und höre Rauschen. --Rôtkæppchen₆₈ 21:53, 4. Feb. 2022 (CET)
@Falk2: Der Straftatbestand des Rundfunkverbrechens ist nicht wirklich neu. Im Dritten Reich mussten Merkzettel an den Endgeräten angebracht werden. In der DDR machte die FDJ Jagd auf Ochsenkopfantennen und in Nordkorea sind Hörfunkempfänger verplombt. Ganz aktuell gibt es Hickhack zwischen MABB, ZAK und Russia Today um mentalitätsbedingte Differenzen um die Natur von Telemedien mit Drittstaatenlizenz. Da ist es durchaus denkbar, dass eine in Bonn im Rheinland verortete bundesdeutsche Behörde Anstoß daran nimmt, dass der Fragesteller eine Funkempfangsanlage errichtet oder zu errichten gedenkt, die potentiell dazu in der Lage ist, BOS-Funk zu empfangen. --Rôtkæppchen₆₈ 02:32, 5. Feb. 2022 (CET)
Dran denken schon, nur gibt es dagegen eben kein Gesetz oder eine anderweitige Rechtsnorm und dafür den Grundsatz Was dem Bürger nicht ausdrücklich verboten ist, ist erlaubt (und im Gegenzug, was dem Staat nicht ausdrücklich erlaubt ist, ist ihm verboten). Andersrum wäre es ein Polizeistaat und der Zirḱus mit Russia Today kann Rundfunkteilnehmern ziemlich Wurscht sein. Wenn die Frequenz moduliert ansteht, kann man sie empfangen, demodulieren und auswerten. Das mit der FDJ und den Ochsenkopfantennen war vor meiner Zeit und rechtlich gedeckt war es auch nicht. Gebühren entstehen beim Empfang terrestrischer ausländischer Sender ebensowenig wie beim Satellitendirektempfang. Es besteht schließlich kein Vertragsverhältnis. Tatsächlich verboten ist das Verwenden der Sendeenergie für andere als Rundfunkzwecke (wie im Garten in Sendernähe die Laube beleuchten).
Was es gab, war eine innerdienstliche Anweisung, dass die einheimischen Sender auf den Rundfunkempfängern bei der NVA markiert sein sollten. Ich habe das auch gemacht, dummerweise aber schon zu Hause und hundert Kilometer weiter lagen die UKW-Frequenzen natürlich anders. Interessiert hat das letztlich niemanden und bei den wenig später aufkommenden PLL-Empfängern wäre es mangels Skalen mit Zeiger sowieso nutzlos gewesen. –Falk2 (Diskussion) 03:34, 5. Feb. 2022 (CET)
Ich würde die Empfangseinheit auf die Rügener Ostküste verlegen, um wenigstens die Wahrscheinlichkeit eines unabsichtlichen Empfangs des deutschen BOS-Funks zu minimieren. --2003:D2:4F2A:E4B0:65FE:2002:3F78:8D59 10:24, 5. Feb. 2022 (CET)

Artikelwörter im Deutschen

In welchen Fällen können die eigentlich weggelassen werden? In meinem persönlichen Sprachempfinden bin ich mir oft nicht so sicher, bei abstrakten Konzepten und ... tja ich weiß es nicht recht, empfinde ich manchmal sowohl die Form mit (dem) als auch die ohne (den) unbestimmten Artikel (manchmal, wie hier auch, betrifft es auch den bestimmten) als richtig. Das betrifft natürlich vor allem nur den unbestimmten Artikel, bereist der bestimmte Artikel ist ja selbst gewissermaßen bedeutungstragend, alle anderen Artikelwörter (Demonstrativpronomen, attributive Possessive) sowieso. Bei manchen Wörtern wie Wasser sind ja auch alle drei Formen unterschiedlicher Bedeutung: "Wasser" (von stark flektierten Adjektiven begleitet) ist allgemein, "ein Wasser" ist eine Wassersorte und "das Wasser" ist etwas bestimmtes. Unstrittige Fälle sind der unbestimmte Plural und ein vorangestelltes Genitivattribut. Dann tritt auch jeweils die starke Deklination auf. Außerdem wird der bestimmte Artikel bei Personennamen und geografischen Namen neutralen Genus regelmäßig weggelassen, erscheint allerdings wieder, wenn das Wort in einer, wie wurde es auf Wiktionary erklärt, spezifischen Qualität steht, beispielweise durch ein Adjektivattribut oder einen Relativsatz. Der unbestimmte Artikel tritt hier auch regelmäßig auf, bedeutungstragende Wörter wie "dieser" oder "dein" sowieso. (Beispiele: "Lisa geht nach Hause", aber "die schöne Lisa" und "Ich sah die Lisa, von der du gesprochen hattest"; "Österreich war früher eine Großmacht", aber "Das alte Österreich ging unter"; außerdem: "Ich kenne diese Laura nicht", "Ich traf deine Lea", "Wir wollen ein neues Deutschland erschaffen", "Gestern unterhielt ich mich mit einer Madeleine, die etwas über dich erfahren wollte" oder "Ich kannte eine Laura")

Was ich mich nun (noch) frage ist, ob sich die Frage, in welchen weiteren Fällen (was in vollständigen Sätzen normalerweise wortabhängig sein dürfte) der unbestimmte Artikel weggelassen werden kann (oder sogar muss?) bzw. allgemein Artikelwörter weggelassen werden können (müssen?) irgendwie mit einer Regel oder Systematik klären lässt, beziehungsweise wie häufig das ist (und vielleicht auch noch, ob es eine Erklärung für das besondere Verhalten jener Namensgruppen gibt).

--Universal-InteressierterDisk.Arbeit 23:59, 3. Feb. 2022 (CET) (ergänzt um 00:22, 4. Feb. 2022 (CET))

Gefühlt sind das 23 Fragen. Vielleicht portionierst du das erst einmal.--2003:D0:2F18:5702:3002:92AA:ED3B:8CE0 00:42, 4. Feb. 2022 (CET)
Ich glaube, dass es historisch einfach eine Konvention ist, dass man bei Personennamen und den meisten Ländern einfach keinen Artikel verwendet ("Der Tisch war dort.", aber nicht: "Die Laura war dort", sondern nur "Laura war dort.") [man beachte aber, dass man das regional, z.B. in Österreich, durchaus so sagen kann]. Auch andere Eigennamen fallen in diese Kategorie (statt "Ich arbeite bei dem Daimler-Benz" sagt man "Ich arbeite bei Daimler-Benz") (auch hier wieder regional: "Ich schaffe beim [= bei + dem] Bosch."). Interessanterweise ändert sich das, wenn der Eigenname nur Teil von etwas ist: "Ich ging zu Apple" vs. "Ich ging zu dem Apple-Store." Auch bei Buchtiteln usw. lässt man den Artikel weg, allerdings nicht so konsequent ("Ich las 'Faust'." und "Ich las den 'Faust'."). Vielleicht kann man also verallgemeinern, dass man den Artikel bei Eigennamen weglässt, aber, wie Du ja oben selbst zeigst, eben doch verwendet, wenn es bedeutungsunterscheidend ist (eine bestimmte vs. irgendeine Laura; "der Faust" im Sinne von Goethes Faust und eben nicht irgendein anderes Buch, das so heißt). Das gilt auch, wenn man ein Adjektiv verwendet (der schöne Franz, also nicht der andere, weniger attraktive Franz). Dann gibt es noch ein paar Ausnahmen, etwa sehr bekannte Persönlichkeiten, meist Frauen, die sich einen Artikel "verdient" haben: "Die Dietrich". Auch hier will man eigentlich sagen, dass es eben die eine spezielle Marlene Dietrich ist, und nicht eine x-beliebige Person, die Dietrich heißt. 80.71.142.166 06:55, 4. Feb. 2022 (CET)
Hallo Universalinteressierter, in der deutschen Grammatik ist vieles nicht definiert, bei dem Stilisten zwischen richtig und falsch unterscheiden. „Tisch ist groß“ ist grammatikalisch eine zulässige Aussage, aber inhaltlich kaum verständlich und stilistisch pfui. Du musst also nach Gefühl gehen.--Blue 🔯 19:16, 4. Feb. 2022 (CET)
„Nach Gefühl“ find ich gut. jetzt weiß ich wenigstens, warum meine Deutschnote früher so mäßig war, der Deutschlehrer hatte ein anderes Gefühl als ich. --2003:D0:2F18:5777:3816:7A66:D2AD:7CBF 19:39, 4. Feb. 2022 (CET)
Wenn es eine grammatikalische Regel gibt, dann gilt das mit dem Gefühl nicht mehr, aber zeig mir bitte die grammatikalische Regel, nach der ‹Essen ist fertig› und ‹Das Essen ist fertig› in richtiges Deutsch und falsches Deutsch einsortierbar wären. Alles, was Stil betrifft, ist Gefühlssache, dann stimmte eben was mit deinem Gefühl nicht.--Blue 🔯 21:09, 4. Feb. 2022 (CET) Beispiel für Sprachgefühlherausforderung: „Er meint, er könne Caritas nicht leiden.“ Oder: „Er meint, er könne die Caritas nicht leiden.“ Bei dem zweiten Satz denke ich an die Organisation, beim ersten an Nächstenliebe an sich, aber wenn der Sprecher auch beim ersten Satz die Organisation meint, ist sein Weglassen des Artikels lediglich ein stilistischer Lapsus.
Es kommt auch immer auf den Verwendungskontext an. Bei Verwandten und Bekannten, die nur mit dem Vornamen bezeichnet werden, benutze ich in der zwanglosen mündlichen Kommunikation immer den bestimmten Artikel, in der schriftlichen Kommunikation nie. --Jossi (Diskussion) 21:20, 5. Feb. 2022 (CET)

Schmerzen unter dem rechten Rippenbogen

--2003:ED:D717:A100:2D86:2BF6:B259:62EC 19:08, 4. Feb. 2022 (CET)

Gehen Sie bitte zum Arzt.--Blue 🔯 19:09, 4. Feb. 2022 (CET)
Das kann alles möglich sein, von völlig harmlos bis tödlich. Ohne medizinische Untersuchung kommt man da nicht weiter. --94.219.22.201 01:16, 5. Feb. 2022 (CET)
Die IP-Adresse ist aus Nordrheinwestfalen. Da könnte es sich lohnen, mal die 116117 anzurufen und dort um Rat zu fragen. --Rôtkæppchen₆₈ 01:41, 5. Feb. 2022 (CET)
Hast Du bessere Quellen als GeoIP? Bei meiner IP kommt da nur Unsinn heraus. --46.78.147.226 18:10, 5. Feb. 2022 (CET)

Probleme beim Lesen der "Welt"

Moin @ all, seit zwei/drei Tagen kann ich die "Welt" nicht mehr online lesen. Bei längeren Artikeln springt das Bild wieder auf fast den Anfang zurück. Bin ich der Einzige?

System: Win10prof, FF, alles aktuell... Gruss --Nightflyer (Diskussion) 20:03, 5. Feb. 2022 (CET)

Bekomme ich bei diesem Artikel [9] auch. Wenn ich dann hochscrolle ist viel weiß. Edge und Chrome funtionieren dagegen beim gleichen Artikel einwandfrei.--Mhunk (Diskussion) 20:37, 5. Feb. 2022 (CET)
Hast Du es schon mit deaktiviertem Werbe- und Cookiebannerblocker versucht? --Rôtkæppchen₆₈ 20:57, 5. Feb. 2022 (CET)

Akkus testen, aber wie?

Ich habe ein kleines Prüfgerät, das kann Stromstärke, Spannung und Widerstand messen. Nun habe ich mehrere alte Akkus, Größe AA und AAA, vor mir. Kann ich nun mit diesem Gerät
a) messen, wie voll der Akku ist, also ganz voll, halb voll oder fast leer? und
b) feststellen, ob der Akku reif zum wegwerfen/riseiklen ist? Wenn ja, wie? Danke! --Gruenschuh (Diskussion) 10:35, 1. Feb. 2022 (CET)

Dazu brauchst Du ein spezielles Prüfgerät, das die Akkus unter Last misst. Mit einem normalen Multimeter misst Du nur die Leerlaufspannung und die ist nicht oder nicht immer aussagekräftig. --Rôtkæppchen₆₈ 10:48, 1. Feb. 2022 (CET)
Braucht er gar nicht. Er braucht einen oder mehrere definierte Lastwiderstände, die den Akku rechnerisch in definierten Zeiten (man kann z. B. 10, 30 oder 60 min wählen), entladen. Dann lädt man ihn voll auf, schließt den Lastwiderstand an und dazu parallel das Voltmeter, liest es in regelmäßigen Abständen - z. B. alle halbe Minute - ab und notiert die Meßwerte, bis die Spannung unter Last unter 0,95 V abgesunken ist, dann bricht man die Messung ab und steckt den Akku wieder ins Ladegerät. Aus den Spannungen erhält man die Lastströme und dadurch die entnommene Ladung - kann man sich auch alles graphisch darstellen. Und wenn sich dabei zeigt, daß die Kapazität arg gering oder der Innenwiderstand übel hoch sind, dann darf man sich von der Zelle auch mal verabschieden. Rechenbeispiel: Die Zelle mag nominell eine Kapazität von 1500 mAh haben. Um die in 30 min zu entladen, muß sie mit rund 3000 mA belastet werden. Bei einer Spannung von 1,2 V unter Last braucht man also eine Bürde von 400 mOhm, die 3,6 W Leistung verträgt - man nimmt also einen Widerstand 0,42 Ohm/5 W und mißt den erst einmal genau. Entsprechend für andere Zeiten. - Andere rechnen so: Eine neue Zelle kostet zwei Euro. Wieviel Theater soll man denn machen, wenn eine gebrauchte zickt? Weg damit (ggf. an Bastler mit zuviel Zeit verschenken), neue her... --77.0.249.48 11:16, 1. Feb. 2022 (CET)
Den Ladezustand (noch entnehmbare Restkapazität) einer Akkuzelle kann man prinzipiell nicht messen. Bei Anwendungen, bei denen es darauf ankommt, "zählt" eine Meßschaltung die seit der letzten Aufladung bereits entnommene Ladungsmenge und berechnet bzw. schätzt daraus die noch verfügbare Restmenge. Die gleiche Schaltung (Digitaltechnik mit Mikroprozessor etc.) kann auch die aktuelle Akkukapazität alternder Akkus schätzen, indem sie mißt, wieviel Ladung zwischen "entladen" und "voll" aufgenommen wurde. Und natürlich gibt es teure Lade- und Prüfgeräte, die solche Funktionalitäten eingebaut haben. Wer's - für eine im Grunde überholte Akkutechnologie - braucht... --77.0.249.48 11:30, 1. Feb. 2022 (CET)
Akkutechnologie überholt? OK, mag sein. Aber, liebe 77er URL, was schlägst du jetzt als Akkuersatz vor, ohne dass ich alles wegwerfen muss, das die Akkus verwendet? --Gruenschuh (Diskussion) 13:36, 1. Feb. 2022 (CET)
Klugschiss am Rande: Das ist eine IP, keine URL. Eine URL ist so was wie fußmarsch://kühlschrank.küche.wohnung/tür/unten/bierflaschen/1.bttl --Kreuzschnabel 14:06, 1. Feb. 2022 (CET)
Sorry, Kreuzschnabel, war ein Freudscher Fehler. Ich kann wohl URL und IP unterscheiden. Hab schneller getippt als gedacht. --Gruenschuh (Diskussion) 15:22, 1. Feb. 2022 (CET)
Dann folg doch mal meiner IP und bring mir eine mit :) --Kreuzschnabel 18:18, 1. Feb. 2022 (CET)
Nicht Akkutechnologie allgemein ist überholt, sondern diese spezielle Akkutechnologie (NiMH-Zellen). Stand der Technik sind LiPo-Akkus, und das mit dem Wegschmeißen kommt von ganz alleine, weil diverse Geräte ohnehin fast schneller entsorgt als neu gekauft werden, was nicht unbedingt bedeutet, daß sie defekt sind - sie sind einfach nicht mehr "modern". (Und im übrigen kann man sich auch einfach mal überlegen, ob man für manche Zwecke nicht einfach lieber Primärelemente verwenden möchte - Kosten pro Stück 20 Cent und haben ähnliche Kapazitäten wie NiMH-Zellen, sind aber weitaus nervenschonender.) --77.0.249.48 14:17, 1. Feb. 2022 (CET)
Nö. Alkaline-Batterien haben in Geräten ohne Entladeschlussüberwachung ungefähr einen doppelt so hohen Energieinhalt als LSD-NiMH-Akkus. Dazu haben NiMH-Akkus eine niedrigere Entladeschlussspannung als Alkaline-Batterien, weshalb für Alkaline-Betterien ausgelegte Geräte NiMH-Akkus gar nicht ganz leer machen können. Da braucht das Gerät dann viel häufiger als vorausberechnet frische Akkus. Nickel-Zink-Akkus sind auch keine Lösung, da Schnellkaputtgeher. --Rôtkæppchen₆₈ 15:38, 1. Feb. 2022 (CET)
Doch. Der theoretische Energieinhalt tut's nicht. Der Schaltungsentwickler geht von einer minimal erforderlichen Betriebsspannung aus, wobei die meisten Schaltungen zumindest bei geringen Leistungsaufnahmen auch keine Probleme damit haben, wenn die Betriebsspannung 20-50 % höher ist. (Eine vernünftige Auslegung für die verfügbare Betriebsspannung unter Last ist z. B. 1,05-1,10 V/Zelle; dann sollte die Schaltung also auch mal 1,65 V/Zelle abkönnen, was regelmäßig bei frischen bzw. aufgeladenen Alkaline-Batterien vorkommt.) Und hier ergibt sich ein Dilemma: je niedriger die Betriebsspannung festgelegt wird, desto stärker wird die Zelle ausgenutzt, aber desto schlechter wird auch die entnommene Energie verwertet, weil alles, was oberhalb der projektierten Betriebsspannung an Spannung vorhanden ist, als Verlust zu betrachten ist. Früher habe ich auch gedacht, man sollte die Abschaltspannung auf 0,9 V/Zelle festlegen. Nur ist das Unfug: im Bereich zwischen 0,9 und 1,1 V läßt sich der Zelle kaum noch Ladung entnehmen, aber man verschenkt 0,2 V, also ungefähr 20 % der Energie, gegenüber einer Auslegungsspannung von 1,1 V. Wenn Geräte etwas leistungshungriger sind, ist die nominelle Batteriespannung von 1,5 V ohnehin uninteressant. Die früher so beliebten kleinen Stabtaschenlampen für 3-V-Batterien hatten 2,4-V-Glühlampen, und das war auch völlig korrekt, weil die Klemmenspannung bei mittleren Lasten wegen des Innenwiderstands der Batterie auf 1,2 V/Zelle absinkt, was wunderbar zur Spannung von NiCd- und NiMH-Zellen paßt. (Entsprechend waren zu Glühlampenzeiten für die Fahrradbeleuchtung auch fünf und nicht nur vier Zellen als Spannungsversorgung vorzusehen, denn der Scheinwerfer brauchte tatsächlich 6 V = 5*1,2 V unter Last.) Bei sehr hohen Lasten (z. B. bei Digitalkameras) sind die Akkus gegenüber den Alkaline-Batterien im Vorteil, weil wegen ihres niedrigeren Innenwiderstands die Spannung weniger einbricht und sie trotz der niedrigeren Nennspannung eine höhere Spannung unter Last aufweisen. (Ich hatte allerdings mal eine kleine Digitalcamera der "Spielzeugklasse", deren ahnungsloser Entwickler die Abschaltspannung auf ca. 1,4 V/Zelle festgelegt hatte, wohl deswegen, weil er dachte, daß eine Alkaline-Batterie mit nur noch 1,4 V Leerlaufspannung annähernd leer ist. Die "fraß" folglich Batterien und ließ sich mit Akkuzellen gar nicht betreiben - das ist nicht geplante Obsoleszenz, sondern "Schrott ab Werk".) --77.10.161.222 03:51, 6. Feb. 2022 (CET)
Du kannst du Spannung messen, wenn er aufgeladen ist. Erreicht er nicht mehr die Nennspannung, ist er hinüber. Erreicht er die Nennspannung, muss er deswegen nicht gut sein. Jetzt kannst du ihn an einen Widerstand anschliessen und die Spannung messen. Nimmt die Spannung sehr schnell ab, ist der Akku defekt. Sodann kannst du die Zeit messen, bis seine Entladeschlussspannung erreicht wird. Ist die sehr kurz, ist er kaputt. Ist sie sehr lang, ist er ok. D.h. die einzige Möglichkeit zur Prüfung ob er wirklich noch gut ist, ist einmal voll aufladen und dann entladen und die Zeit messen. --217.138.222.28 12:35, 2. Feb. 2022 (CET)
Ich halte diese Aussage für fragwürdig. Bei mehrzelligen Akkus kannst Du durch Spannungsmessung während der Ladung die Anzahl intakter und defekter Zellen feststellen, das war's. Bei einer Einzelzelle kannst Du während der Ladung den Innenwiderstand messen und dann anhand des Datenblattes entscheiden, ob die Zelle noch gut ist. --Rôtkæppchen₆₈ 18:06, 2. Feb. 2022 (CET)
Nur mit einem Multimeter kann man eben nicht den Innenwiderstand messen. Da funktioniert nur die sehr umständliche Methode, die Batterie aufzuladen und dann über einen Widerstand zu entladen. --77.10.161.222 03:55, 6. Feb. 2022 (CET)

In welchem Artikel findet man etwas zu den Anforderungen an ein (gutes) Gesetz?

Neben der Verfassungsmäßigkeit, sollte/muss(?) ein Gesetz verschiedene Kriterien erfüllen: Zieldefinition (meist § 1, wenn dort nicht der Anwendungsbereich); Geeignetheit, das beabsichtigte Ziel zu erreichen; Subsidiarität, stattlicher Eingriff nur, wenn nicht andere abhelfen können; Verhältnismäßigkeit, Zielerreichung mit möglichst geringem Aufwand/Nebenwirkungen.--Wikiseidank (Diskussion) 14:28, 2. Feb. 2022 (CET)

In Liste der Anforderungen an gute Gesetze (zu Recht rot). Du hast ein paar Aspekte genannt. Diese und andere können in jedem Land und von jedem einzelnen anders bewertet werden. Was jemand als "gut" ansieht, ist zum großen Teil Ansichtssache. Z.B. ist die Subsidiarität ein Prinzip, das eher in freiheitlichen Staaten als gut angesehen wird. Vor kurzem gabs hier eine Frage zum sozialen Wohnungsbau: private Vorhaben fördern (eher susidiär) oder staatlich organisieren (weniger susidiär). Was ist besser? 62.157.2.126 14:50, 2. Feb. 2022 (CET)
Siehe Artikel Rechtsförmlichkeit mit einigen Verweisen. --Rudolph Buch (Diskussion) 14:56, 2. Feb. 2022 (CET)
Rechtsförmlichkeit (eben) formal. Mir geht es eher inhaltliche Anforderungen, um das Maß an Rechtsstaatlichkeit und Freiheit einzuschätzen/nachzuweisen (und nicht nur zu behaupten).--Wikiseidank (Diskussion) 08:06, 3. Feb. 2022 (CET)
Laien-Verständlichkeit wäre vielleicht mal ein Kriterium, dass berücksichtigt werden sollte. Immerhin sollen Gesetze ja von Laien und nicht von Juristen befolgt werden. :-) 80.71.142.166 09:20, 3. Feb. 2022 (CET)
Laienverständlichkeit ist IMHO kein Kriterium. Das Gesetz muss juristisach präzise sein. Für Küchenjura sind Präambel und/oder Kommentar zuständig. Auch gekünstelte geschlechtergerechte Sprache ist eher schlecht, wie die schwerverständliche, geschlechtergerechte Straßenverkehrsordnung beweist. --Rôtkæppchen₆₈ 13:21, 3. Feb. 2022 (CET)
Zehn Gebote: kurz & bündig, aber doch umfassend! 178.142.227.35 13:30, 3. Feb. 2022 (CET)
Die sind kein Gesetz, das hätten die Christen nur gerne. Solche Verhaltensregeln gibt es in vielen Religionen und zur Gesetzeskraft schaffen sie es nur in fundamentalreligiösen Staaten. Sie sind dann das genaue Gegenteil von einer guten Gesetzgebung, zu deren Grundsätzen die Wahrung des Rechtsfriedens und die Gleichbehandlung aller Bürger zählen. –Falk2 (Diskussion) 13:55, 3. Feb. 2022 (CET)
Gottes Gebot ist nicht Gesetz? Na... (Was die "fundamentalistischen" antiken Juden des AT natürlich nicht daran hinderte, noch umfängliche Kommentare und anderweitige juristische Schriften dazuzuschreiben. Ihre Nachfolger tun es ihnen nach: das CIC ist auch ziemlich dick. Bei den Protestanten werden die Ausführungsbestimmungen "ausgelagert", s. z. B. die Einleitung der Schweizer Bundesverfassung.) --77.10.161.222 04:12, 6. Feb. 2022 (CET)
Natürlich nicht. Wie soll jemand, den es nicht gibt, Gesetzgebungskompetenz erlangen? –Falk2 (Diskussion) 04:46, 6. Feb. 2022 (CET)

Hintergrundstrahlung

380.000 Jahre nach dem Urknall entsprach die Hintergrundstrahlung einer Temperatur von 3000 Kelvin. 13.810.000.000 Jahre nach dem Urknall entspricht die Hintergrundstrahlung einer Temperatur von 2,725 Kelvin. Mit welcher Gleichung kann man den zeitlichen Verlauf dieser Temperatur beschreiben? Sinkt diese Temperatur mit dem Kehrwert des Alters des Universums, oder sinkt diese Temperatur mit dem Kehrwert des Volumens des Universums? Die interessanteste Frage ist aber: Wann hatte das Universum rund 300 Kelvin? -- Karl Bednarik (Diskussion) 08:33, 3. Feb. 2022 (CET).

An sich kann es ja wegen des Energieerhaltungssatzes nur etwas mit dem Volumen zu tun haben: Die Temperatur wird sozusagen „ausgedünnt“, wobei natürlich zu beachten ist, dass im Laufe der Zeit die Energie auch auf andere Art gebunden sein kann als am Anfang. Es entstanden ja Atome usw., die sehr viel Energie in sich binden, die dann nicht mehr als Temperatur ablesbar ist. 80.71.142.166 09:17, 3. Feb. 2022 (CET)
Die Temperatur (=Energie der einzelnen Photonen = Wellenlänge der Strahlung) der Hintergrundstrahlung ergibt sich nur aus der Kosmologischen Rotverschiebung seitdem sie aufgetreten ist (Das Auftreten war ja nur einmalig, als das Universum von einem undurchsichtigen Plasma hin zu transparent wurde). Diese hängt mit der Ausdehnung des Raumes zusammen die das individuelle Photon auf seinem Weg zu uns gesehen hat (und dabei deren Wellenlänge streckt). Zwar nimmt durch die Ausdehnung auch das Volumen zu und damit auch die Dichte der Photonen ab, das betrifft aber nicht die Temperatur, sondern die Strahlungsleistung bzw Wärme pro Zeit (=Energie der Summe der auftreffenden Photonen pro Zeit). Um die Frage zu Beantworten muss man also "nur" die Rotverschiebung ausrechnen (und davor die Temperatur in Wellenlänge und später wieder zurück rechnen).--Naronnas (Diskussion) 10:27, 3. Feb. 2022 (CET)
Hier ein Video dazu: Wie misst man die Temperatur der Hintergrundstrahlung? 80.71.142.166 22:06, 3. Feb. 2022 (CET)
Das ist ziemlich unangenehm, dass die Frage ausgerechnet jetzt gestellt wird. Wir haben eben erst festgestellt, dass sich die Ausdehnung des Universums beschleunigt, statt langsamer zu werden (Dunkle Energie). Und wir haben noch keine Ahnung, ob sich die Rate der Ausdehnung proportional zu irgendwas verändert, d.h. da sind so viele Einflussfaktoren, dass du mit dieser linearen Denke kaum zu brauchbaren Ergebnissen kommen wirst. Klär' doch schon mal das mit der Dunklen Energie. Dann sind wir einer Antwort auf deine Frage schon deutlich näher. Yotwen (Diskussion) 15:10, 5. Feb. 2022 (CET)
Nimm es nicht so tragisch. Die Frage ist doch offenbar nur dafür gestellt, um eine SF-Geschichte einen einigermaßen glaubwürdigen Hintergrund zu verschaffen, daher ist es einigermaßen egal, was hier in der AU vermutet wird. --2001:871:F:25E:9D3F:68B2:260:4641 18:59, 5. Feb. 2022 (CET)
Eine solche SF-Geschichte gibt es schon: Das Drei-Minuten-Universum (Star Trek Classic, Band 46). http://fictionfantasy.de/dasdreiminutenuniversum -- Karl Bednarik (Diskussion) 03:47, 6. Feb. 2022 (CET)

Verbrechen nach Ablauf der Zeit

T-Rex hat da einen interessanten Gedanken. Gibt es denn, abgesehen vom genannten Fall, Handlungen, die nach deutschem Recht strafbar sind und nach Ablauf einer bestimmten Zeitspanne (ohne dass sich ansonsten etwas geändert hat) nicht mehr strafbar sind?

--2A02:8109:BD40:65C4:8D3A:9F33:28FA:9911 10:14, 3. Feb. 2022 (CET)

Siehe Verjährung (Deutschland) -- Nightflight to Venus (Diskussion) 10:25, 3. Feb. 2022 (CET)
Es geht nicht um die Dauer, nach der eine vorgenommene Tat nicht mehr strafrechtlich verfolgt wird, sondern um die Dauer, nach der eine Tat zu begehen den Straftatbestand an sich aufhebt.--2A02:8109:BD40:65C4:8D3A:9F33:28FA:9911 11:44, 3. Feb. 2022 (CET)
Straftat bleibt Straftat. Sogar Straftaten, die nach heutigem Gesetz keine Straftaten mehr sind (z. B. Rassenschande, Majestätsbeleidigung), bleiben Straftaten nach dem jeweiligen zu der Zeit gültigen Gesetz. Aufgehoben ist die Straftat damit nicht, aber aufgehoben ist das entsprechende Gesetz, somit auch die Strafverfolgung, abgesehen von der Verjährungsfrist. Seither kann man also den König straffrei beleidigen.--Giftzwerg 88 (Diskussion) 11:59, 3. Feb. 2022 (CET)
Majestätsbeleidigung ist eigentlich immer noch strafbar: Verunglimpfung des Bundespräsidenten. Der muss allerdings selbst die Verfolgung veranlassen. Die "Beleidigung" ausländischer Regierungen allerdings ist seit Ende 2017 nicht mehr strafbar, wohl wegen der Böhmermann-Affäre. --Universal-InteressierterDisk.Arbeit 12:05, 3. Feb. 2022 (CET)
Die Unterscheidung zwischen "wird ab Zeitpunkt X nicht mehr verfolgt/geahndet" und "ist ab Zeitpunkt X nicht mehr strafbar" erscheint zumindest mir rein akademisch. Aber wenn die Frage so zu verstehen ist, dann hat Giftzwerg 88 im Allgemeinen Recht und die Antwort lautet "eigentlich nie" (ein Grab auszuheben erfüllt - nach meinem IANAL-Verständnis den Sachverhalt nach § 168, aber Archäologen und Friedhofsangestellte sind wohl dazu "befugt") Was allerdings eintreten kann ist, dass der Gesetzgeber nach Aufhebung eines Straftatbestandes diesen auch "im Nachhinein" aufheben kann, ggf. unter Einschränkungen. Prominentes Beispiel ist § 175. --Nightflight to Venus (Diskussion) 12:19, 3. Feb. 2022 (CET)
Nach deutschen Recht gibt es auch für die Störung der Totenruhe keinen Zeitablauf für die Strafbarkeit, wie der Fragesteller annimmt (s. den schon verlinkten § 168 StGB). Wenn Archäologen bei einer ordnungsgemäßen Ausgrabung auch eine Leiche ausgraben, trifft der Tatbestand nicht zu (unbefugt aus dem Gewahrsam des Berechtigten wegnehmen oder beschimpfenden Unfug verüben). 62.157.2.126 13:07, 3. Feb. 2022 (CET)
Anekdote: in Bremen gab es vor einigen Jahren einen skurrilen Fall. Da hatte ein Hobby-Archäologe Totenschädel, mutmaßlich von Bombenkriegsopfern, ausgegraben und sich damit auch noch stolz in einem - nun ja - "Presseerzeugnis" präsentiert und dann prompt was von der Landesarchäologin aufs Dach gekriegt - aber nicht wegen "Störung der Totenruhe", sondern wegen Verstoß gegen das Altertümergesetz, oder wie das heißt - archäologische relevante Funde sind nämlich der Behörde anzuzeigen und dürfen nicht ausgegraben oder zerstört werden. (Die Frau Halle war so richtig sauer und in Rage. Ich fand's auch nicht so prall: es leben noch genügend Leute, von denen die Schädel, die dieser durchgeknallte Aasgeier in die Kamera gehalten hatte, Opa/Oma oder sonstwas hätten sein können.) --77.10.161.222 04:38, 6. Feb. 2022 (CET)
Um wieder auf die Eingangsfrage zurückzukommen: Im T-Rex-Beispiel sind einfach die verletzten Rechte verblasst. Ähnlich wäre das Verunglimpfen des Andenkens Verstorbener, hier dürften die Rechte spätestens mit den letzten Antragsberechtigten, den Enkeln, erlöschen. Die Störung der Totenruhe hält länger, wird aber gegen andere Interessen abgewogen, im Beispiel gegen die der Archäologen. --Seewolf (Diskussion) 13:11, 3. Feb. 2022 (CET)
Und selbst die Archäologen graben (mittlerweile!) sehr selten aus Lust oder Wissenshunger. Die Abwägung ist meist eine andere: Werden die Skelette einfach vom Bagger weggeschaufelt, da dort ein Wohnhaus (Schule, Supermarkt, Fabrik) entstehen darf, oder wird das ganze sorgfältig geborgen, analysiert und die Knochen dann später an anderer Stelle wieder begraben? Von daher ist auch der Comic nicht richtig. --195.200.70.51 13:19, 3. Feb. 2022 (CET)
Ich denke Sachbeschädigung wäre ein treffendes Beispiel. Wenn du z.B. mit einem Hammer auf ein neues Auto einprügelst, machst du dich strafbar. Wartest du aber lang genug, und zwar bis der Zeitwert des Autos so weit gesunken ist, dass er durch zusätzlichen Schaden nicht weiter sinkt, dürfte die Sache anders aussehen. --94.216.227.11 00:02, 4. Feb. 2022 (CET)

Das Urheberrecht ist ein anderes Beispiel für eine Änderung der Rechtslage im Zeitablauf. Wenn vor Ablauf der Schutzfrist ein geschütztes Werk genutzt werden soll, haben die Rechteinhaber einen Anspruch. Nach Ablauf der Schutzfrist besteht dieser Anspruch nicht mehr.--Vertigo Man-iac (Diskussion) 13:57, 3. Feb. 2022 (CET)

Wenn ich das richtig verstehe, geht es in der Frage doch um den Ablauf von Schutzfristen. Beispiel aus dem Comic: ab wann darf man ein Grab wegbuddeln, anderes Beispiel: ab wann darf man ein urheberrechtlich geschütztes Werk kopieren. --87.182.92.30 14:14, 3. Feb. 2022 (CET)

Der TE fragte nach Verbrechen, nicht nach Ordnungswidrigkeiten oder Vergehen.--84.130.90.208 14:34, 3. Feb. 2022 (CET)

Sagen wir mal Konsum von Cannabis ist strafbar, wird aber legalisiert, dann fällt auch der Grund für die Inhaftierung wegen Cannabiskonsum weg. Es wäre absurd, jemand dafür weiterhin in Haft zu halten. Aber es gibt auch neue Straftatbestände, die früher nicht geahndet wurde, z. B. illegale Straßenrennen.--Giftzwerg 88 (Diskussion) 20:33, 3. Feb. 2022 (CET)
Ich glaube, diese Unterschiede machen praktisch nur Juristen und ich muss davon ausgehen, dass viele den zwischen Vergehen und Verbrechen gar nicht kennen. Die Unterscheidung der Straftaten in Vergehen und Verbrechen ist durchaus fließend und sie ergibt sich häufig erst im Verlauf eines Strafverfahrens. –Falk2 (Diskussion) 22:34, 3. Feb. 2022 (CET)

Amtshilfe zu Wahlkreisartikeln gesucht

Ich nutze mal das Potential der Auskunft. In der Weimarer Republik entsandte jeder Wahlkreis mehrere Abgeordnete in den Reichstag, tw. bis zu 20. Es gab allerdings auch reichsweit nur 35 Wahlkreise. Kennt jemand vergleichbare Wahlsysteme mit Wahlkreisartikeln dazu, wo ich mir die WK-Artikel mal anschauen kann?--scif (Diskussion) 15:23, 3. Feb. 2022 (CET)

Gab es auch in den USA, hier der Artikel zum entsprechenden Wahlsystem: en:Plural district, und auch: en:Electoral district#District magnitude. Ob die en WP auch Artikel zu einzelnen solcher Wahlbezirke hat, kann ich auf die Schnelle nicht sehen; in den beiden genannten Artikeln sind jedenfalls, soweit ich sehe, keine verlinkt. --Proofreader (Diskussion) 15:35, 3. Feb. 2022 (CET)
Ist das so ungewöhnlich? In Bayern gibt es aktuell 7 Wahlkreise, die zwischen 16 und 61 Abgeordnete in den Landtag entsenden. Und wenn Du nicht an der exakten Bezeichnung hängst, gibt es ja auch auf Bundesebene 16 Wahlkreise mit jeweils 5 bis 155 Abgeordneten. --Rudolph Buch (Diskussion) 15:54, 3. Feb. 2022 (CET)
Auch bei Gemeinderats- und Kreistagswahlen gibt es sowas: Für jeden Teilort bzw Ort werden soundsoviel Mitglieder in den Gemeinderat bzw Kreistag gewählt. --Rôtkæppchen₆₈ 16:02, 3. Feb. 2022 (CET)
Für zwei der deutschen Wahlkreise habe ich vor Jahren mal nds:Wahlkreis Süüdhannover-Bruunswiek und nds:Wahlkreis Oosthannover angelegt. --::Slomox:: >< 16:09, 3. Feb. 2022 (CET)
Mir ging es eigentlich weniger um irgendwelche Spitzfindigkeiten sondern eher um das Layout der Artikel...--scif (Diskussion) 16:31, 3. Feb. 2022 (CET)
Das italienische Wahlrecht ist derart kompliziert, dass es eigentlich alle hier bereits diskutierten und noch zu diskutierenden Fälle mit abdecken müsste. Dort leistet man sich bspw. Regionalwahlkreise, die dann wiederum auf Mehrpersonenwahlkreise verteilt werden. Zum Überdruss dann auch in den Regionen überall etwas unterschiedlich. Und dann mischt man dort auch noch relative und proporzionale Wahlsysteme. 80.71.142.166 22:11, 3. Feb. 2022 (CET)
Wahlkreise in Finnland. --147.142.218.189 10:13, 4. Feb. 2022 (CET)
Bei den Bürgerschafts- (=Landtags-)Wahlen in Bremen ist das Land in zwei Wahlbereiche (nämlich Stadt Bremen und Bremerhaven) eingeteilt, die getrennt eine bestimmte Anzahl von Abgeordneten wählen. (Besonderheit: die stadtbremischen Bürgerschaftsmitglieder sind zugleich kommunale Ratsleute, d. h. sie bilden die "Bürgerschaft Stadt", die ggf. auch getrennt von der "Bürgerschaft (Landtag)" tagt.) Ob jetzt das "Stadtbremische Überseehafengebiet Bremerhaven" bei den Wahlen zum Wahlbereich Bremen oder Bremerhaven gehört, bin ich überfragt. --77.10.161.222 04:50, 6. Feb. 2022 (CET)

China Olympia Boykott?

Hallo,

man hört viel dass China keine demokratischen Werte beachtet und die Olympiade nur Propagande ist, warum macht man keinen Boykott wie damals bei den SPielen in Moskau? Alternativ konnte man doch die SPiele spontan an die USA vergeben?--00:42, 4. Feb. 2022 (CET) (unvollständig signierter Beitrag von 77.12.25.153 (Diskussion) )

1. Weil man das nicht will. 2. Weil das so schnell nicht geht (und weil man das [beim dafür zuständigen IOC] nicht will). --BlackEyedLion (Diskussion) 01:23, 4. Feb. 2022 (CET)
Sport ist Sport und Politik ist Politik. Darauf achten alle Sportverbände weltweit. Sport ist Völkerverständigung dort, wo die Politik versagt. Es ist unfair, diejenigen für eine verfehlte Politik zu bestrafen, die dafür am wenigsten können. --Rôtkæppchen₆₈ 02:04, 4. Feb. 2022 (CET)
Eine kurzfristige Vergabe an eine andere Stadt wäre nicht so ohne Weiteres möglich und würde auch gigantische Entschädigungszahlungen nach sich ziehen. Theoretisch könnte man bei der zukünftigen Vergabe von Sportveranstaltungen strengere Kriterien anlegen. Nur welche sollten das sein? Und wie vermittelt man den Ländern der Erde, dass zwar jedes Land gerne Athleten schicken soll, aber nur eine kleine Auswahl sich auch als Gastgeber bewerben darf. --Carlos-X 02:22, 4. Feb. 2022 (CET)

wenn es kein anderer macht: Das ist eine Pre-Troll-Frage oder von einem unerfahrenen Menschen ohne Lebenserfahrung und/oder ohne Medienkompetenz. Wenn man es bis zur Wikipedia schafft, findet man über die Suche, Artikel zu allen Begriffen (die ich diesmal nicht verlinke).--Wikiseidank (Diskussion) 08:59, 4. Feb. 2022 (CET)

Wer ganz undedarft ist, der kommt nicht zu Wikipedia, sondern googelt erstmal und nimmt dann das erste Google-Angebot, das oft auf Wikipedia sitzt. Wer gezielt Wikipedia anwählt, dann auch noch das für Unbekannte gar nicht leicht auffindbare Autorenportal und schließlich die ebenfalls dort versteckte Wikipedia:Auskunft, der ist nicht unbedarft, der sucht gezielt; aber manche eben nicht eine Antwort, sondern ein Gesprächsthema. Selbst die unbedarft erscheinende Syntax, Grammatik und/oder Rechtschreibung kann Verstellung sein, hier oben sogar sehr wahrscheinlich (und bitte jetzt nicht fragen, woher ich das denn wissen will). --2003:D0:2F18:5777:F997:C374:FDD5:3870 09:12, 4. Feb. 2022 (CET)
+1 ;) -Ani--46.114.152.4 10:36, 5. Feb. 2022 (CET)

Der Boykott der Spiele von Moskau (bemerkenswerterweise als Reaktion auf den Einmarsch in Afghanistan) gilt als Fehlschlag, weil politisch nichts bewirkt wurde. Mehr dürfte ein Boykott der diesjährigen Spiele auch nicht bewirken. Im Übrigen hatte das IOK nur die Auswahl zwischen Peking und Almaty, unter anderem weil die Münchner Olympische Spiele in München abgelehnt haben. Wenn sich nur Diktaturen bewerben, darf man sich nicht wundern, wenn die Spiele in einer Diktatur stattfinden. --Mhunk (Diskussion) 09:46, 4. Feb. 2022 (CET)

Zumindest eine Diktatur hat aber gemerkt, dass sportliche und kulturelle Großveranstaltungen immer negative Publicity bringen, egal ob das jetzt Formel 1, Eurovision Song Contest, Europaspiele sind: Aserbaidschan und die aserbaidschanische Erdölgesellschaft Socar ziehen sich aus internationalen sportlichen und kulturellen Großprojekten zurück. --Rôtkæppchen₆₈ 09:59, 4. Feb. 2022 (CET)
Das "man" in der Frage zeigt schon, dass der Anfrager keine Ahnung von Demokratie etc. hat. Die OS werden vom IOC veranstaltet, den "man", der das verändern könnte, gibt es nicht. Und trotz aller berechtigter Kritik an den OS in Peking, es ist gut, dass es den nicht gibt. --Hachinger62 (Diskussion) 11:47, 4. Feb. 2022 (CET)
Man könnte natürlich alternative Sportturniere veranstalten, wie das zum Teil 1936 der Fall war. Das würde aber wahrscheinlich dazu führen, dass die Verbände, die dort antreten, von den Olympischen Spielen ausgeschlossen werden oder dass Sportler, die dort individuell antreten, aus ihren Verbänden ausgeschlossen werden. --Digamma (Diskussion) 18:47, 4. Feb. 2022 (CET)

Solange es die OS gibt, gibt es schon immer den Widerstreit zwischen Politik und Sport. Das Gedächtnis scheint aber recht kurz, denn auch das bundesdeutsche NOK spielte da nicht immer die beste Rolle. Man wird den Widerstreit nie lösen können, vor allem, weil da viel Heuchelei von allen Seiten dabei ist. WM laufen ohne viel Diskussionen in Diktaturen ab, bei Olympia macht man dann das große Faß auf. Von Bach könnte man sich sicherlich klarere Worte wünschen, aber er ist der Präsident aller NOK´s nicht nur der westeuropäischen. Und wer meint, das ein Dudu von Bach ein Jota an der chinesischen Haltung zu bestimmten Fragen ändert, nun der lebt in einer Traumwelt.--scif (Diskussion) 22:38, 6. Feb. 2022 (CET)

Frithjof Karlsson

Der Autor Konrad Hansen hat Hochdeutsch und Plattdeutsch geschrieben. Ca. 1984 hat er das plattdeutsche Theaterstück Schöne Utsichten veröffentlicht, das als Übersetzung eines Stückes von Frithjof Karlsson angegeben wird. Ich habe versucht, mehr über diese Person herauszufinden und hatte arge Schwierigkeiten. Die DNB hat einen GND-Eintrag zu ihm (1121924638), in dem angegeben ist, dass er von 1898 bis 1975 gelebt hat und einen Bezug zu den Ålandinseln hat. Als Quelle ist Frithjof Karlsson: Schöne Utsichten (THP-Hamburg 1983/84) angegeben. THP konnte ich spontan nicht entschlüsseln. Oder bedeutet es schlicht Theaterprogramm? Uraufführung war 1984 im Ohnsorg-Theater ([10]).

Schwedische und finnische Wikipedia kennen einen solchen Autoren aber nicht und auch das restliche Internet zeigt wenig Anzeichen für seine Existenz. Beim Googeln bin ich dann auf Konrad Hansens Roman Simons Bericht von 2002 gestoßen, wo ein Frithjof Karlsson von Åland vorkommt, der auf Poel lebt. Allerdings spielt der Roman im Jahr 1402 zur Zeit der Hanse... Muss ich davon ausgehen, dass Frithjof Karlsson ein Gag von Konrad Hansen ist und er Schöne Utsichten selber geschrieben hat? Kann jemand mit der DNB-Quelle etwas anfangen? --::Slomox:: >< 22:49, 5. Feb. 2022 (CET)

Wenn er es selbst geschrieben hat, hat er alle sehr gut in die Irre geführt. Als eigentlichen Autor finde ich Frithjof Karlsson, s.a. DNB des Stückes. Konrad Hansen wird als Bearbeiter der niederdeutschen Fassung genannt, auch bei der Fernsehausstrahlung am 9.2.1985 unter dem hochdeutschen Titel. Das Stück wird auch für Theateraufführungen verkauft [11], vielleicht kann der Verlag mehr zu Autor und der Erstveröffentlichung (längere Zeit vor 1984?) sagen. Ob die Spur nach Åland überhaupt richtig ist, sollte geprüft werden. Vielleicht wurde beim Autor und der Romanfigur etwas durcheinander gebracht. 91.54.47.93 12:49, 6. Feb. 2022 (CET)
Was spricht dagegen, dass Frithjof Karlsson ein Pseudonym von Konrad Hansen ist? Das Stück wurde in Buchform herausgebracht (1983) und uraufgeführt (1985), als Konrad Hansen zufällig gerade Intendant des uraufführenden Theaters war (1980–1985). Da sollte es ein Leichtes sein, fiktive Lebensdaten zu seinem Pseudonym in das Theaterprogrammheft zu schmuggeln. Zudem erschien das Stück in einem Theaterverlag und Bühnenvertrieb Karl Mahnke Verden (Aller).[12] --Rôtkæppchen₆₈ 13:06, 6. Feb. 2022 (CET)
Zur Romanfigur in Simons Bericht: „Er stammte, hieß es, von den Ålandinseln“ [13]. Neben der niederdeutschen (Schöne Utsichten) und hochdeutschen (Schöne Aussichten) erschien 1995 auch eine schweizerdeutsche Fassung (Heiteri Ussichte) von Carl Hirrlinger: [14]. Die GND ist hier – bedingt durch die schlechte Quellenangabe (THP-Hamburg 1983/84) – kein gutes Hilfsmittel. Mein Gedanke war daher, bei Carl Hirrlinger anzufragen, wem genau er die Tantiemen bezahlt, aber von dem ist auch nichts bekannt [15].--Pp.paul.4 (Diskussion) 15:20, 6. Feb. 2022 (CET)
Von Carl Hirrlinger gibt es zumindest ein Foto.[16] --Rôtkæppchen₆₈ 18:12, 6. Feb. 2022 (CET)

Ein Film mit Yul Brynner ohne Yul Brynner

Ich bin verwirrt und verlassen und deshalb ist schon der Titel dieses Abschnittes irreführend, denn ich weiß eigentlich gar nicht, ob der gesuchte Film mit oder ohne Yul Brynner ist. Es ist aber ein Film, der so aussieht, als wäre er mit Yul Brynner (wenn das überhaupt Sinn ergibt). Zunächst: Es handelt sich hundertprozentig nicht um "Westworld". Es könnte sich um einen dieser "italienischen Abklatschfilme" handeln. In dem Film kommen Roboter (?) in einer Westernstadt (?) vor, die von Besuchern (?)erschossen werden dürfen. Und nun das Allesentscheidende: Die Roboter (?) haben vorne und hinten auf ihren Hemden, direkt dort, wo das Herz sitzt, ein rotes Kreuz aufgenäht. Zum Zielen (deshalb kann es auch nicht Westworld sein, weil dort gibt es diese roten Kreuze nicht). Wie heißt dieser Film? Für jeden Hinweis bin ich sehr dankbar. Vielleicht gibt es in dem Film auch keine Westernstadt und keine Roboter. Es könnte sich auch um Gangkriege in einem Science-Fiction-Setting handeln. Aber diese roten Kreuze, die gibt es auf jeden Fall. Ich habe diesen Film nur ein einziges Mal in meiner frühen Jugend geschaut und bin seither auf der Suche. Wie heißt der Film mit den roten Kreuzen auf den Hemden? Vielen, vielen Dank! --77.11.134.247 15:29, 6. Feb. 2022 (CET)

In Welcome to Blood City (1977) haben die Schauspieler "red crosses on the front and back – they become literally walking targets". --Pp.paul.4 (Diskussion) 17:03, 6. Feb. 2022 (CET)

Federung Fahrradgabel

An meinem Fahrrad habe ich an der Vordergabel in der Härte verstellbare Gabelbeine(?). Je weicher man sie einstellt, desto höher ist der Fahrkomfort. Aber wieso gibt es diese Einstellung eigentlich? Hat es was mit erhöhtem Kraftaufwand beim Treten zu tun, wenn man sie weicher einstellt? Irgendeinen Grund wird es ja geben, wieso es diese Einstellung gibt. Oder hat es andere Nachteile, wenn sie weich eingestellt sind? Ich kann mir bisher keinen Reim darauf machen. Jedenfalls merke ich beim Fahren keinen höheren Kraftaufwand beim Treten, bei verschiedenen Härtegraden. Weiß hier jemand Bescheid?--2A02:810C:1C0:1BD4:F918:95D:75A6:1F51 22:19, 5. Feb. 2022 (CET))

Verluste und Geschmackssache. Dass Du den Unterschied nicht merkst, bedeutet nicht, dass es ihn nicht gibt. Bei einem kräftigen Antritt gibt die Federung nach, statt das in Vortrieb umzusetzen. Eine vernünftige Federung hat Dämpfer, in denen beim Einfedern Bewegungsenergie in Wärme umgewandelt wird. Diese Energie ist für den Vortrieb verloren, muss aber aufgebracht werden. Auf ebener Fahrbahn ist die Federung nicht nötig. Ich persönlich benutze die Federung nur auf ruppigem Untergrund, weil mich das weiche Fahrgefühl ansonsten nervt. --46.78.147.226 08:16, 6. Feb. 2022 (CET)
Anschlussfrage (von einem anderen User): Bei einem Ferngesteuerten Auto kann eine zu weiche Federung/Dämpfung dazu führen, dass das Auto mit dem Unterboden aufschlägt. Kann das eigentlich auch bei einem Fahrrad passieren? (will es lieber nicht ausprobieren! :D ) --77.11.134.247 08:47, 6. Feb. 2022 (CET)
Nein.--Wikiseidank (Diskussion) 09:02, 6. Feb. 2022 (CET)
Eine vergleichbare Fehlkonstruktion ist mir bei Fahrrädern unbekannt. Aber: Natürlich kann bei ungünstiger Kurbelstellung das Pedal an Hindernissen hängen bleiben. Sogar ohne Federung. --46.78.147.226 09:03, 6. Feb. 2022 (CET)
Uphill: Hart. Downhill: Weich. (Und unten angeschrabbelte Pedale zeugen von ambitionierten Kurvenfahrstil.) 87.167.100.61 10:18, 6. Feb. 2022 (CET)
Ja, etwa im Wiegetritt beim Bergauf- oder auch Gegenwindfahren stört ein Eintauchen der Gabel nur und schluckt unnötig Energie. -- Fice (Diskussion) 10:35, 6. Feb. 2022 (CET)
Beim Bremsen in der Kurve kann eine Federung zum Sturz führen, sie kann aber auch einen Sturz verhindern. --Ralf Roletschek (Diskussion) 18:24, 6. Feb. 2022 (CET)
Wow. --87.167.100.61 00:21, 7. Feb. 2022 (CET)

Bin ich Sichter?

An einen Admin: Bin ich befähigt/ist es mir erlaubt, Wikipedia-Änderungen im Artikelnamensraum zu sichten? Wenn ja (aber auch wenn nicht), wo sehe ich diesen Status? Bin nicht so oft als Bearbeiter hier, meistens lese ich nur. Gerade eben habe ich aber den Artikel Schneidkeramik bearbeitet (kleine Änderung, ein Typo und einen chemischen Namen etwas klarer hervorgestellt). Wo müsste ich klicken, um die Änderung zu sichten? --DerMarkus1982 (Diskussion) 11:40, 6. Feb. 2022 (CET)

Hinweis: Für Fragen speziell zur Wikipedia gibt es ein eigenes Diskussionsforum: Fragen zur Wikipedia.

Du gehörst zur Gruppe "Passiver Sichter". Was du selbst editierst, gilt automatisch als gesichtet. Nutze deine Macht weise! ;) Die Artikeländerung hast du aber offenbar unangemeldet als IP getätigt. Grüße Dumbox (Diskussion) 11:47, 6. Feb. 2022 (CET)
"Passiver Sichter". Roger that!
Ja, ich hatte mich aus Versehen im falschen Tab angemeldet, im anderen Browser :D
Danke, Leute.
Meine Frage sehe ich als beantwortet an. --DerMarkus1982 (Diskussion) 13:00, 6. Feb. 2022 (CET)
Du könntest auf "Sich-ten" klicken. Aber überfordere Dich nicht.💅 (nicht signierter Beitrag von 89.204.154.172 (Diskussion) 11:53, 6. Feb. 2022 (CET))
Ich habe als aktiver Sichter (immer noch weit unten in der Hierarchie) die IP-Änderungen gesichtet, weil sie augenscheinlich kein Vandalismus sind. Ob sie sinnvoll sind, wird auf einer höheren Ebene entschieden. Grüße Dumbox (Diskussion) 12:03, 6. Feb. 2022 (CET)
Wie und warum auch immer, diese Änderung wurde von einer IP vorgenommen. Vermutlich warst du abgemeldet, ohne es zu merken. Dann kann das System natürlich nicht wissen, dass dort ein User mit Sichterrechten sitzt :) Generell kannst du (und jeder andere) deine Rechte unter Spezial:Benutzerrechte einsehen. Und wenn du wirklich mal einen Admin brauchst (hierfür brauchst du keinen), dann findest du sie am besten unter WP:AA. --Kreuzschnabel 12:29, 6. Feb. 2022 (CET)
Spezial:Benutzerrechte.
Macht Sinn. Danke. --DerMarkus1982 (Diskussion) 13:03, 6. Feb. 2022 (CET)
Nicht, daß es mich ernsthaft interessiert, aber offenbar können passive Sichter dann wohl doch aktiv sichten, indem sie dort, wo sie fremde Edits sichten wollen, einfach noch eine Kleinigkeit ändern und damit dann automatisch alle vorangegangenen fremden Edits mitsichten. Und später machen sie ihre eigene Änderung ggf. wieder rückgängig. --77.10.161.222 22:26, 6. Feb. 2022 (CET)
Nein, das funktioniert nicht so. Bearbeitet ein passiver Sichter eine gesichtete Version, so wird dieser Edit automatisch gesichtet. Wird allerdings eine nicht gesichtete Version (egal ob nun nur die letzte(n) Version(en) ungesichtet sind, oder der gesamte Artikel noch nie gesichtet wurde), so ist die neue Version ebenso nicht gesichtet. --178.165.172.77 23:45, 6. Feb. 2022 (CET)
Das dürfte dann zu einer Inkonsistenz führen, wenn ein User ohne Rechte einen Edit A vornimmt und dann der passive Sichter an dem gleichen ungeänderten Abschnitt den Edit B. Aber: nicht mein Problem... --77.8.86.111 09:04, 7. Feb. 2022 (CET)
Nein, beim Bearbeiten bekommst du immer die aktuelle Version in den Editor, ob gesichtet oder nicht. Und im geschilderten Fall wird beim Veröffentlichen dazu aufgefordert, den vorherigen Edit zu sichten. Ein paar Hirnzellen kannst du den Leuten, die so was konzipieren, schon zutrauen :) --Kreuzschnabel 10:43, 7. Feb. 2022 (CET)

Hauptflüsse des Amazonasbeckens - Rio Anapu?

Liebe Geographie- und Südamerika-Expert:innen,

ich unterrichte zur Zeit fachfremd Geographie und soll/will mit meinen Achtklässler:innen Südamerika - insbesondere den Amazonas-Regenwald - behandeln. Ich habe Unterrichtsmaterial aus dem Verlag an der Ruhr erworben und fand darin die Information, dass die Hauptflüsse des Amazonasbeckens, neben dem Amazonas selbst, der Rio Anapu und der Rio Tocantins seien. Da ich gerne vorbereitet bin, wollte ich die beiden mir unbekannten Flüsse auf der Karte suchen und stellte fest, dass der Rio Anapu über weite Teile so klein zu sein scheint, dass er noch nicht mal auftaucht. Auf keiner einzigen Amazonien-Übersichtskarte, die ich im Internet gefunden habe, ist er verzeichnet. Ich musste sehr explizit in Google Maps suchen und sehr weit reinzoomen, um ihn zu entdecken.

Nun meine Frage: ist der Rio Anapu wirklich so bedeutend, dass er zur Definition des Begriffes "Amazonasbecken" dienen kann? Der einzige Hinweis darauf ist die Einleitung des deutschen Wikipedia-Artikels https://de.wikipedia.org/wiki/Amazonasbecken - die am 2.6.2009 erstellt und seitdem nicht mehr wesentlich verändert wurde. Ich finde praktisch keine Sekundärliteratur im Internet, die diese Behauptung stützt - nur Plagiate des erwähnten Wiki-Artikels. Selbst im portugiesischen Artikel https://pt.wikipedia.org/wiki/Bacia_do_rio_Amazonas kommt das Wort "Anapu" nicht vor.

Nun bin ich kein Geograph und verfüge über keine Fachliteratur oder entsprechende Kompetenzen, das zu beurteilen. Ich weiß auch nicht, an wen ich mich diesbezüglich wenden soll. Daher schreibe ich einfach mal hier und schaue, ob sich ein:e Geographie- und/oder Südamerika-Expert:in findet, der*die dieser Fragestellung einmal qualifiziert nachgehen könnte.

Vielen Dank im Voraus und herzliche Grüße! Thomas --84.166.216.185 11:04, 4. Feb. 2022 (CET)

Hallo Thomas, du hast recht. Leider ist die ensprechende Autorin seit 8 Jahren verschwunden, so dass wir nicht nachfragen können - aber offensichtlich hat irgendjemand ohne Quellen das mal in die Wikipedia geschrieben. Das geht so nicht und ich habe die Einleitung vom Amazonasbecken entsprechend geändert. -- southpark 12:05, 4. Feb. 2022 (CET)
Es steht aber noch im Abschnitt Geographie. In en:Amazon basin steht mitten im Artikel: "The Tocantins basin, arguably not part of the Amazon basin ..." Ist das ein Hinweis auf abweichende Ansichten? 91.54.47.93 12:15, 4. Feb. 2022 (CET)
Hallo southpark. Vielen Dank für die Rückmeldung und das Ändern. Es ist ein wenig erschreckend, wie lange diese Information jetzt schon unbemerkt hier drinsteht. Wie viele Schüler:innen und sonstige Laien dem wohl schon aufgesessen sind...? Thomas --84.166.221.131 23:18, 7. Feb. 2022 (CET)
Hallo Thomas, Du kannst auch hier nachsehen und selbst beurteilen, wie wichtig das ist. --46.79.42.178 18:07, 4. Feb. 2022 (CET)
Habe ich das richtig verstanden, dass die Angaben in den Unterrichtsmaterialien auf dem Wikipedia-Artikel beruhen? --Digamma (Diskussion) 18:55, 4. Feb. 2022 (CET)
Tss, tss, tss... --77.10.161.222 05:13, 6. Feb. 2022 (CET)
Und irgendwann wird das vielleicht jemand hier wieder hereinsetzen, mit dem Unterrichtsmaterial aus dem Verlag als „reputabler Quelle“. --2003:D0:2F18:5738:5D36:7DF6:7B58:6F83 12:08, 6. Feb. 2022 (CET)
Hallo Digamma. Ich fürchte, so ist es, ja. Der besagte Abschnitt in dem Arbeitsblatt scheint eine Paraphrasierung der Einleitung des Wikipeda-Artikels zu sein. Anders kann ich mir das eigentlich nicht erklären, denn, wie gesagt, finde ich Hinweise auf den Rio Anapu sonst nirgendwo. Thomas. --84.166.221.131 23:15, 7. Feb. 2022 (CET)

Größe eines alten Buches

Ein Antiquar gibt die Größe mit "in-8" an. Was bedeutet das in cm? --2001:16B8:6601:5300:F49E:896:325C:FD52 11:31, 4. Feb. 2022 (CET)

42. Was ist die Größe? Höhe, Breite, Dicke, Buchdeckelfläche, Volumen? --Magnus (Diskussion) 11:35, 4. Feb. 2022 (CET)
Das sagt weniger über die Grösse als über die Art des "Faltens", vgl. Oktavformat und - dort steht die Abk. explizit - en:Octavo. --Nightflight to Venus (Diskussion) 11:45, 4. Feb. 2022 (CET)
Siehe auch Papierformat#Historische_europäische_Formate. --L47 (Diskussion) 15:22, 4. Feb. 2022 (CET)
Oder Buchformat, da findet sich eine übersichtliche Tabelle. Gestumblindi 20:57, 6. Feb. 2022 (CET)

Das dürfte das Oktavformat sein. Ganz grob etwa die Größe DIN A5 --Elrond (Diskussion) 13:46, 7. Feb. 2022 (CET)

Was sind Beispiele für unregelmäßige Verben?

Unser Artikel verwirrt mich. Ich finde dort Beispiele für regelmäßige Verben, aber nicht für unregelmäßige. Und das "unregelmäßige Verben in der Sprachwissenschaft nicht einheitlich gehandhabt werden"--Wikiseidank (Diskussion) 18:49, 4. Feb. 2022 (CET)

Ich habe da gerade mal einen Wartungsbaustein gesetzt, der Abschnitt ist wirklich nicht sehr gut geschrieben. Generell scheint deine Frage hier aber falsch platziert zu sein und eher in die Disk zum Artikel zu gehören. Was deine Frage nach Beispielen betrifft: Hast du dir im Wiktionary schon mal das hier angesehen? Gruß, --Stilfehler (Diskussion) 19:06, 4. Feb. 2022 (CET)
In dem Artikel finde ich folgende Beispiele für unregelmäßige Verben: sein, haben, gehen, stehen, dürfen, können, mögen, müssen, wissen, wollen. --Morino (Diskussion) 19:14, 4. Feb. 2022 (CET)
Macht der Begriff im Deutschen überhaupt Sinn? Ich kenne ihn eigentlich nur aus dem Englischen. --Digamma (Diskussion) 19:20, 4. Feb. 2022 (CET)
Wer behauptet, Sinn zu machen, macht nichts als Unsinn.
Unregelmäßige Verben gibt es in vielen europäischen Sprachen. Denk an französisch var und spanisch ir, beides in erster Linie gehen. Das beste und naheliegenste deutsche Beispiel ist sein. –Falk2 (Diskussion) 19:34, 4. Feb. 2022 (CET)
Was soll "var" für eine französische grammatische Form von "gehen" sein? --95.208.204.172 13:33, 7. Feb. 2022 (CET)
Im Deutschunterricht wurde versucht(!), Endungen bei Partizip 1 und Partizip 2 mit den Unterschieden bei regelmäßigen und unregelmäßigen Verben zu erklären.--Wikiseidank (Diskussion) 21:11, 4. Feb. 2022 (CET)
Das verstehe ich nicht. Das Partizip 1 (wenn damit das Partizip Präsens aktiv gemeint ist) wird doch in allen Fällen völlig regelmäßig gebildet. Das Partizip Perfekt hat hingegen bei unregelmäßigen Verben oft einen eigenen Verbstamm (z.B. gegangen, gedacht, gewesen, ...). --Digamma (Diskussion) 22:38, 4. Feb. 2022 (CET)
Das verstehe ich. Nach unserem Artikel haben Lehrer für das Partizip 1 SECHS verschiedene Begriffe, für Partizip 2 sogar ACHT. Wie sollen Schüler das verstehen und wie sollen Zentralklausuren verstanden werden?--Wikiseidank (Diskussion) 09:58, 5. Feb. 2022 (CET)
Zumindest in Mathe ist es bei uns so, dass in den Lehrplänen die Begriffe festgeschrieben sind. --Digamma (Diskussion) 10:06, 5. Feb. 2022 (CET)
Im genannten Artikel gibt es zu den vier Verbklassen je zwei Beispiele: suppletive Verben wie sein und gehen, Präteritopräsentia wie müssen und dürfen, Rückumlautverben wie brennen und kennen, starke Verben wie schreiben und trinken. Der Artikel Germanisches schwaches Verb kennt im Übrigen zwei dieser Klassen nicht. --BlackEyedLion (Diskussion) 22:46, 4. Feb. 2022 (CET)
Wer starke Verben als unregelmäßig bezeichnet, beweist zunächst einmal, dass er keine Ahnung von der deutschen Sprache (bzw. Sprachgeschichte) hat, denn Verben wie schreiben, reiben, treiben und bleiben konjugieren eigentlich sehr regelmäßig, nur der Komplexität ihrer Unterklassen wegen (und der Tatsache, dass man die Zuordnung auswendig lernen muss, es ist einfach zu erkennen, ob ein Verb stark oder schwach ist) werden sie als unregelmäßig gelehrt. Das ist auch das Problem des Begriffes des unregelmäßigen Verbes: Damit werden die verschiedensten Formen zusammengefasst. --Universal-InteressierterDisk.Arbeit 23:13, 4. Feb. 2022 (CET)
Ich würde sagen „Wer starke Verben als unregelmäßig bezeichnet, beweist zunächst einmal, dass er keine Ahnung von der deutschen Sprache (bzw. Sprachgeschichte) hat“ und „nur der Komplexität ihrer Unterklassen wegen […] werden sie als unregelmäßig gelehrt“ widersprechen sich. Ich weiß, dass es im Deutschen im Wesentlichen starke und schwache Verben gibt und habe diese Bezeichnungen mit den Bezeichnungen unregelmäßig und regelmäßig identifiziert. Der Fragesteller hat in seiner Frage gezeigt, dass er gerne diese Bezeichnungen sowie die Bezeichnungen aus dem Artikel Unregelmäßiges Verb verwenden möchte. Ich halte es im Übrigen für gerechtfertigt, starke Verben als unregelmäßig zu bezeichnen, weil man die Beugung schwacher Verben als Regel bezeichnen und alle Ausnahmen davon, auch wenn sie einer anderen Regel folgen, als dieser Regel widersprechend, also unregelmäßg bezeichnen kann. --BlackEyedLion (Diskussion) 23:47, 4. Feb. 2022 (CET)
Vielleicht eine "Intelligenzstufe" tiefer: Wie erkennt man dass bspw. "haben" (hat, hatte) unregelmäßig und "reiten" (ritt, geritten) regelmäßig ist?--Wikiseidank (Diskussion) 09:41, 5. Feb. 2022 (CET)
Die Unregelmäßigkeiten bei Verben wie "haben" bestehen ja nicht nur in den Präteritum- und Partizip-Perfekt-Stämmen, sondern auch in den Formen "hast" und "hat". Das sind doch beim Lernen die größeren Probleme. Genauso wie die Präsensformen z.B. bei "geben", "du gibst", "er gibt" usw. --Digamma (Diskussion) 09:48, 5. Feb. 2022 (CET)
"geben" ist im Artikel aber regelmäßig, Trotz gleichartiger Veränderung?--Wikiseidank (Diskussion) 10:03, 5. Feb. 2022 (CET)
Inwiefern wäre „reiten – ritt – geritten“ regelmäßig? Schwache Konjugation kann damit nicht gemeint sein, die lüte „reiten – reitete – gereitet“. --Kreuzschnabel 13:34, 5. Feb. 2022 (CET)
Regelmäßig ist ein Verb, das nach einer erkennbaren Regel, das heißt, so wie andere Verben gleicher Art, konjugiert wird. Neben reiten gibt es noch schreiten und gleiten, die werden genauso konjugiert, also gibt es da eine Regel. Unregelmäßig sind Verben, die so konjugiert werden, wie kein anderes Verb außer ihren eigenen Komposita. Also z.B. sein und haben. MfG --Φ (Diskussion) 13:42, 5. Feb. 2022 (CET)
Kann man sagen, dass nicht das Verb regelmäßig ist, sondern seine Verlaufsform regelmäßig, also gleichartig ist (oder abweichend/unregelmäßig)?--Wikiseidank (Diskussion) 14:44, 5. Feb. 2022 (CET)
Das ist aber eine doofe Regel, wenn sie auf weiten, leiten oder (zu)bereiten, die im Infinitiv ebenso gleichartig sind, nicht zutrifft. --Kreuzschnabel 18:05, 5. Feb. 2022 (CET)
Weil es in den germanischen Sprachen nun einmal starke und schwache Verben gibt, deren Zuordnung nicht regelmäßig ist, die aber zumindest historisch beide regelmäßig konjugierten. Lustiger Fakt übrigens: "leiten" ist ursprünglich ein (immer schwach konjugierter) Kausativ zu "leiden", das wie die genannten Beispiele zur 1. starken Klasse gehört und bis auf den 2. Konsonanten im Präsensstamm gleich konjugiert, im Niederländischen (rijden, glijden, lijden) sogar vollständig, dazu siehe Zweite Lautverschiebung. (Zu einer kurzen Recherche über die Entwicklung einzelner Wörter empfehle ich immer das englische Wiktionary.) --Universal-InteressierterDisk.Arbeit 01:06, 7. Feb. 2022 (CET)
Es ging mir nicht um die Herkünfte der Unterschiede, sondern um die Behauptung, ein Verb sei schon dann regelmäßig, wenn es überhaupt andere Verben gebe, die nach demselben Schema gebeugt werden. Das halte ich für angreifbar; eine Regel erkenne ich frühestens dann, wenn ein Großteil der gleichartigen Verben (hier: Infinitive reimen sich auf -eiten) dem Schema folgt. Das ist hier IMHO nicht gegeben, ich habe ebensoviele Gegenbeispiele angeführt – dann kann ich auch nicht sagen, reiten sei regelmäßig, weil schreiten und gleiten genauso konjugiert werden. --Kreuzschnabel 08:44, 7. Feb. 2022 (CET)
Die Rede von „unregelmäßigen Verben“ ist nur sehr bedingt sinnvoll; im Grunde nur dann, wenn Suppletion oder Analogie vorliegt. Sie erweckt den Eindruck, es gäbe Formen, die einfach so vom Himmel gefallen sind. Jedes Lehrbuch führt ziehen (zog, gezogen) als unregelmäßiges Verb, dabei hat es sich historisch vollkommen regelmäßig entwickelt. Demgegenüber ist, historisch gesehen, schlagen unregelmäßig, da der grammatische Wechsel durch Analogie generalisiert wurde. --GALTZAILE PPD () 18:40, 5. Feb. 2022 (CET)
Die Rede von unregelmäßigen Verben ist dann sinnvoll, wenn man eine Sprache lernt und sich entscheiden muss, welche Konjugationen man sich gesondert reinpauken sollte und welche man sich sparen kann. Das hat natürlich nicht (direkt) mit Linguistik und Sprachgeschichte zu tun, aber es ist trotzdem auf eine sehr praktische Weise real. --2003:ED:AF05:8058:EFD9:44A9:B1D3:AEB8 18:56, 5. Feb. 2022 (CET)
Definitiv unregelmäßig sind suppletive Verben wie "sein": "ich bin (Indikativ Präsens, 1. Person Singular), ich war (Indikativ Präteritum), er ist (Indikativ Präsens, 3. Person Singular), du seiest (Konjunktiv I (Präsens), 2. Person Singular)" hat definitiv keinerlei Regelmäßigkeit. Aber auch hier zeigt sich eine im gesamten Neuhochdeutschen zu beobachtende Einheitlichkeit und sehr starke Regelmäßigkeit: Ich beziehe mich jetzt nur auf die "eigentlichen" konjugierten Verbformen, nicht auf die aus mehreren verschiedenen zusammengesetzten: Infinitiv, Partizip I und Partizip II sind weitgehend alleinstehende Verbformen, wobei alle Infinitive auf "-n" enden (mit Ausnahme von "sein" und "tun") und das Partizip I immer gleich gebildet wird. Präteritum (Indikativ), Konjunktiv I und Konjunktiv II haben (wenn man es so sehen will) immer die gleichen an einen Verbstamm angehängten Flexionsendungen (nach Person und Numerus des Subjekts): Je nachdem, ob man das "e" dem Stamme zurechnet: gar keine Endung (oder e, das aber nicht bei starken Präterita und dem Konjunktiv I von sein) in der 1. und 3. Person Singular, -(e)st in der 2. Person Singular, die Infinitivendung -en in der 1. und 3. Person Plural und -(e)t in der 2. Person Plural.
Die unregelmäßigsten Verbformen im Deutschen sind wohl "wir sind" und "sie sind"[A 1], die in ihrer Endung von allen anderen Verbformen der 1. und 3. Person Plural abweichen, und "ich bin, du bist, er ist" die als einzige Präsensformen (Indikativ) keinen "äußerlichen Bezug" zu ihrem Infinitiv aufweisen
Für alles findet man wenigstens Gruppen vergleichbarer Verben, so gibt es beispielsweise viele Verben mit unterschiedlichen Präsensstämmen (Indikativ Präsens, Imperativ, Konjunktiv I, Infinitiv und Partizip I) und Präteritumsstämmen (Indikativ Präteritum, Konjunktiv II und Partizip II; hier dann auch normalerweise stark mit teilweise deutlich abweichendem Partizip), das Thema der Präteritopräsentia, Rückumlautverben und starken Verben mit ihren durchaus vorhandenen Regelmäßigkeiten wurde hier schon ausgiebig diskutiert --Universal-InteressierterDisk.Arbeit 00:49, 7. Feb. 2022 (CET)
  1. oder vielleicht besser Die unregelmäßigsten Verbformen im Deutschen dürften wohl "wir sind" und "sie sind" sein? Hmmm?

Anschlussfrage
Warum verzichtet man in der Lehre von Sprachen nicht vollständig auf theoretische Regelvermittlung und lässt Lernende sprechen, sprechen und nochmals sprechen bzw. schreiben, schreiben und nochmals schreiben und korrigiert ständig, gerne auch mit digitaler Unterstützung (Autokorrektur). Irgendwann erkennen Lernende wiederkehrendes/Regeln. Dann (bei Fortgeschritten(d)en;o), kann man um theoretische Regelkunde erweitern?--Wikiseidank (Diskussion) 10:11, 5. Feb. 2022 (CET)

Grosartik! Dass Shrajbä-wi-du-shprichz-Debahkell auf Fremmtschprahchenn ausgewajtät. --2003:F7:DF08:9C00:E41C:432A:D9F0:BA9E 12:41, 5. Feb. 2022 (CET)
Nein, dass ist Rechtschreibung. Satzbau, Verlaufsform und Endungen "maken the problems" bzw. "Schwierigkeiten die machen". Eindeutige und logische Regeln können/sollten selbstverständlich von Beginn an vermittelt werden.--Wikiseidank (Diskussion) 14:44, 5. Feb. 2022 (CET)
Grammatik benötigst du für Textanalysen. Verschiedene Stilmittel nutzen grammtische Regeln. Andere nutzen den Wortschatz. Es kommt also auf die Zielsetzung an. Und dafür muss man begleitend mit der Sprachentwicklung Grammatik lernen. Man könnte behaupten, dass die Vielfalt der Sprache aus der Kombination von Grammatik und Wortschatz entsteht, was sich im Gebrauch wiederspiegelt, den jeder davon macht, was wiederum von der Kenntnis und der Nutzung der Werkzeuge abhängt, die man dir in den Mund gelegt hat. Yotwen (Diskussion) 14:52, 5. Feb. 2022 (CET)
So lernen Kinder Sprachen. Erwachsene lernen Sprachen (angeblich?) leichter bzw. schneller über die Grammatik (z.B. [17], [18]). Als deutscher Muttersprachler kann ich zumindest sagen, dass mich die deutsche Grammatik nie groß interessiert hat und ich mich schwer tue, diesbezügliche Fragen von Deutschlernern zu beantworten. Bei "meinen" Fremdsprachen "kenne" ich aber die Grammatik. Es gibt z.B. Sprachen, in denen es "ich helfe dich" oder "ich glaube dich" heißt. Da ist es halt einfacher und effektiver, die Liste der abweichenden Verben einmal richtig zu lernen, als einfach draufloszureden. Hinzu kommt oft das Problem, dass man nicht verbessert wird, aus Höflichkeit oder um das Gespräch nicht ständig zu unterbrechen, und sich dann ganz schnell falsche Sachen einprägen. --88.64.172.178 17:58, 5. Feb. 2022 (CET)
Ich denke, das wäre sehr ineffizient. Nach meinem Dafürhalten wäre es am zielführendsten, nicht nur die Regeln, sondern das System zu vermitteln, aus dem heraus sich die Regeln erklären: so viel verstehen wie möglich, so wenig lernen wie nötig. --GALTZAILE PPD () 18:40, 5. Feb. 2022 (CET)
Ich glaube ja, dass sich viele deshalb schwertun mit Latein, weil man Latein gerade nicht so lernt, sondern praktisch nur über die Regeln. --Digamma (Diskussion) 17:47, 6. Feb. 2022 (CET)
Ich glaube, dass (konventionell) alle Sprachen über Regeln gelehrt werden, der Unterschied bei Latein ist nach meinem Dafürhalten eher, dass mit komplexen Texten gearbeitet wird und eigene Produktion praktisch nicht stattfindet. --GALTZAILE PPD () 18:54, 6. Feb. 2022 (CET)
Liest eigentlich irgendwer mal unsere eigenen Artikel, z.B. Immersion (Sprachwissenschaft und Erziehung) oder Berlitz Sprachschulen? --Stilfehler (Diskussion) 17:56, 6. Feb. 2022 (CET)

Ein Physiker und ein Mathematiker erhalten vom Dekan den Auftrag, die Höhe des Fahnenmasts vor dem Haupteingang der Universität zu ermitteln. Die beiden treffen sich also vor dem Gebäude, um den Gegenstand ihres Auftrags zu begutachten. Sie fachsimplen, was wohl die beste Methode sein, den Auftrag zu erledigen, als plötzlich ein Kollege von der Ingenieurswissenschaftlichen Fakultät heraneilt.
"Was macht ihr denn da?" fragt er.
"Der Dekan hat uns beauftragt, die Höhe des Fahnenmasts zu ermitteln," erwidern sie
Der Ingenieur zückt ein Massband, geht zum Mast, hebt diesen aus seiner Halterung, legt ihn auf den Boden, misst ihn ab, stellt den Mast wieder auf und sagt:"4 Meter 50." Sodann eilt er weiter zu seiner Vorlesung.
Sagt der Physiker verächtlich:"So ein Idiot! Wir sollen die Höhe ermitteln, und er misst die Länge!"

unbekannter Autor
Wir reden hier über Sprachen, nicht über Lesen! Yotwen (Diskussion) 12:29, 7. Feb. 2022 (CET)

Reagiert der menschliche Körper mit negativem Stress bei Immunantworten?

Ich kann mir nicht vorstellen, dass das Hirn da irgendwie sich nicht dran juckt wenn plötzlich alles schmerzt, die Temperatur hoch geht etc. Hat da jemand eventuell Links zu Studien (gerne Cochrane)? Habe schon ein wenig gestöbert aber leider bisher nichts brauchbares gefunden; meist immer nur Stress der sich negativ auf das Immunsystem auswirkt, was dann ja ein Kreislauf der wechselseitigen Verstärkung wäre, wenn das wonach ich frage existiert. --KleinerKorrektor (Diskussion) 18:41, 4. Feb. 2022 (CET)

Deine Frage ist wirr. Mach Dir mal klar, daß eine Immunantwort i.d.R. die Reaktion auf einen Stressor ist. Natürlich ist eine Immunantwort mit einem gewissen Aufwand verbunden, aber das gilt für sämtliche Körperfunktionen, darum würde man zumindest eine physiologische Immunantwort nicht als Streß bezeichnen. --94.219.22.201 01:12, 5. Feb. 2022 (CET)
Oh eine feige Stänker-IP, wie putzig. --KleinerKorrektor (Diskussion) 10:13, 5. Feb. 2022 (CET)
Hä? Die Antwort auf deine Frage muss doch mit einer Definition von Stress beginnen, und nach gängiger Meinung reagiert das Immunsystem auf Stressoren. Der Körper aktiviert Energie in die Abwehr, die er nicht mehr in seine sonstigen Zellen, wie gewohnt in stressfreier Zeit, investieren kann, also hat der Körper Stress. Wenn sich jetzt noch dein Hormonhaushalt ändern und deine Muskeln sich verspannen sollten, weil dein Immunsystem aktiv ist, dann hätte der Körper noch zusätzlichen Stress (und es hängt davon ab, bei welcher Intensität das Wort Stress angemessen erscheinen darf, Auslegungssache). Die IP hat m.E. richtig geantwortet.--Blue 🔯 11:20, 5. Feb. 2022 (CET)
Die Definition von (Dis)stress findet sich im jeweiligen Artikel und nach der habe ich hier auch nicht gefragt. Studien, Wissenschaft sind die Schlagworte, welche weder von Stänker-IP noch von dir beachtet wurden, mich interessieren Meinungen oder Ansichten hier nicht, ich will Fakten, das geht aus meiner Fragestellung im ersten Post hier auch deutlich hervor. Grüße --KleinerKorrektor (Diskussion) 14:29, 5. Feb. 2022 (CET)
@Bluemel: Kleimer Korrektor schrieb ja nicht das die Antwort der IP falsch sei. Ich nehme an das ihm die Einstiegsphrase "Frage/wirr" nicht gefallen hat. Und da es sich bei dem Antwortenden um die bekannte 77er/88er/94er-IP handelt,die grundsätzlich herablassend in Diskussionen einsteigt, ist seine Einschätzung dazu nicht ganz falsch. Wer permanent anderen vor den Kopf stößt dessen u.U. richtiger Ansatz gerät eben schnell mal in den Hintergrund. Ist schon ok so. Der Ton macht die Musik. -Ani--46.114.156.207 14:11, 5. Feb. 2022 (CET)
Richtig! ;) --KleinerKorrektor (Diskussion) 14:29, 5. Feb. 2022 (CET)
Aha. "Negativer Stress" - hat die IP erstens erklärt und zweitens, welchen Stress soll es denn sonst auslösen?--Blue 🔯 14:32, 5. Feb. 2022 (CET)
Nur mal als Hinweis: Wer nach erneutem daraufhinweisen, welche Posts nicht zielführend und oder erwünscht sind, solche immer noch postet, wird im Internet schnell als Troll klassifiziert. Und folgendes entspricht tatsächlich der Bezeichnung wirr: einen Wikipedianer (mehrfach) um Belege bitten... Oh man... Wenn da nichts vernünftiges mehr von dir kommt ignoriere ich dich ab jetzt.--KleinerKorrektor (Diskussion) 14:50, 5. Feb. 2022 (CET)
Einverstanden. Sollen doch die Experten der Sache nachgehen, ob sich (ich zitiere) „das Hirn dran juckt“. Peace.--Blue 🔯 16:16, 5. Feb. 2022 (CET)
Mehr Expertise also von mir wird der TO nicht bekommen, weil sein gedanklicher Ansatz völlig daneben ist und es zu Nonsens nunmal keine seriöse wissenschaftliche Forschung gibt. Ich empfehle den Janeway zum Selbststudium und anschließend einen bösen Leserbrief an Springer, weil darin seine unsinnige Hypothese nicht behandelt wird. --94.219.12.84 00:31, 7. Feb. 2022 (CET)
Es ist nicht wirklich unsinnig. Endogene Symtpome können, wenn es einer erforschen möchte, durchaus etwas mit Immunabwehr zu tun haben (stärkeres endogenes Ekzem etc.), was genau er meint, wenn es bis jetzt nicht befriedigend war, weiß man nicht.--Blue 🔯 09:45, 8. Feb. 2022 (CET)

Hintergrundstrahlung, Teil zwei

Mit welcher Gleichung kann man den zeitlichen Verlauf der Temperatur der Hintergrundstrahlung beschreiben?
Kann das folgende richtig sein?
1.) 2,725 Kelvin = 15.685.401 / ( 13.810.000.000 Jahre ^ ( 2 / 3 ) )
2.) 2990 Kelvin = 15.685.401 / ( 380.000 Jahre ^ ( 2 / 3 ) )
3.) 299 Kelvin = 15.685.401 / ( 12.000.000 Jahre ^ ( 2 / 3 ) )
Die beiden ersten Zeilen sind die Ausgangswerte der Überlegung.
15.685.401 ist ein konstanter Faktor.
Die dritte Zeile ist eine Vermutung.
Dass es ein Kehrwert sein muss, ist vermutlich richtig.
Zeit hoch ( zwei Drittel ) ist eine Vermutung.
-- Karl Bednarik (Diskussion) 05:54, 5. Feb. 2022 (CET).

Ich gehe mal davon aus, dass sich darüber schon zahlreiche Physiker und Mathematiker Gedanken gemacht haben, die aber 1. weder in WP:AU nach hilfreichen Informationen gesucht haben, noch 2. hier in der WP:AU mitlesen und Deine neuesten Erkenntnisse kommentieren werden. --2001:871:F:25E:9D3F:68B2:260:4641 18:52, 5. Feb. 2022 (CET)
Die Gleichungen 1.), 2.) und 3.) sind in sich konsistent, wenn der Skalenfaktor proportional t^(2/3) angesetzt wird. Nur ist die materiedominierte Epoche, in der das galt, schon seit 4 Milliarden Jahren vorbei, weswegen die Gleichung den zeitlichen Verlauf unzureichend beschreibt. Dass man durch zwei Punkte 1.) und 2.) jede beliebige Kurve legen kann, ist klar. --Pp.paul.4 (Diskussion) 22:59, 5. Feb. 2022 (CET)
Ja, leider fehlt die Information über die Krümmung der Kurve. -- Karl Bednarik (Diskussion) 03:47, 6. Feb. 2022 (CET)
Ich habe noch einen dritten Bezugswert für die Kurve gefunden. Das Universum, noch warm vom Urknall: https://www.spektrum.de/news/kosmisches-thermometer-das-universum-noch-warm-vom-urknall/1978891 16,4 Kelvin, theoretisch vorhergesagt war für die Zeit von 880 Millionen Jahren nach dem Urknall eine CMB-Temperatur von 20 Kelvin.
4.) 17,08 Kelvin = 15.685.401 / ( 880.000.000 Jahre ^ ( 2 / 3 ) )
Langsam glaube ich an die Zeit hoch ( zwei Drittel ). -- Karl Bednarik (Diskussion) 05:02, 6. Feb. 2022 (CET).
Ich habe noch etwas dazu gefunden: https://physics.stackexchange.com/questions/76241/cmbr-temperature-over-time -- Karl Bednarik (Diskussion) 05:52, 6. Feb. 2022 (CET)
https://en.wikipedia.org/wiki/Chronology_of_the_universe#Speculative_"habitable_epoch" -- Karl Bednarik (Diskussion) 07:43, 6. Feb. 2022 (CET).
Diagramm: http://s880616556.online.de/UNIVTEMP.png -- Karl Bednarik (Diskussion) 21:58, 7. Feb. 2022 (CET)
Zur Materiedichte: 13.810.000 Jahre sind 1/1000 des Alters des Universums. Die Temperatur sollte also 100 mal höher sein, also 272,5 Kelvin. Die mittlere Materiedichte sollte 1.000.000.000 mal höher sein, weil das Volumen um den Faktor 1000 hoch 3 kleiner ist, also 2.000.000.000 Wasserstoffatome pro Kubikmeter, anstatt nur 2. Zum Vergleich: 1 Gramm atomarer Wasserstoff hat 6 mal 10 hoch 23 Atome, und bei 0 Grad Celsius und 1 Atmosphäre 22,4 Liter Volumen. Die laue Frühlingsluft des Universums ist viel dünner, mit nur 2 mal 10 hoch 9 Atomen pro Kubikmeter. -- Karl Bednarik (Diskussion) 04:06, 8. Feb. 2022 (CET)

Seltsame Codes unter Windows

Bei einer Bekannten (nicht computer-affin) erscheinen im Startmenü (Win 10) über den Kacheln lauter Plus-Zeichen in einer Reihe. Bei der Suche erscheinen auch lauter Pluszeichen, zudem noch "Ref A:" (Buchstaben- und Zahlencode, "64A0D343C..."), "Ref B:" (Buchstaben- und Zahlencode), "Ref C:" (Buchstaben- und Zahlencode (2022-02-03T17:12:54Z"). Kann jemand weiterhelfen? Gruß --2003:C6:1718:FFBB:C8D5:E7BA:FAF3:CEB3 18:34, 6. Feb. 2022 (CET)

Über den Kacheln: einmal dort bei "Gruppe benennen" eingegeben (rechts beim Sysmbol mit den 2 waagerechten Strichen) - dann bleibt das dort stehen. --TheRunnerUp 18:57, 6. Feb. 2022 (CET)
Ja danke, aber was soll sie dort eingeben. Ich habe jetzt einmal bei mir am Rechner in dem Feld, in dem "Gruppe benennen" steht, Buchstaben eingegeben, aber verändert hat sich dann nichts.
Gruß --2003:C6:1718:FFBB:94A5:947C:339A:20FE 21:17, 6. Feb. 2022 (CET)
Das ist sehr seltsam! Das ist eine ganz einfache Grundfunktion. Was du dort eingibst, sollte auch sofort dort erscheinen, genau so, wie wenn du einen Ordner im Windows-Explorer umbenennst. Oder kannst du das etwa auch nicht? ◅ SebastianHelm (Diskussion) 13:14, 7. Feb. 2022 (CET)
Doch, ich kann Ordner umbenennen. Und Gruppen umbenennen "bei den Kacheln"funktioniert auch. Aber wie meiner Bekannten eine Gruppe umbenennen helfen soll bei ihrem Problem mit den Codes und Plus-Zeichen, ist mir unklar. Gruß --2003:C6:1718:FF00:C14E:24:BB30:C32A 14:03, 7. Feb. 2022 (CET)
Da es sich hier anscheinend um ein sehr seltsames Problem handelt, das aufgrund deiner bisherigen Beschreibung wohl keiner nachvollziehen kann, könnte es vielleicht helfen, es genauer, am besten Schritt für Schritt, zu beschreiben. Da du deine Bekannte als „nicht computer-affin“ bezeichnet, ist im Umkehrschluss anzunehmen, dass du dich selbst als „computer-affin“ bezeichnen würdest. Somit sollten dir solche exakten Problembeschreibungen eigentlich vertraut sein. ◅ SebastianHelm (Diskussion) 11:25, 8. Feb. 2022 (CET)

Wie lange finanziert die norwegische Regierung schon religiöse Gruppen?

Mich interessiert seit welchen Jahr eine Religionsgemeinschaft in Norwegen staatliche Subventionen verlangen und bekommen kann.-31.0.76.68 22:43, 4. Feb. 2022 (CET)

Für den Anfang: 1537 führte Christian III. in Norwegen die Reformation ein. Die evangelische Glaubenslehre erlangte den Status einer Staatsreligion, der erst 2012 ein Ende fand. Also mindestens 1537, wobei zu vermuten ist, die Verquickung zwischen Religion und Herrschaft ist deutlich älter als ein Begriff der "staatlichen Subvention." Oder anders gesagt: seitdem es staatliche Subventionen in Norwegen gibt. -- southpark 10:23, 5. Feb. 2022 (CET)

Hilft nicht weiter... -37.248.213.187 20:50, 6. Feb. 2022 (CET)

Weil die Frage falsch gestellt ist. Das Wort "Subvention" ist anachronistisch. Christian III. hat nämlich nicht nur die Reformation eingeführt und zur Staatsreligion erklärt, sondern sich damit selbst zum religiösen Oberhaupt gemacht. Die Sache mit der Staatskirche war also keineswegs eine uneigennützige "Subventionierung" der Kirche, sondern eine Kumulation von Machtfülle in seiner Hand. Netter Nebeneffekt, hier nachzulesen: Die Kirchengüter fielen bei der Gelegenheit praktischerweise der Krone zu. Wer "subventioniert" da nun wen? --217.239.4.223 09:31, 8. Feb. 2022 (CET)

Es geht trotzdem darum dass in den norwegischen Nachrichten jetzt groß geredet wird dass die Zeugen Jehovas keinen einzigen Pfennig mehr erhalten. Bisher bekamen die die letzten 3 Jahre pro Jahr 16 Millionen Norwegische Kronen als Subvention oder Geschenk. Jede religionsgemeinschaft mit mehr als 50 Mitglieder kann die norwegische Regierung um eine solche finanzielle Hilfe bitten, abgerechnet wird pro Mitglied. Die katholische Kirche wurde z.b. dabei erwischt Menschen in ihren vereinsregister zu haben welche nie der katholischen Kirche beigetreten sind. -5.226.122.218 01:59, 9. Feb. 2022 (CET)

Was hat das alles mit der Ausgangsfrage zu tun? Suchtest Du nur einen Anlass für ein bisschen Kirchenbashing mit Deinen unbelegten Zahlen? Dann geh bitte ins Café, das ist der richtige Ort dafür. --217.239.4.223 09:18, 9. Feb. 2022 (CET)

geht eine CD durch Kälte und Nässe verloren

Geht eine CD durch Kälte und Nässe verloren?--2003:E3:3736:76E:25AE:FB10:5435:1B09 15:44, 6. Feb. 2022 (CET))

Es kommt ganz auf die Lackschicht an. Wenn der Lack im Wasser aufquillt, dann kann Sauerstoff, ggf. auch im Wasser gelöst, an die Aluminiumschicht, die darauf mit Oxidation reagiert. Dadurch sinkt die Reflexivität der Aluminiumschicht und beim Lesen der CD gibt es Lesefehler. Es kann auch sein, dass der aufgequollene Lack abblättert und dabei die Aluschicht mitnimmt. --Rôtkæppchen₆₈ 15:51, 6. Feb. 2022 (CET)
Nach meinem Sprachgefühl geht die CD dann kaputt, aber nicht verloren. Verloren gehen dabei die Daten. --Digamma (Diskussion) 18:06, 6. Feb. 2022 (CET)
Einem Bekannten von mir ging mal durch Kälte und Nässe seine Armbanduhr verloren: Autopanne im Winter, beim Radwechsel Uhr in Schneewehe gefallen, nicht wiedergefunden: weg war sie... Wäre es warm und trocken gewesen, dann hätte sie gut sichtbar auf dem Boden gelegen. --77.10.161.222 18:25, 6. Feb. 2022 (CET)
Durch Nässe, das kann natürlich passieren, z. B. beim nächsten Jahrhunderthochwasser. --2001:871:F:319D:6CD6:E03:97C:4EF2 19:00, 6. Feb. 2022 (CET)
Meine Oma hat überzählige CDs zur Vogelabwehr in die Bäume gehängt. Beschreibbare CD/DVD Rohlinge halten es nicht sehr lange aus, dann lösen sich die Schichten voneinander. Gepresste CDs oder DVDs leben etwas länger, aber kaum mehr als zwei drei Jahre, dann fängt das Aluminium an zu korrodieren. Der Kunststoff selbst steckt die Witterung nahezu unbeeindruckt weg. Da das ohnehin Wegwerf-CDs waren, alte Treiber oder sowas, habe ich nie den Versuch gemacht eine davon wieder auf Lesbarkeit zu testen. --Giftzwerg 88 (Diskussion) 20:47, 7. Feb. 2022 (CET)

Nur der korrekten Vollständigkeit halber und weil das weiter oben eventuell falsch herausgelesen wird: Wenn bei einer gepressten CD „der aufgequollene Lack abblättert und dabei die Aluschicht mitnimmt“ mag ein normaler CD-Player das als „kaputt“ bewerten. Die Daten sind deswegen aber nicht „verloren“, sondern weiterhin vorhanden. --87.147.186.221 22:05, 9. Feb. 2022 (CET)

 
Querschnitt einer industriell gepressten CD
Eine Rettung ist hier sogar möglich: CD vollständig entlacken, ohne den Polycarbonatpressling zu beschädigen, dann neu mit Metall (Gold oder Aluminium) bedampfen und wieder lackieren. Bei der Bedampfung muss die ebene Seite der CD abgedeckt werden, damit sie durchsichtig bleibt. --Rôtkæppchen₆₈ 22:13, 9. Feb. 2022 (CET)
Sicher, wenn man das Wiederverwenden in handelsüblichen CD-Playern zum Ziel hat. Ohne dieses kommt man mit anderen optischen Lesern auch so weiter. --87.147.186.221 23:35, 9. Feb. 2022 (CET)

Verwendung von Ascorbinsäure in der Lebensmittelindustrie

Seit wann wird Ascorbinsäure überhaupt und seit wann weitreichend als Lebensmittelzusatz verwendet?

Außerdem unterscheidet ein Video (über die Lobbygeschichte zum Vitamin C, am Ende dieses Artikels) zwischen D-Ascorbinsäure und L-Ascorbinsäure. Erstere sei nicht als Vitamin wirksam, wäre aber bei dem ersten Synthetisierungsverfahren als Produkt rausgekommen. Meine Frage zielt darauf ab, dass ja immer wieder von verschiedensten Seiten diese Lobby-Behauptung, es herrsche ein Mangel an Vitamin C und es würde helfen, wiederholt wird. Noch immer werden Produkte mit "+ Vitamin C" beworben. Gleichzeitig ist Ascorbinsäure (in welcher Form) in vielen haltbar oder schöner zu machenden Produkten im Einsatz. (sogar im Fleisch als Umrötungsmittel: "Die eingesetzte Menge der Ascorbinsäure beträgt 300-500 mg pro kg Fleisch").

Ist die als "Ascorbinsäure" deklarierte Zutat als Vitamin wirksam und 'sind wir nicht alle überversorgt'? Diese Seite sagt zu Ersterem ja. (Eigentlich gesucht um die Bezeichnung E300 rauszusuchen :) --Amtiss, SNAFU ? 17:00, 3. Feb. 2022 (CET)

Zur vorletzten Frage: Die allerersten Syntheseverfahren ergaben Racemat. Es war also nur die Hälfte der Substanz Vitamin C, die andere Hälfte D-(−)-Ascorbinsäure, die nicht als Vitamin, sondern nur als Reduktions-, Antioxidations- oder Umrötungsmittel taugte. Die heutigen Syntheseverfahren basieren alle auf der Reichstein-Synthese oder biologischen Verfahren und führen so alle zu reiner L-(+)-Ascorbinsäure. --Rôtkæppchen₆₈ 22:46, 3. Feb. 2022 (CET)
Die Werbekampagnen über die segensreiche Wirkung von Vitamin C, etwa bei Erkältungskrankheiten, sind ein Nachhall der Thesen der sogenannten orthomolekularen Medizin, eine Methode der "Alternativmedizin", populär gemacht vor allem durch den Einsatz des Nobelpreisträgers Linus Pauling. Da wird heute viel suggeriert, aber wohlweislich wenig konkretes versprochen. Vitamin C hilft gegen Skorbut. Wobei und wogegen es in welcher Dosierung ggf. außerdem noch hilft, ist nicht so ganz eindeutig. Wer es in seiner Produktdarstellung nicht auf den Glorienschein des Vitamins abgesehen hat, verwendet eben die chemische Bezeichnung.--Meloe (Diskussion) 09:27, 4. Feb. 2022 (CET)
Die Studie unter doi:10.1002/14651858.CD000980.pub4 hat herausgefunden, dass Vitamin C zwar nicht die Inzidenz von Erkältungen mindert, aber wenn es vorbeugend genommen wird die Dauer einer Erkältung kürzer ist. --Rôtkæppchen₆₈ 09:35, 4. Feb. 2022 (CET)
Cochrane, also als Quelle erste Wahl. "The failure of vitamin C supplementation to reduce the incidence of colds in the general population indicates that routine vitamin C supplementation is not justified, yet vitamin C may be useful for people exposed to brief periods of severe physical exercise. Regular supplementation trials have shown that vitamin C reduces the duration of colds, but this was not replicated in the few therapeutic trials that have been carried out." Das klingt nicht sehr enthusiastisch. In jedem Fall wäre jeder positive Aspekt von "Vitamin C" exakt genauso gegegeben, wenn ich (L-)"Ascorbinsäure" zuführe. Ihre Hinzufügung als Lebensmittelzusatz erfolgt aus anderen Gründen, aber die Wirkung wäre gleich.--Meloe (Diskussion) 09:42, 4. Feb. 2022 (CET)
Allgemein zu Vitaminen: wenn eines wirklich fehlt, dann funktioniert der Körper nicht mehr richtig. Mangel an Vitaminen nahm bis vor wenigen Jahren zu (aktuelle Zahlen habe ich leider nicht zur Hand). Ein krasses Beispiel: https://www.aerzteblatt.de/blog/105718/Fast-Food-kann-blind-machen / Allgemeiner: Hypovitaminose
Es ist nun nicht so, dass Vitamin C vor Erkältungen schützt. Aber ein Mangel "stresst" den Körper, und er hat mehr Probleme die Infektion zu Bekämpfen.
Vitamin C kann mit der Ernährung und normal dosierten Vitaminpräparaten nicht überdosiert werden, da es sich als wasserlösliches Vitamin nicht anreichert im Körper. Bei Dosierungen wie sie Linus Pauling zu sich nahm, gibt es aber dann schon schädliche Effekte [19]. -- Tschimu (Diskussion) 10:44, 4. Feb. 2022 (CET)
"Mangel an Vitaminen nahm bis vor wenigen Jahren zu " - ist das so? Die DGE sagt: DGE: Deutschland ist kein Vitaminmangelland - beim google bin ich zwar auf lauter Seiten gestoßen die etwas anderes behaupten. Zufälligerweise kann ich da dann auch gleich noch die passenden Präparate kaufen .oO ...Sicherlich Post 11:38, 4. Feb. 2022 (CET)
Stand meines Wissens: Ja. Auf sehr tiefem Niveau bei sehr alten Menschen, Veganern, ...). Also vor ~20 Jahren gab es einfach noch ein paar mehr Unterversorgte z.B. mich. Ich muss auch ein Vitamin zuführen und habe entsprechende Infos vom Arzt erhalten. Wie auch im von dir zitierten Artikel erwähnt, ist die Versorgung in einigen Altersgruppen z.T. nicht üppig. Unterschritten werden die Referenzwerte für Folat und Vitamin D .... Z.B. in Heimen wurden in den letzten Jahren teilweise Speisepläne angepasst um die Versorgung zu verbessern. -- Tschimu (Diskussion) 11:53, 4. Feb. 2022 (CET)
Das DGE kann vieles sagen, solange kein Tagesbedarf festgelegt ist, sie zu niedrig angesetzt sind, weil die Forschung dazu noch gar nicht so weit ist. Individuell vs. Gesellschaft: Ich bin auch auf Granit gestoßen bei der B12-Thematik beim BfR. Die haben die Obergrenzen für den Zusatz so niedrig gehalten, dass die Durchschnittbevölkerung zwar kein Problem hat, aber wer das nicht über die Ernährung aufnimmt, muss die Grenzwerte für den Zusatz in Lebensmitteln ignorieren oder täglich 4x was nehmen, in der Hoffnung dass der IF das schon regelt... Sorry. PS: langfristige Überdosierung mit Vitamin C ist mit Nierensteinen assoziiert. --Amtiss, SNAFU ? 17:24, 4. Feb. 2022 (CET)
@Amtiss:, zur Ausgangsfrage: Irwin Stone reichte 1935 drei Patente zur Verwendung von Ascorbinsäure als Lebensmittelzusatzstoff ein. Erst zwei Jahre zuvor hatte Tadeusz Reichstein die Synthese von L-(+)-Ascorbinsäure/Vitamin C entwickelt. --Rôtkæppchen₆₈ 14:23, 5. Feb. 2022 (CET)
Also zumindest die Verwendung als Umrötungsmittel ist segensreich. Dadurch konnte nämlich der Nitritanteil für den gleichen Zweck deutlich gesenkt werden und damit die Krebsrate. Das ganze Bild sieht manchmal besser aus, wenn auch nicht so schön, wie die Werbung es malt. Yotwen (Diskussion) 15:05, 5. Feb. 2022 (CET)

Neben der Wirkung als Vitamin ist Ascorbinsäure wichtig, um Eisen in den Körper zu kriegen. Es gibt unter Normalbedingungen zwei "Sorten" von Eisenverbindungen. Solche, in denen das Eisen dreiwertig ist und solche, in denen es zweiwertig ist. In Gegenwart von Sauerstoff ist die dreiwertige Form stabiler. Der Körper kann zweiwertiges Eisen aber weit besser aufnehmen als dreiwertiges, oder umgekehrt ausgedrückt, das stabilere dreiwertige Eisen wird schlechter aufgenommen. Ascorbinsäure wandelt (reduziert) dreiwetiges Eisen in zweiwertiges, stabilisiert diese Form zudem und trägt so in nennenswertem Umfang zur Eisenversorgung bei. --Elrond (Diskussion) 13:41, 7. Feb. 2022 (CET)

Steht da wirklich die Reduktion im Vordergrund oder nicht durch eher der Schutz vor Oxidierung? Ich habe es vor Jahren mal so gelesen, daß das oxidative Millieu im Dünndarm das Fe2+ versaut und es deswegen gut ist, wenn das Eisen zusammen mit Ascorbinsäure aufgenommen wird, um das Fe2+ "abzuschirmen", damit es gar nicht erst sein e- verliert. --88.68.84.155 23:23, 7. Feb. 2022 (CET) P.S.: Meine Lösung war dann, das zu substituierende Eisen zusammen mit Sanddorn zu geben und auf die Art konnte der Eisenmangel tatsächlich mit einem Minimum an per os Eisengabe ausgeglichen werden.
Strenge %-Zahlen kann ich Dir nicht sagen, aber die Hauptmenge an Eisen in der Nahrung liegt in dreiwertiger Form vor, das hängt sicher auch von diversen Umständen ab Aber ich schrieb: "Ascorbinsäure wandelt (reduziert) dreiwetiges Eisen in zweiwertiges, stabilisiert diese Form zudem" was ja dem entspricht, was Du frugst. --Elrond (Diskussion) 10:33, 8. Feb. 2022 (CET)
Habe mal selber nachgeschaut. Es ist wohl so, daß Nicht-Vegetarier ihren Bedarf vorwiegend über Häm-Eisen decken, das zweiwertig ist. Das dreiwertige Eisen bekommen wir aus Pflanzen, aber davon wird eher wenig resorbiert. Ascorbinsäure u.ä. benötigt der Körper, wenn er Fe3+ in Fe2+ umwandeln will. Zudem kann das der Stabilisierung des Fe2+ dienen. Die Reduktion dürfte daher nur bei den Personen im Vordergrund stehen, die wenig oder gar kein Fleisch essen und daher auf das dreiwertige Eisen aus Pflanzen besonders angewiesen sind. --178.4.182.69 02:43, 9. Feb. 2022 (CET)
Das ist nur zum Teil richtig. Das Eisen im Häm ist bei lebenden Wesen zweiwertig, wandelt sich aber in toten Tieren relativ schnell und in nennenswerten Mengen in dreiwertiges. Je stärker das Fleisch verarbeitet ist, desto höher ist, zumindest tendentiell, der Anteil des dreiwertigen Eisens. Daher ist Ascorbinsäure (& Co) auch bei Fleischessern nicht unwichtig. Und selbst wenn man einer lebenden Kuh in den Hintern beißt oder deren warmes Blut trinkt wird das Eisen beim Verdauen auch teilweise oxidiert. --Elrond (Diskussion) 13:44, 9. Feb. 2022 (CET)
<quetsch> Dazu hätte ich nun doch gerne Zahlen, in wie weit Fleischesser auf Ascorbinsäure angewiesen sind, um ihren Eisenbedarf zu sichern. Eine Vit. C Unterversorgung ist nicht leicht zu bewerkstelligen, wenn man noch etwas anderes als Schiffszwieback zu sich nimmt. Zumindest kenne ich keinen Fall von Eisenmangel infolge von Vit. C Unterversorgung, hingegen durchaus Eisenmangel infolge von Vegetarismus. Außerdem schützen die Porphyrin-Moleküle und die Globine das Fe2+ recht gut vor Oxidation, so daß ein Gutteil des Eisens aus Fleisch tatsächlich als Häm mit zweiwertigem Eisen resorbiert wird. --88.68.28.146 02:28, 10. Feb. 2022 (CET)
Das dürfte auch der Effekt sein, der beim Umröten des Myoglobins in Pökelwaren erwünscht ist. Yotwen (Diskussion) 13:59, 9. Feb. 2022 (CET)
Das ist etwas anderes. Beim Umröten wirde das Myoglobin in Nitrosomyoglobin umgewandelt, das eine stabile rote Farbe hat. Die Ascorbinsäure reduziert dabei das Nitrit zu NO und fördert so die Bildung des roten Farbstoffs. --Elrond (Diskussion) 14:53, 9. Feb. 2022 (CET)
Hauptsache, der Nettoeinsatz von Nitrit wurde reduziert. Yotwen (Diskussion) 15:39, 9. Feb. 2022 (CET)

Es harren der Wiederauferstehung...

Hallo und schönen guten Abend. Ich habe heute gemeinsam mit einer guten Freundin aus Kolumbien einen Friedhof in Berlin besichtigt. Auf einem der Grabsteine war, neben den Namen der Verstorbenen natürlich, folgende Formulierung zu lesen: "Es harren der Wiederauferstehung". Was diese Formulierung bedeutet (Die hier genannten Personen warten auf die Wiederauferstehung)konnte ich noch erklären. Nun würde mich aber total interessieren, wie diese grammatische Form mit "es" eigentlich heißt? Habe trotz angestrengten Guhgelns nichts finden können und auf Wikipedia auch bislang nichts. Wer kann mir helfen/ mich auf die richtige Fährte führen/ mir einen Tipp geben? Vielen herzlichen Dank! :)

--77.11.155.6 21:34, 5. Feb. 2022 (CET)

Ich glaube, ich habe es mit Hilfe der BKL ES gefunden: Expletivum. --MannMaus (Diskussion) 22:16, 5. Feb. 2022 (CET)
hey super, das muss es sein. Vielen Dank. Da habe ich mich wohl von dem alten Grabstein und der antik wirkenden Wortwahl ins Bockshorn jagen lassen, denn tatsächlich gibt es ja zwischen "Es kommt immer wieder zu Computer-Abstürzen" und "Es harren der Wiederauferstehung" keinen wirklichen Unterschied in der Grammatik. Wunder, was hatte ich mir ausgemalt, um was für eine archaische Form es sich wohl handeln könnte und dabei verwendet man es (haha!) jeden Tag. Herzlichen Dank nochmal, MannMaus. (und um ehrlich zu sein, hatte ich auch nicht damit gerechnet, dass eine Suche nach "Es" bei Wikipedia zu irgendeinem Ergebnis führen könnte und hatte deshalb einen dsbzgl Versuch unterlassen! XD) --77.11.155.6 22:28, 5. Feb. 2022 (CET)
Das halte ich für falsch. Ein Expletivum wäre das Subjekt des Satzes, dann wäre das Prädikat aber im Singular. Subjekt des Satzes sind die am Ende genannten Personen. Die Struktur ist in Subjekt (Grammatik)#Das unpersönliche Passiv mit dem Beispiel "Es hat jeder den gleichen Anteil bekommen" erklärt, auch wenn es sich gar nicht um umpersönliches Passiv handelt. --BlackEyedLion (Diskussion) 22:55, 5. Feb. 2022 (CET)
Der Satz hat zwei Besonderheiten: Einmal das Verb harren, das hier in gehobener Sprache in religiöser Bedeutung verwendet wird, vgl. zur Bedeutung dwds:harren, Bedeutung 5/6 und fwb-online:harren. Dann das „Es“ am Satzbeginn, das eine beliebige deutsche Grammatik als Platzhalter erklärt: (Zitat:) „Es“ ist hier also nur ein Platzhalter am Satzbeginn, das die Stelle vor dem konjugierten Verb besetzt. So wird es dem Subjekt oder einer Adverbialbestimmung, die normalerweise diese Position besetzen, ermöglicht, an eine andere Stelle im Satz zu rutschen. --Pp.paul.4 (Diskussion) 01:40, 6. Feb. 2022 (CET)
Hallo nochmal! :) Ich denke eigentlich, dass der zweite graue Kasten unter dem Abschnitt "Expletive im Deutschen" im Artikel "Expletive" die Formulierung beispielhaft beschreibt mit "es darf gelacht werden" und es sich also um ein so genanntes "Vorfeld-es" handelt. Oder liege ich da so falsch? --77.11.134.247 08:40, 6. Feb. 2022 (CET)
Es ist letztlich eine Frage der terminologischen Festlegung. Der zitierte Artikel nennt es ein "spezielles Expletivum", welches kein expletives Subjekt ist. Andere beschränken den Gebrauch des Begriffs Expletivum auf ebendieses expletive Subjekt und nennen das Platzhalter-s schlicht Platzhalter-s oder Vorfeld-s. Seine Funktion ist es ausschließlich, die Position vor dem Verb zu füllen, wenn sie nicht durch ein anderes Satzglied gefüllt werden kann oder soll ("Der Auferstehung harren..." wäre möglich, die Betonung des Objekts ist aber nicht gewünscht), da sich bei Frontstellung des Verbs im Deutschen die Syntax einer Frage ergibt ("Harren der Auferstehung...(?)"). Grüße Dumbox (Diskussion) 08:55, 6. Feb. 2022 (CET)
77.11.155.6, 77.11.134.247, die Unterschiede sind hier übersichtlich zusammengefasst. --Pp.paul.4 (Diskussion) 14:28, 6. Feb. 2022 (CET)
Zu "Es" wurde oben ja bereits viel geschrieben. Aber vielleicht ist es eher die Tatsache, dass das Verb "harren" mit einem Genitiv gebildet wird, die es Dir "archaisch" erscheinen lassen? Hier muss "Es" nicht Subjekt sein: "Die Grünen harren der Entscheidung ihres Koalitionspartners." Allerdings klingt das wirklich etwas aufgequollen. 80.71.142.166 04:36, 7. Feb. 2022 (CET)
„Es harren der Entscheidung ihres Koalitionspartners die Grünen.“
„Es“ ExpletivumVorfeld-Es, „der Entscheidung“ Objekt, „ihres Koalitionspartners“ Ergänzung zum Objekt, „die Grünen“ Subjekt.--Blue 🔯 09:05, 7. Feb. 2022 (CET)
Seltsames Deutsch. Ich würde sagen „Es harren die Grünen der Entscheidung ihres Koalitionspartners.“. --2003:D5:FF17:5000:5146:6ADB:93C2:E78 09:33, 7. Feb. 2022 (CET)
Natürlich geht das auch, aber in der Thread-Überschrift steht nach dem Verb ein Objekt.--Blue 🔯 10:54, 7. Feb. 2022 (CET)
Nein, das ist falsch. Wenn, dann ganz normal unexpletiv: „Die Grünen harren der Entscheidung ihres Koalitionspartners.“ --Rôtkæppchen₆₈ 02:24, 10. Feb. 2022 (CET)
Rotkäppchen, das stimmt nicht. Auch „Es harren die Grünen der Entscheidung ihres Koalitionspartners“ kann so gesagt werden, denn alle Wörter sind kongruent und alle Satzglieder stehen an einer erlaubten Stelle. Analog: „Es harren Franz und Peter der Wiederauferstehung.“ Korrekt.--Blue 🔯 11:38, 10. Feb. 2022 (CET)
Das ist aber kein expletives Es, sondern ein Vorfeld-Es. Bei einem expletiven Es steht das Prädikat notwendig im Singular. --BlackEyedLion (Diskussion) 14:39, 11. Feb. 2022 (CET)

Darf man Verdächtige als Mörder bezeichnen?

Guten Abend,

eine Frage zum Polizstenmord von RLP.

[20], hier schreibt die BILD sie haben den Mörder, darf die Zeitung dass?

Ich meine es ist doch noch gar nicht klar ob er der Mörder ist, hier wird sogar der Klarname verwendet, und er ist doch nur der Verdächtige? Wie ist die Rechtslage?--77.181.38.187 17:52, 31. Jan. 2022 (CET)

Pressekodex, Richtlinie 13.1 sagt: "Die Berichterstattung über Ermittlungs- und Gerichtsverfahren dient der sorgfältigen Unterrichtung der Öffentlichkeit über Straftaten und andere Rechtsverletzungen, deren Verfolgung und richterliche Bewertung. Sie darf dabei nicht vorverurteilen. Die Presse darf eine Person als Täter bezeichnen, wenn sie ein Geständnis abgelegt hat und zudem Beweise gegen sie vorliegen oder wenn sie die Tat unter den Augen der Öffentlichkeit begangen hat. In der Sprache der Berichterstattung ist die Presse nicht an juristische Begrifflichkeiten gebunden, die für den Leser unerheblich sind." Nach meinem Verständnis sind hier die Vorraussetzungen nicht gegeben, d.h. korrekt wäre die Bezeichnung "Tatverdächtiger", wie z.B. im SPIEGEL [21]. --Proofreader (Diskussion) 18:17, 31. Jan. 2022 (CET)

Und die nächsten Stufen wären dann "Beschuldigter" und "Angeklagter". Die Rechtslage ergibt sich allerdings nicht aus dem Pressekodex sondern aus dem Gesetz (vgl. unschuldsvermutung). --84.58.54.112 19:11, 31. Jan. 2022 (CET)
Weiter unten steht sogar in dem Blatt "Tatverdächtiger". Für die Schlagzeile könnte es sich vielleicht eine Rüge oder sogar Klage einfangen. Und darauf wird es - wie immer - scheißen. --46.79.42.178 18:44, 31. Jan. 2022 (CET)

Seit wann schert sich Bild (≠ Bild-Zeitung) um Recht und Gesetz? Oder Anstand? Gruß --Schniggendiller Diskussion 18:25, 31. Jan. 2022 (CET)

Die Bild ist glücklicherweise nicht mehr das Maß der Dinge. Auflage der Bild: Entwicklung der verkauften Auflage[1]Die Darstellung von Grafiken ist aktuell auf Grund eines Sicherheitsproblems deaktiviert. 80.71.142.166 18:45, 31. Jan. 2022 (CET)
  1. laut IVW, jeweils viertes Quartal (Details auf ivw.de)
  2. Und darunter jetzt bitte die Aufrufzahlen von bild.de und ihren Ablegern setzen. Printausgaben gehen überall zurück, das sagt null über relative Beliebtheit aus. --Kreuzschnabel 19:23, 31. Jan. 2022 (CET)
    (BK) Mach's dir mal nicht zu einfach. Die Reichweite der Bildzeitung, ebenfalls nach Statista, lag 2021 bei immer noch 7,82 Millionen. --Aalfons (Diskussion) 19:25, 31. Jan. 2022 (CET)
    Darf sie nicht. Macht sie aber, wie immer. Sie hat damit bekanntlich auch schon Unschuldige sozial zugrunde gerichtet, das ist ihr aber wurscht. Du kannst Klage beim Deutschen Presserat einreichen, dann erscheint übermorgen in der BILD in 5-Punkt-Schrift unterm Impressum eine Gegendarstellung. --Kreuzschnabel 19:18, 31. Jan. 2022 (CET)
    @Kreuzschnabel: Beim Deutschen Presserat legt man Beschwerde ein. [22] --2003:E7:BF06:A3CC:64EC:6C67:8FCC:25FB 21:07, 1. Feb. 2022 (CET)
    Von mir aus legt Gurken ein :) ist klar, was gemeint war, oder? --Kreuzschnabel 21:09, 1. Feb. 2022 (CET)
    Ich habe mir nach dem Tod Raimund Harmstorfs, den die Bild-Zeitung in den Tod getrieben hat, geschworen, nie wieder eine Bild-Zeitung anzurühren, und habe es bis heute eingehalten. 80.71.142.166 20:38, 31. Jan. 2022 (CET)
    Ja klar: Er nimmt Parkinsonmedikamente ein, die ihn zu einem Suizidversuch veranlassen, und sucht deswegen eine psychiatrische Klinik auf. BILD berichtet darüber. Anschließend nimmt er die Medikamente weiter ein und suizidiert sich. Völlig klar: BILD hat ihn in den Tod getrieben, und das schon vor einem Vierteljahrhundert! (An den Hexenprozessen im Mittelalter war BILD aber zufällig nicht schuld?) --95.116.29.20 21:51, 31. Jan. 2022 (CET)
    BILD hatte was damit zu tun: Raimund Harmstorf#Tod.--BanditoX (Diskussion) 22:50, 31. Jan. 2022 (CET)
    Da steht nichts von Hexenprozessen. Da wird nur mit substanzlosem Geraune versucht, einen zu inszenieren. --77.0.249.48 03:32, 1. Feb. 2022 (CET)
    Er kann nicht Klage beim Presserat einreichen. Klagen reicht man beim Salzamt ein. --95.116.29.20 21:36, 31. Jan. 2022 (CET)

    Der Pressekodex ist aber nur eine freiwillige Selbstverpflichtung, kein Gesetz. --MrBurns (Diskussion) 20:52, 31. Jan. 2022 (CET)

    Natürlich gilt im Prozess vor einem neutralen Richterkollegium die Unschuldsvermutung, das muss so sein und dürfte jedermann einsichtig sein. Wenn aber die Tat so klar ist wie hier, wo Selbsttötung, Unfall, Notwehr etc. ausgeschlossen ist, ist klar, dass es ein Mord ist, der aber dem Täter noch bei Gericht nachgewiesen werden muss. Der mutmaßliche Mörder könnte also jeden verklagen, der ihn als Mörder bezeichnet, das funktioniert aber auch nur bis zu seiner Verurteilung. Dazu muss er aber irgendwie in Erscheinung treten, versteckend auf der Flucht geht das nicht. Und wenn ein nur „des Mordes Beschuldigter“ solche eine Zeitung verklagt, muss er aufpassen, dass er dabei keine Aussagen macht, mit denen er sich selbst belastet. Außerdem würde man überlegen, ob solche Klage unter dem Aspekt der Verhältnismäßigkeit Erfolg hat. Noch ein Beispiel zu Schluss: Heinrich Himmler werden sicher alle nur als „mutmaßlichen Mörder“ bezeichnen, denn es ist durchaus fraglich, ob gegen ihn eine Beschuldigung wegen eines Tötungsvorwurfs erhoben und diese darüber hinaus noch als Mord bezeichnet worden wäre, wenn er nicht vorzeitig den Tod gefunden hätte. --2003:D0:2F18:57E0:84D:9E9D:F26:7E85 01:19, 1. Feb. 2022 (CET)

    Was für ein Unsinn. Selbstverständlich ist Heinrich Himmler ein Mörder gewesen, weil er gemeinschaftlich Massenmorde organisiert hat. Unser Artikel schreibt durchaus richtig: Er ist einer der Hauptverantwortlichen für den Holocaust, den Porajmos, die Ermordung von Millionen von Zivilisten und Kriegsgefangenen im Rahmen seines Generalplans Ost sowie für zahlreiche andere Verbrechen gegen die Menschlichkeit. Alles andere ist rechtsextreme Relativiererei. Es gibt auch so etwas wie ein historisches Urteil, das dazu berechtigt, einen Mörder als einen Mörder zu bezeichnen. Das gilt nicht nur für die Naziverbecher. Und in einem rechtsstaatlichen Kontext gibt es dieses Grundprinzip ohnehin erst seit 1948. Und noch etwas: Selbstverständlich gilt die Unschuldsvermutung auch außerhalb eines Strafprozesses bis zu einem anderslautenden rechtskräftigen Urteil. Eine Mißachtung kann sowohl strafrechtliche als auch zivilrechtliche Folgen haben bis hin zu der Frage der Unvoreingenommenheit von Gerichten. Drittens: Zum Zeitpunkt der Veröffentlichung war nichts so klar, wie du es darstellst. Die Fahndung war noch nicht abgeschlossen, um mögliche weitere Mittäter zu ermitteln. Diese Frage war noch offen. Und der Tathergang war nicht abschließend geklärt. So ziemlich alles an deinem Beitrag steht also auf tönernen Füßen. --84.58.54.112 04:37, 1. Feb. 2022 (CET)
    Bitte spar Dir krampfige Worthülsen wie "rechtsextreme Relativiererei", wenn Verharmlosung gemeint ist. Inflationärer Gebrauch von Kampfbegriffen führt für gewöhnlich dazu, daß keiner sowas mehr ernst nimmt. --88.68.87.67 04:04, 2. Feb. 2022 (CET)
    Wer Himmler als „mutmaßlichen Mörder“ darstellt betreibt das Geschäft der Rechtsextremen und Neonazis. Da kann man nicht deutlich genug werden und es ist Aufgabe von Demokratinnen und Demokraten, gegen den braunen Dreck zu kämpfen, auch in Wikipedia. Eine wehrhafte Demokratie kann getrost Rechtsextreme als Rechtsextreme und Relativierungen als Relativierungen benennen. Du magst das als „Worthülsen“ abwerten, ich meine es aber so wie ich es geschrieben habe und benötige da sichern nicht deinen Rat. --84.58.54.112 23:15, 3. Feb. 2022 (CET)
    Anscheinend doch, denn sowas steht Deinem Ansinnen im Weg, weil solche laxen Formulierungen unseriös wirken. Wie Du es gemeint hast, tut da nichts zur Sache. --178.4.180.189 02:10, 4. Feb. 2022 (CET)
    Es wird immer welche geben, auf die etwas nicht wirkt. Das tut für mich nichts zur Sache. Wenn sich aber jemand an der Formulierung "rechtsextreme Relativiererei" mehr stört als an dem Relativieren dieser Verbrechen an vielen Millionen Ermordeten, der sollte vielleicht lieber über Empathie und menschliche Verrohnung nachdenken als über Formulierungen. --84.58.54.112 05:56, 4. Feb. 2022 (CET)
    Noch zur Methode: Die Charakterisierung der mit „rechtsextrem“ und „relativieren“ formulierten Kritik als „Kampfbegriff“ oder „lax“ (was auch immer damit transportiert werden soll) und „unseriös“ versucht ja diese Begriffe zu entwerten, indem ihr Gebrauch als negativ besetzt hingestellt wird. Das Relativieren der ungeheuerlichen Naziverbrechen hat aber seit dem 8. Mai 1945 eine widerliche Evidenz bei Tätern und Mitläufern. Die kritisierende Auseinandersetzung damit ist anerkannter Bestandteil der historischen Aufarbeitung und des aktuellen politischen Diskurses. Was also ist die Intention, diese Begriffe umzudeuten und ihnen ihre Sinnhaftigkeit zu nehmen? --84.58.54.112 20:09, 4. Feb. 2022 (CET)
    Du denkst noch immer am Thema vorbei. Durch die Kombination von rechtsextrem & Relativiererei bewirkst Du genau das Gegenteil von dem, was Du erreichen möchtest. Letztlich ziehst Du damit die Naziverbrechen (und eben nicht die Naziverbrecherei) ins lächerliche. Das mag alleine an sprachlicher Unbeholfenheit liegen, darum will ich Deine Absichten nicht infrage stellen, aber wenn Deine Absichten lauter sind, solltest Du meinen Hinweis ernst nehmen. --94.219.12.84 00:52, 7. Feb. 2022 (CET)
    Du gehörst sicher nicht zu denen, die darüber entscheiden, über welche Themen ich nachzudenken habe. Du bist nicht der Nabel der Welt und deine Interpretation der gewählten Worte, das ziehe ins Lächerliche und überzeuge nicht, ist deine und keine allgemeingültige Sicht. Das mag daran liegen, dass du dich selbst wichtiger nimmst als du bist und gerne Leute überzeugst. Jemand, der die unglaubliche Dreistigkeit besitzt, Himmler als "mutmaßlichen" Mörder zu bezeichnen und dann widerlicherweise auch noch behauptet, das "werden sicher alle" so sehen, wirst du nicht mehr überzeugen. Mich interessieren aber nicht deine Probleme mit meiner Sprache. Mich interessiert viel mehr, warum - unabhängig von mir - hier nicht sofort ein Sturm der Entrüstung gegenüber IP 2003:D0:2F18:xxx losgebrochen ist - dich eingeschlossen. Mich erinnern deine Einwände an ein Interview mit Max Schmeling, den an der SS nur gestört hat, dass sie mit Stiefeln ins Theater gegangen sind... --84.58.54.112 18:06, 7. Feb. 2022 (CET)
    Genau das ist Dein Denkfehler. Hier ist ein Seite zur Beantwortung von Wissensfragen. Das hier ist kein Forum, keine Diskussionsseite und auch keine moralische Instanz. Aber jemand wie Du, der einen sachlichen Hinweis durchgehen mit persönlichen Angriffen kontert, ist vermutlich nicht dazu in der Lage, diesbezüglich zu einen ausreichend differenzierte Betrachtungsweise zu gelangen. Sich zu echauffieren ist keine Leistung und da Du offenbar nicht mehr kannst als das, wird es Dir auch zukünftig nicht gelingen, Dich irgendwie sinnvoll in die Debatte einzubringen, also reg Dich endlich ab, bevor Du der Sache noch mehr schadest. --88.68.84.155 00:16, 8. Feb. 2022 (CET)
    Es scheint tatsächlich so, dass du gerne Noten verteilst und anderen nicht weniger gerne ungebetene fordernde Ratschläge geben musst. Dein Umgangston ist völlig unangemessen und peinlich. EOD --84.58.54.112 18:05, 9. Feb. 2022 (CET)
    Du konterst einen sachlichen Hinweis mit einem Schwall an persönlichen Angriffen. Das bestätigt meine gewachsene Vermutung, daß Du nicht mehr kannst als Dich zu echauffieren. Solche Leute können wir hir nicht gebrauchen. Geh bitte woanders spielen. --88.68.28.146 03:49, 10. Feb. 2022 (CET)
    Du wirfst hier zwei unterschiedliche Sachen gefährlich durcheinander! Dass die Tat ein Mord ist, wird von niemandem bezweifelt und bestritten, und dass der oder die dafür Verantwortliche(n) Mörder ist/sind, bestreitet auch keiner. Aber solange nicht feststeht, dass die gestern festgenommene reale Person auch tatsächlich diejenige ist, die die Tat begangen hat oder daran beteiligt war, ist es nach allen mir erreichbaren Maßstäben vollkommen daneben, sie schon aufgrund dieser Festnahme als „Mörder“ zu bezeichnen. Ich persönlich erinnere mich mit Grausen an den Kindsmord in Emden 2012, wo ein 17-jähriger unter Tatverdacht festgenommen und nicht zuletzt aufgrund einer reißerischen BILD-Schlagzeile fast gelyncht wurde, bevor sich schließlich herausstellte, dass er keinesfalls der Täter war. --Kreuzschnabel 07:46, 1. Feb. 2022 (CET)
    Ob es Mord oder Totschlag oder was auch immer ist, muß erst einmal ermittelt werden. (Mal so als Denkanregung: - Die toten Polizisten wollten den (oder die) Täter erpressen / ausrauben / ermorden, die haben sich gewehrt. - Oder: Sie gingen von einer falschen Polizeikontrolle durch Fake-Polizisten aus. - Völlig absurd? Woher willst Du das wissen? Warst Du dabei? - Deshalb sind sorgfältige objektive Ermittlungen so wichtig.) Die Verteidigung ist übrigens deutlich im Vorteil: die Toten können keine Aussage mehr machen. Und deswegen raten Strafverteidiger Tatverdächtigen auch immer dazu, bei der Polizei keine Aussage zu machen. Ein strafmilderndes Geständnis können sie im Prozeß immer noch ablegen, da brennt nichts an, aber vorlaut geäußertes Täterwissen kriegt man nicht so einfach aus der Welt. --77.0.249.48 10:49, 1. Feb. 2022 (CET)
    Was soll das denn sein, der "Aspekt der Verhältnismäßigkeit"? Und wieso sollte es um eine Zivilklage gehen? --77.0.249.48 10:17, 1. Feb. 2022 (CET)

    Ja, darf man! Die Unschuldsvermutung gilt nur vor Gericht und dort auch nur im Strafverfahren. Medien wie der BLÖD sind Pressekodizes regelmäßig wurscht. GG und StPO gelten nur für den Staat, nicht für mich! Denn ich brauche mich an die Unschuldsvermutung nicht zu halten! --Heletz (Diskussion) 08:35, 1. Feb. 2022 (CET)

    Nein, darf man nicht einfach. Es gibt den Straftatbestand der üblen Nachrede. Wenn etwas noch nicht bewiesen ist und man es trotzdem über jemanden behauptet und das dessen Ruf schadet, ist das strafbar. Der BILD ist das natürlich egal, da die gute Anwälte bezahlen können die das schon auf höchstens eine Geldstrafe hinbiegen.--Naronnas (Diskussion) 08:57, 1. Feb. 2022 (CET)
    Und dann stellt sich natürlich noch die Frage, wer denn der mutmaßliche Täter bei der "üblen Nachrede" ist... --77.0.249.48 10:20, 1. Feb. 2022 (CET)
    Warum? Inwiefern? --Digamma (Diskussion) 20:54, 1. Feb. 2022 (CET)
    Weil man natürlich "die Zeitung" nicht vor Gericht stellen und strafrechtlich belangen kann, denn dafür braucht man Tatverdächtige, denen ihr Verschulden nachgewiesen werden muß. (Ich habe selbst stets sorgfältig darauf geachtet, daß von mir gefälsc "gestaltete" Fotos ohne Autorennennung abgedruckt wurden. Zum Glück sind die Leute meistens aber auch zu dämlich, zu erkennen, daß ein bestimmtes Foto gar nicht das zeigen kann, was die Bildunterschrift aussagt, weil in der vorgeblichen Situation das Foto gar nicht gemacht werden konnte. Bei Bildern geben sich die Leute i. a. damit zufrieden, daß sie stimmig aussehen. Meine Kunden wissen das natürlich, die sind ja nicht doof...) --77.8.217.248 11:11, 2. Feb. 2022 (CET)
    Der Autor eines Zeitungsartikels sollte wohl dennoch ermittelt werden können. Und der zuständige Redakteur dürfte wohl genauso verantwortlich sein. --Digamma (Diskussion) 19:42, 2. Feb. 2022 (CET)
    Guckssu Falschmeldung und Berichtigungsanspruch (Medienrecht): Die Redaktion des Mediums ist alsdann verpflichtet, eine Berichtigung der eigenen Erklärung abzugeben, welche geeignet ist, die Beeinträchtigung der Rechte des Betroffenen zu beseitigen. --88.68.87.67 22:04, 2. Feb. 2022 (CET)
    Und das hat jetzt doch gleich was nochmal mit dem Straftatbestand der "üblen Nachrede" und dessen Ahndung zu tun? Wenn wir da den Autor einerseits und den zuständigen Redakteur andererseits haben, dann sind das ja schon zwei, die sich den Schwarzen Peter gegenseitig zuschieben können, und "sollte wohl ermittelt werden können" klingt doch sehr verzweifelt nach "auf gut Glück" und "Weidmannsheil". Es gilt jedenfalls der alte Rechtsgrundsatz "Die Nürnberger hängen keinen, sie hätten ihn denn". --77.10.161.222 03:07, 6. Feb. 2022 (CET)
    So wird damit nunmal umgegangen nach geltendem Recht. Ob das "richtig" oder "falsch" ist, ist Ansichtssache und über Ansichten diskutieren wir hier nicht (s. Intro). --94.219.12.84 00:52, 7. Feb. 2022 (CET)
    Und daher ja auch die Frage, wer denn der mutmaßliche Täter bei der "üblen Nachrede" ist... --77.10.174.118 16:39, 12. Feb. 2022 (CET)

    Wer sind denn jetzt die Covid-gefährdeten Personen - wo bleiben die Daten?

    Alle "Alten" sind nicht die Risikopersonen. Dass alle gefährdet sind, hat sich als falsch herausgestellt (Symptomlose, Krankheitsverläufe). Aber wer ist durch eine Infektion mit dem SARS-Cov-2 tatsächlich gefährdet? Eine "Überlastung des Gesundheitswesens" ist eine irreführende Theorie, da Kapazitäten in kürzester Zeit geschaffen werden können und wurden. Und wie man mit Beschäftigten im Gesundheitswesen seit 40! Jahren umgeht (Bertelsmann Stiftung 2019 "Eine bessere Versorgung ist nur mit halb so vielen Kliniken möglich"), ist eine ganz andere, notwendige Diskussion, unabhängig von der aktuellen Situation. Maßnahmen der Gefahrenabwehr stehen unter dem Vorbehalt der Verhältnismäßigkeit und des Übermaßverbots: Legitimer Zweck, Geeignetheit, Erforderlichkeit, Angemessenheit. Mittlerweile sollten Daten vorliegen, die Legislative und Exekutive die Wahl der Mittel ermöglichen. Wo findet man die Daten (Open Data, Informationsfreiheitsgesetz? Was sagen die Daten aus?--Wikiseidank (Diskussion) 09:27, 6. Feb. 2022 (CET)

    Ist das ein Café-Beitrag?--Blue 🔯 09:45, 6. Feb. 2022 (CET)
    Auf jeden Fall ist das gedanklich noch viel zu ungeordnet, um eine Wissensfrage daraus zu erkennen. -- southpark 10:40, 6. Feb. 2022 (CET)
    Im Angesicht des Todes; die Plexiglasindustrie. Spätestens nächsten Monat.💅 (nicht signierter Beitrag von 89.204.154.172 (Diskussion) 11:14, 6. Feb. 2022 (CET))
    ich mache mal eine Wissensfrage draus: wenn man eine Studie mit 30000 Personen macht, die für die Studie täglich Nudeln essen sollen, dann wird man feststellen, dass täglich ein Studienteilnehmer verstirbt. Nudeln sind also nicht ganz ungefährlich. Analysiert man jetzt die Studiendaten dann stellt man fest, dass Nudeln zwar gefährlich sind, dass aber die Alten im besonderen Maße gefährdet sind und besonders häufig nach Nudelkonsum versterben. Was hier ziemlich skurril klingt, ist aber mathematisch erst einmal nicht falsch und es wird auch in dieser Weise immer wieder verwendet. Wo ist der Denkfehler und wie korrigiert ein Statistiker diesen Fehler? --77.81.139.182 11:33, 6. Feb. 2022 (CET)
    Man nimmt eine Kontrollgruppe von ebenso 30000 Personen, die als einzigen Unterschied statt Nudeln beispielsweise Reis essen und ansonsten dasselbe Alter/Geschlecht/Lebenstil/wasauchimmer vorweisen. --91.141.45.121 11:43, 6. Feb. 2022 (CET)
    Dann verstirbt auch jeden Tag einer. Ist Reis genaus gefährlich wie Nudeln?? --79.249.3.109 20:19, 10. Feb. 2022 (CET)
    Gefährdet sind vor allem Ungeimpfte. Da ist Einiges an Daten verlinkt. --46.78.147.226 13:10, 6. Feb. 2022 (CET)
    "Selbst unabhängig von Covid-19 gilt das: Geimpfte sterben selbst dann weniger als Ungeimpfte, wenn man Covid-19 Tote nicht berücksichtigt. Das liegt vermutlich daran, dass sie auch sonst sich eher um ihre Gesundheit kümmern." - Ja, eben. Um die Aussage zu belegen, daß Ungeimpfte ein höheres Sterberisiko haben, müßte man hinreichend viele Impfwillige ohne deren Wissen mit einem Placebo-Vakzin "impfen". Lt. Kekulé hat nämlich die Immunantwort auch ziemlich viel mit Placebo- und Nocebo-Effekten zu tun, weswegen er auch der Ansicht ist, daß man Impfgegnern mit panischer Angst vor "der Spritze" auch ohne weiteres und medizinisch-psychiatrisch wohlbegründet eine Impfunfähigkeit attestieren sollte. --77.10.161.222 18:18, 6. Feb. 2022 (CET)
    Ja und? Spritzenphobien, Angst vor den Wirkstoffen, echte oder eingebildete Unverträglichkeiten, Noceboeffekte usw. gibt nicht erst seit Corona. Seit Corona gibt es aber besonders viele Leerköpfe, die laut "Freiheit!" schreien, aber genau diese Freiheit für die Impfung den Leuten auszureden versuchen (bei manchen leider erfolgreich). Da gilt es gegenzuhalten und aufzuklären. --46.78.147.226 19:36, 6. Feb. 2022 (CET)
    Unter der Fahne der Aufklärung, aber mit unsauberen Argumenten? Man merkt die Absicht, und man ist verstimmt. (Wie kann man nur so dämlich sein: wer so dumm argumentiert, daß man direkt merkt, daß es nicht um Information, sondern um Indoktrination geht, verliert komplett seine Glaubwürdigkeit und schadet seiner Sache sogar dann, wenn er eigentlich im Recht ist. Noch besser kann man seinen Kontrahenden doch gar nicht in die Hände spielen.) --77.10.161.222 22:20, 6. Feb. 2022 (CET)
    "mit unsauberen Argumenten" - Du (77.10.161.222) hast offensichtlich damit angefangen. Als ob der positive Effekt von einer Impfung reine Placebo sei. Wo ist das jetzt nochmal Aufklärung oder gut belegt? --37.4.249.100 09:08, 10. Feb. 2022 (CET)
    Du hast offensichtlich ein Problem mit der Attribuierung der Beweispflicht. Auch sind die von Dir unterstellten unsauberen Argumente a.a.O. nicht ersichtlich. --77.10.174.118 16:50, 12. Feb. 2022 (CET)
    Die Eingangsfragen lauten: Wo findet man die Daten? Was sagen die Daten aus? Das kann man ganz nüchtern beantworten mit: beim RKI. Die sind dafür zuständig, eben solche Daten zu sammel, auszuwerten und die Ergebnisse für jedermann leicht nachvollziehbar darzustellen. Diskussionen über Sinn und Unsinn der Frage sind hier fehl am Patze (s. Intro) und zu unterlassen. --94.219.12.84 01:02, 7. Feb. 2022 (CET)
    Allgemeiner: Solche Daten findet man in Fachpublikationen. Gemeint sind Artikel in doppelt-blind begutachteten Fachzeitschriften, für welche die Autoren ein Studiendesign gewählt haben, dass entsprechende Rückschlüsse zulässt. Ein Beispiel für eine solche Fachzeitschrift ist The Lancet. Zu beachten ist, dass solche Datenquellen nicht immer in Deutschland zu finden sind. In Dänemark kann man bspw. auf ein komplett durchdigitalisiertes Gesundheitssystem zurückgreifen. Theoretisch kann man dort sogar Regressionen zwischen der Länge von Gefängnisaufenthalten, dem Besuch bestimmter Schulbezirke, der Einnahme bestimmter Medikamente vor 10 Jahren und der Länge des Krankenhausaufenthaltes im Rahmen von Covid-19 berechnen. Solche Auswertungen wären aufgrund des Datenschutzes in Deutschland undenkbar. Auch aus dem Vereinigten Königreich liegen viele Daten vor, die wir in Deutschland einfach nicht sammeln dürfen. Aber auch in Deutschland profitiert man natürlich von solchen Auswertungen. Man muss natürlich aufpassen: In Dänemark sind 96% aller 60+ geimpft. Das sind eigentlich alle, die man impfen kann. In Deutschland sind es nur etwa 75%. Die Folge: In Dänemark sind durch die hohe Impfrate die Krankenhäuser ausreichend leer, sodass man komplett lockern kann. Davon sind wir in Deutschland leider durch die vielen Impfmuffel noch weit entfernt. Dass es tatsächlich sehr stark am Alter liegt, kann man aus den Daten aus Dänemark (Abrufbar vom Statens Serum Institut) und UK sehr gut erkennen. Übrigens sollte man nicht mehr allgemein von Covid-19 sprechen: Omikron ist ein ganz anderes Virus als es Delta war. Man muss also Daten gewissermaßen für Omikron neu auswerten. 80.71.142.166 04:26, 7. Feb. 2022 (CET)
    Das letzte ist sehr wichtig. Im großen und ganzen stirbt man an Omikron nicht, auch nicht ungeimpft, weswegen die Hexenjagd auf die "Impfmuffel" einfach grotesk ist. (Wenn das Impfen wirklich so wichtig wäre, dann sollte man doch übergangsweise die beiden chinesischen und das indische Coronavakzin freigegeben - auch millionenfach erprobt und bewährt - anstatt die Wagenknechts und andere dazu zu zwingen, auf Valneva warten zu müssen. (Der Frau Wagenknecht geht es erfreulicherweise gut, sie ist jetzt zwar immer noch nicht geimpft - sagt sie - aber "genesen".) Mit Valneva kann man dann später immer noch boostern. Und niemandem ist es verboten, sich mit den bisher zugelassenen Vakzinen impfen zu lassen.) --77.8.86.111 08:58, 7. Feb. 2022 (CET)
    FunFact: Das chinesische Zeugs hilft schon mal so gut wie gar nicht gegen Omikron, würde also nix nutzen. Nach dem indischen habe ich noch nicht geschaut, würde aber ähnliches vermuten. --88.68.84.155 00:27, 8. Feb. 2022 (CET)
    Naja, der Gruppe der Impfmuffel ist es leider eben doch zu verdanken, dass die Maßnahmen, anders als in Dänemark, weiterhin nicht aufgehoben werden können. Es ist inzwischen bekannt, dass Ungeimpfte bei Omikron zwar deutlich seltener auf die Intensivstation müssen als noch bei Delta. Aber dafür ist (1) Omikron deutlich ansteckender (wodurch die Intensivstationen eben dennoch stark belastet sind) und (2) ungeimpfte landen mit Omikron immer noch sehr oft auf Normalstationen (was das Gesundheitssystem ebenfalls stark belastet). Es ist bekannt, dass eine Impfung bei Omikron in vielen Fällen vor einer Ansteckung schützt (wenngleich nicht so verlässlich wie bei Delta) und die Krankheitslänge und Krankheitsintensität und damit die Ansteckung Dritter herabsetzt. Hätte Deutschland eine vergleichbare Impfquote wie Dänemark, könnte man auch in Deutschland schon jetzt die Maßnahmen deutlich verringern oder sogar aussetzen. Insofern ist das natürlich keine Hexenjagd gegenüber Impfmuffeln, aber eben doch eine Frustration, denn an den Geimpften liegt es ja nun einmal nicht, dass man weiterhin mit Masken rumrennen muss und in der Freiheit eingeschränkt wird. Aber leider hat ja kein Appell an diese kleine Gruppe etwas gebracht. Im Gegenteil, das Geschreie dieser kleinen Gruppe wird z.T. ja immer lauter. 80.71.142.166 18:09, 7. Feb. 2022 (CET)
    Völliger Quark. Auf Intensiv befinden sich kaum noch Covid-Patienten seit Omikron Delta verdrängt hat. Dafür füllen sich die wegen der Freihalteprämien massiv unterbelegten Normalstationen langsam wieder etwas. Den Pflegemangel, der uns aktuell Probleme macht, hatten wir allerdings schon vor Corona - das ist politisch so gewollt und kann deswegen kein Grund für eine Impfpflicht sein. --88.68.84.155 00:27, 8. Feb. 2022 (CET)

    Zwischenantwort?: Es gibt (für Deutschland) keine Daten, aus denen hervorgeht, welche Eigenschaften eine Person zu einer Corona-Covid19-SARSCov2-Omikron Risikoperson macht. (beim RKI habe ich nichts derartiges gefunden)--Wikiseidank (Diskussion) 09:36, 7. Feb. 2022 (CET)

    Bloß, weil Du das nicht findet, heißt das noch lange nicht, daß es das dort nicht gibt. Die meisten anderen finden das dort nämlich. Das ist aleo eine gute Gelegenheit für Dich, sowas mal zu üben. --88.68.84.155 00:27, 8. Feb. 2022 (CET)
    Dann setze doch bitte einen (Beispiel) Link, danach frage/suche ich doch. So geht es.--Wikiseidank (Diskussion) 08:29, 8. Feb. 2022 (CET)
    Wie kommst du auf die Idee, dass man im Voraus genau sagen könnte, wer durch Corona gefährdet wird? Ob eine Krankheit für jemanden lebensgefährlich oder gar tödlich ist, hängt von ganz vielen Dingen ab. Was für eine Art von Daten erwartest du? --Digamma (Diskussion) 18:57, 8. Feb. 2022 (CET)
    Guck mal auf seine Benutzerseite. Dann siehst Du, was sein Ansinnen ist. --178.4.182.69 03:02, 9. Feb. 2022 (CET)
    Ich habe Dir schon viel zu oft gezeigt, wie man sowas macht. Du bist schlicht unwillig, selber zu recherchieren. Dafür ist die Auskunft nicht gedacht. --178.4.182.69 03:02, 9. Feb. 2022 (CET)
    Bitte kein Trolling von Nummern-IPs oder Café
    "Risikoperson": Wie unser Artikel schreibt, eine ungenaue/falsche Bezeichnung, gemeint sind eigentlich "gefährdete Personen". Gefahr meint (runtergebrochen) Wahrscheinlichkeit des Eintritts plus Größe/Erheblichkeit des Schadens. Die Erheblichkeit des Schadens ist bei einer Erkrankung (der Atemwege), ab dann gegeben, wenn eine stationäre Behandlung erforderlich ist. Es geht also um die Erkenntnis bzw. Analyse der Personen, die aufgrund des Corona-Covid19-SARSCov2-Omikron-Virusses eine stationäre Behandlung benötigen.
    "Was für eine Art von Daten erwartest du?" beispielsweise: Alter, biologisches Geschlecht, gesamter Impfstatus, Arten von Vorerkrankungen, Berufsgruppen, Land/Stadt, Bindung in sozialen Gruppen (Familie, Heim, Teams), Datum der Erkrankung, Ort der Infektion (Kontaktnachverfolgung), usw.--Wikiseidank (Diskussion) 20:35, 9. Feb. 2022 (CET)
    Ich ergänze zum zweiten Absatz: Daten in Bezug auf das Immunsystem
    • Viruslast? wie gut wird der Start der Infektion verhindert, und kann das beim selben Individuum sehr unterschiedlich verlaufen, weil das Individuum gerade ständig trockene Schleimhäute in Nase und Mund hat?
    • Genetische Voraussetzungen, Ernährung
    • Das Risiko bei Covid19 ist ja die Überreaktion des Immunsystems ("Zitokinsturm"). Es gibt sicherlich auch ne Handvoll Wissenschaftler, die sich die Frage stellen, warum der bei manchen Menschen aufkommt und bei vielen nicht. (Allerdings scheint auch das Spikeprotein selbst schädlich zu wirken, auch wenn ich dazu auch recht wenig lese und dementsprechend unsicher diese Aussage von mir ist, siehe auch WP:AU/Archiv). --37.4.249.100 09:08, 10. Feb. 2022 (CET)